Está en la página 1de 281

09/10/2020 PRIMER EXAMEN PARCIAL - COMISIONES UDH

PRIMER EXAMEN PARCIAL -


COMISIONES UDH
* Required

Preguntas Múltiple choice

- Seleccione en cada una de las preguntas la opción CORRECTA o INCORRECTA según lo solicitado.
- RECUERDE marcar sólo 1 opción.

Un hombre de 55 años presenta episodios de dolor torácico subesternal grave al


realizar tareas que demandan un esfuerzo moderado. Los episodios han
aumentado en frecuencia y gravedad en el último año, mejorando con
nitroglicerina sublingual. No tiene fiebre y la frecuencia cardíaca es de 78 latidos
x’, regular. No se auscultan soplos ni ritmo de galope. Las pruebas de laboratorio
muestran: creatinina 1,1 mg/dl; glucosa 130 mg/dl y colesterol sérico total de 223
mg/dl. ¿Cuál de estas lesiones cardíacas es más probable que presente este
paciente? *

ateroesclerosis coronaria

pericarditis serosa

miocardiopatía restrictiva

estenosis aórtica calcificada

Acorde a la Clasificación 2015 de la WHO, los Mesoteliomas se dividen en: *

Mesotelioma maligno solitario / Mesotelioma maligno desmoplásico

https://docs.google.com/forms/d/e/1FAIpQLSdf7CxAkIJqFmGGOWxot1ysNUp9QfHW-0du8LMta0Rk6CsRgg/formResponse 1/16
09/10/2020 PRIMER EXAMEN PARCIAL - COMISIONES UDH

Mesotelioma maligno localizado / Mesotelioma maligno difuso

Mesotelioma maligno mesenquimático / Mesotelioma maligno desmoide

Mesotelioma maligno difuso / Mesotelioma maligno sistémico

Un hombre de 32 años con antecedentes de enfermedad de Crohn ha notado la


aparición de un nódulo rojo doloroso de 5cm., rodeado de una zona pálida en la
parte inferior de la pierna izquierda. Su temperatura es de 37,3°C. Esta lesión se
resuelve en las 3 semanas siguientes, pero aparece otra en la pantorrilla
contralateral. En la biopsia se observa “infiltrado inflamatorio mixto en la dermis
con neutrófilos, células redondas y células gigantes, que afectan al tejido
adiposo, además de intenso edema”. Las lesiones se resuelven sin cicatriz,
aunque siguen apareciendo cíclicamente. ¿Cuál es el diagnóstico más probable?
*

eritema nodoso

impétigo

molusco contagioso

dermatitis herpetiforme

Una mujer de 33 años sufrió un accidente automovilístico en el cual su pecho


derecho golpeó contra el volante. Un mes después del accidente presenta una
tumoración en la mama de 3 cm no dolorosa. ¿Cuál de los siguientes es el
diagnóstico más probable? *

Absceso

Carcinoma ductal

Necrosis grasa

Mastopatía fibroquística

https://docs.google.com/forms/d/e/1FAIpQLSdf7CxAkIJqFmGGOWxot1ysNUp9QfHW-0du8LMta0Rk6CsRgg/formResponse 2/16
09/10/2020 PRIMER EXAMEN PARCIAL - COMISIONES UDH

¿Cuál de los siguientes enunciados es CORRECTO en referencia a la Enfermedad


Trofoblástica Gestacional? *

El 90% de las molas completas tienen un cariotipo 46XX y se ha demostrado por


estudios citogenéticos que los cromosomas derivan exclusivamente de la madre.

El coriocarcinoma gestacional tarda bastante tiempo en infiltrar.

Comprende a la mola hidatiforme completa, la mola hidatiforme parcial, la mola


invasiva, el coriocarcinoma y el tumor trofoblástico del lecho placentario.

El concepto "degeneración hidrópica de vellosidades coriales avasculares" define al


coriocarcinoma.

En relación al melanoma marque la opción CORRECTA: *

Son tumores de potencial maligno indeterminado que pueden recidivar pero no hacen
metástasis.

Son tumores malignos con diferenciación melanocítica y que carecen de la mutación


activadora del BRAF.

Son tumores malignos con diferenciación melanocitica y que presenta la mutación


activadora del BRAF.

Es un tumor benigno y por lo tanto no tiene capacidad de hacer metástasis.

Con respecto a la entidad nosológica “Cardiopatía isquémica”, señale cuál de las


siguientes afirmaciones es INCORRECTA: *

Todas las entidades relacionadas a estos síndromes se caracterizan por presentar


isquemia miocárdica.

En más del 90% de los casos se relaciona con ateroesclerosis de las coronarias
epicárdicas.

Entre las complicaciones tempranas del Infarto Agudo de Miocardio se debe tomar en
https://docs.google.com/forms/d/e/1FAIpQLSdf7CxAkIJqFmGGOWxot1ysNUp9QfHW-0du8LMta0Rk6CsRgg/formResponse 3/16
09/10/2020 PRIMER EXAMEN PARCIAL - COMISIONES UDH

cuenta el aneurisma ventricular.

La consecuencia histopatológica del Infarto Agudo de Miocardio es la sustitución de


las fibras necróticas por cicatriz colágena densa.

Una oclusión total de la arteria coronaria descendente anterior provoca IAM en *

Cara anterior del ventrículo izquierdo y 2/3 del tabique interventricular.

Cara anterior de ventrículo izquierdo y 1/3 anterior del tabique interventricular.

Cara anterior del ápex cardiaco.

Región subendocárdica de la pared anterior del ventrículo izquierdo.

Paciente de sexo femenino de 76 años de edad que presenta en región malar


izquierda una lesión cutánea nodular, de bordes poco definidos e irregulares,
rojiza, rugosa, descamativa, con ulceración central, de 8 meses de evolución. Se
realiza una biopsia con sacabocado del borde de la lesión. Al microscopio, se
observan células neoplásicas atípicas que se extienden desde la epidermis hacia
la dermis, poligonales, con abundante citoplasma eosinófilo. Las mismas se
organizan formando nidos con extensas zonas de queratinización (perlas
córneas). A mayor aumento se aprecia la presencia de puentes intercelulares
entre células tumorales vecinas. Esta lesión puede originarse a partir de una
lesión preneoplásica. ¿Cómo se denomina a dicha lesión preneoplásica cuando
asienta sobre mucosa labial? *

Queilitis actínica

Nevo displásico

Queratosis actínica

Queratoacantoma

https://docs.google.com/forms/d/e/1FAIpQLSdf7CxAkIJqFmGGOWxot1ysNUp9QfHW-0du8LMta0Rk6CsRgg/formResponse 4/16
09/10/2020 PRIMER EXAMEN PARCIAL - COMISIONES UDH

Un hombre de 64 años, fumador de 800 paquetes/año, presenta tos y ha perdido


5 kg de peso en los últimos 3 meses. La exploración física demuestra
acropaquias digitales. Está afebril. La Rx. de tórax no muestra adenopatías
hiliares, pero se identifica una cavitación con una lesión de 3 cms. cercana al hilio
derecho. Las pruebas de laboratorio reflejan una calcemia de 12,3 mg/dl, una
concentración de fósforo de 2,4 mg/dl y de albúmina de 3,9 mg/dl. La
broncoscopia muestra una lesión que ocluye casi por completo el bronquio
principal derecho. Se trata de resecar la neoplasia con intención curativa. El
estudio inmunohistoquímico de la neoplasia resecada muestra CK5/6(+); p63(+);
p40 (+); CK7(-); TTF1(-) y sinaptofisina(-). ¿Cuál de las siguientes neoplasias es la
más probable en este paciente? *

Carcinoma indiferenciado de células grandes

Adenocarcinoma in situ

Carcinoma indiferenciado de células pequeñas

Carcinoma epidermoide

Una paciente de 60 años tuvo molestias abdominales difusas y pérdida de 12kg


de peso en 2 meses. En la TAC abdominal se observa una gran masa
retroperitoneal, que comprimía y desplazaba las vísceras abdominales.
“Macroscópicamente, era una masa lobulada, pseudocapsulada de 25x15cm de
coloración blancoamarillenta, e histológicamente mostraba una proliferación de
células redondas, con citoplasma claro vacuolado y núcleos ovoideos periféricos
anisocarióticos, hipercromáticos”. Tinción con Sudán(+). Inmunomarcación:
Proteína S100(+), Actina y Desmina(-)”.¿Cuál de las siguientes opciones es
CORRECTA para esta entidad nosológica? *

Sus diferentes tipos no constituyen una enfermedad uniforme, por tanto, el


conocimiento de sus alteraciones moleculares permitirá el diagnóstico y el
tratamiento más adecuado.
https://docs.google.com/forms/d/e/1FAIpQLSdf7CxAkIJqFmGGOWxot1ysNUp9QfHW-0du8LMta0Rk6CsRgg/formResponse 5/16
09/10/2020 PRIMER EXAMEN PARCIAL - COMISIONES UDH

Es uno de los sarcomas más frecuentes de la infancia.

La variante histológica pleomórfica es la más frecuentemente observada.

La velocidad de aparición de las metástasis no está en relación con el tamaño, la


localización (superficial – profundo) o el grado histológico tumoral.

Paciente de 60 años, diabética, hipertensa y obesa, consulta a su ginecólogo por


presentar metrorragia de 2 meses de evolución. Luego de examinarla, se solicita
una ecografía que informa útero globoso, en el que se observa una masa de
bordes indefinidos y heterogéneos de aproximadamente 1x3x1.5cm. Se realiza
una biopsia que confirma el diagnóstico de carcinoma endometrial tipo 1. ¿Con la
alteración de que gen se asocia la aparición de este tumor? *

CHD4

Rb

BCRA1

PTEN

En relación al Adenocarcinoma de Pulmón. Señale la opción INCORRECTA: *

La hiperplasia adenomatosa atípica se considera como una lesión precursora que


mide <5mm y que se caracteriza histológicamente por poseer neumocitos displásicos
que revisten las paredes alveolares acompañado de ligera fibrosis intersticial.

El Adenocarcinoma in situ se describe como una lesión que mide ≤3cm, con un
patrón de crecimiento lepídico, que puede o no producir mucina pero no evidencia
invasión estromal, de la pleura visceral o linfovascular.

El TTF-1 es el marcador de mayor especificidad y sensibilidad para diferenciar al


Adenocarcinoma de otros carcinomas.

Generalmente la topografía de la lesión es central.

https://docs.google.com/forms/d/e/1FAIpQLSdf7CxAkIJqFmGGOWxot1ysNUp9QfHW-0du8LMta0Rk6CsRgg/formResponse 6/16
09/10/2020 PRIMER EXAMEN PARCIAL - COMISIONES UDH

Paciente de sexo masculino de 15 años de edad que presenta una lesión


osteoblástica voluminosa ubicada en metáfisis de tibia proximal, con destrucción
de la cortical ósea y compromiso de tejidos blandos adyacentes. Al microscopio,
se observa marcado pleomorfismo celular con elementos fusiformes y
producción de abundante material osteoide eosinófilo que se dispone en forma
de encaje. Referente a esta patología, señale la opción CORRECTA. *

El tipo más frecuente asienta en metáfisis de huesos largos, es primario,


intramedular, osteoblástico y de alto grado.

El tipo más frecuente asienta en diáfisis de huesos largos, es primario, intracortical,


osteoblástico, y de bajo grado.

El tipo más frecuente asienta en metáfisis de huesos largos, es secundario,


intracortical, osteoblástico y de bajo grado.

El tipo más frecuente asienta en diáfisis de huesos largos, es secundario,


intramedular, osteoblástico y de alto grado.

En referencia a los "Tumores de vulva", marque el enunciado INCORRECTO. *

La mayoría de los cánceres vulvares no están relacionados con el VPH y aparecen


sobre un liquen escleroso o una hiperplasia de células escamosas a partir de una
lesión premaligna llamada neoplasia intraepitelial vulvar diferenciada (VIN
diferenciada).

El tipo histológico más frecuente de carcinoma vulvar es el adenocarcinoma.

Cerca del 30% de los cánceres de vulva están causados por infecciones por VPH de
alto riesgo, principalmente VPH-16.

El carcinoma de vulva es una neoplasia maligna infrecuente.

https://docs.google.com/forms/d/e/1FAIpQLSdf7CxAkIJqFmGGOWxot1ysNUp9QfHW-0du8LMta0Rk6CsRgg/formResponse 7/16
09/10/2020 PRIMER EXAMEN PARCIAL - COMISIONES UDH

¿Cuál de las siguientes patologías es frecuente que curse con complicaciones


como uronefrosis y pielonefritis en estadios avanzados de la enfermedad? *

Adenocarcinoma de ovario

Carcinoma escamoso de vagina

Adenocarcinoma de endometrio

Carcinoma de cérvix

¿Cuál de las siguientes afirmaciones describe morfológicamente a la entidad


nosológica “Endocarditis infecciosa”? *

Vegetaciones asépticas, múltiples, adheridas a la línea de cierre. Rara vez tienen más
de 0.5 cm de tamaño. Sin embargo, son friables y muy propensas a embolizar.

Vegetaciones pequeñas, asépticas, <5mm a ambos lados de las valvas.

Masas irregulares, friables y generalmente voluminosas sobre las cúspides valvulares


que se extienden sobre las cuerdas tendinosas.

Una hilera de pequeñas (1-2mm) vegetaciones verrugosas a lo largo de las líneas de


cierre de las valvas.

https://docs.google.com/forms/d/e/1FAIpQLSdf7CxAkIJqFmGGOWxot1ysNUp9QfHW-0du8LMta0Rk6CsRgg/formResponse 8/16
09/10/2020 PRIMER EXAMEN PARCIAL - COMISIONES UDH

En referencia a la Enfermedad de Paget ósea, señale cuál de los siguientes


enunciados es INCORRECTO: *

La mayoría de los pacientes suelen presentar dolores óseos generalizados y se


detecta por fracturas patológicas.

Empieza por lo general en la edad adulta (media de 70 años al momento del


diagnóstico) y aumenta progresivamente de frecuencia a partir de esta edad.

Puede dividirse en tres fases secuenciales: una fase osteolítica inicial, una fase
osteoclástica/osteoblástica y por último una fase osteoesclerótica.

Esta enfermedad es monostótica en el 15% de los pacientes y poliostótica en el resto.

Una paciente de 76 años con polimialgia reumática es derivada de Oftalmología


por pérdida súbita de la visión hace 5 días, que resultó en ceguera de ambos
ojos. Al interrogatorio, la paciente refiere episodios de cefaleas, dificultad para
peinarse por dolor al roce y claudicación mandibular. Se palpan nódulos en la
arteria temporal izquierda. ¿Qué patología sufre esta paciente y qué se
encontraría en la biopsia? *

Arteritis de Kawasaki / Infiltrado transmural denso

https://docs.google.com/forms/d/e/1FAIpQLSdf7CxAkIJqFmGGOWxot1ysNUp9QfHW-0du8LMta0Rk6CsRgg/formResponse 9/16
09/10/2020 PRIMER EXAMEN PARCIAL - COMISIONES UDH

Panarteritis nodosa / Infiltrado transmural con necrosis fibrinoide

Arteritis de células gigantes / Inflamación granulomatosa en la lámina elástica


interna

Arteritis de Takayasu / Inflamación granulomatosa y necrosis de la media

Un hombre de 76 años presenta un nódulo en el párpado superior derecho que


ha ido aumentando de tamaño lentamente durante los últimos 2 años. En la
exploración física se identifica lesión de bordes perlados y telangiectasias de 0,8
cm de diámetro mayor. La microscopía de la lesión describe una proliferación
nodular de células de aspecto basaloide con empalizada nuclear periférica y halo
de retracción. ¿Cuál es el diagnóstico más probable? *

Carcinoma epidermoide

Carcinoma basocelular

Queratosis seborreica

Lentigo

¿Cuál de los siguientes enunciados es INCORRECTO en referencia a los


Timomas? *

Los Timomas B2 remedan la cortical tímica y tienen fuerte relación con la miastenia
gravis.

Los Timomas tipo A son encapsulados, están constituidos por células fusiformes que
remedan la medular y son de buen pronóstico.

El carcinoma escamoso tímico suele presentarse no encapsulado, con áreas de


necrosis y hemorragia.

Los Timomas B3 generalmente son tumores mal diferenciados, encapsulados y no


invasivos.

https://docs.google.com/forms/d/e/1FAIpQLSdf7CxAkIJqFmGGOWxot1ysNUp9QfHW-0du8LMta0Rk6CsRgg/formResponse 10/16
09/10/2020 PRIMER EXAMEN PARCIAL - COMISIONES UDH

Paciente de sexo femenino de 36 años de edad que presenta un nódulo


dominante en lóbulo tiroideo derecho. Se realiza una ecografía y se informa
nódulo hipoecoico, sólido-quístico de 45mm de diámetro, de bordes difusos,
con microcalcificaciones y aumento del flujo sanguíneo intranodular. Los
extendidos citológicos obtenidos mediante punción-aspiración con aguja fina
del nódulo muestran células con núcleos de aspecto ópticamente vacío, con
hendiduras nucleares, surcos intranucleares (grooves) y pseudoinclusiones
nucleares. Dichas células se disponen conformando pequeños folículos. ¿Cuál de
los siguientes considera Usted es el diagnóstico más probable? *

Carcinoma folicular de tiroides.

Adenoma folicular de tiroides.

Carcinoma medular de tiroides.

Carcinoma papilar de tiroides.

En referencia al "Cáncer de mama”, marque el enunciado INCORRECTO: *

Los carcinomas asociados a expresión de receptores de estrógenos representan un


grupo infrecuente y agresivo de cáncer de mama, pero susceptible de tratamiento con
tamoxifeno.

Los carcinomas negativos para receptores de estrógeno, receptores de progesterona


y Her2neu ("triple negativos"), son más frecuentes en mujeres premenopáusicas.

El índice histológico de Nottingham mide formación de túbulos, atipia nuclear, y


mitosis.

BRCA1 es uno de los principales genes de susceptibilidad involucrados en cáncer de


mama familiar.

https://docs.google.com/forms/d/e/1FAIpQLSdf7CxAkIJqFmGGOWxot1ysNUp9QfHW-0du8LMta0Rk6CsRgg/formResponse 11/16
09/10/2020 PRIMER EXAMEN PARCIAL - COMISIONES UDH

Respecto a las cardiopatías congénitas, señale la opción INCORRECTA: *

La existencia de comunicación interauricular incrementa el riesgo de sufrir


potenciales infartos en distintos órganos debidos a una embolia paradójica.

Las malformaciones que causan una derivación del flujo sanguíneo desde el lado
derecho al lado izquierdo del corazón se caracterizan por una hipoxia y consecuente
cianosis que se expresa clínicamente en forma temprana.

La infección congénita por rubéola, la diabetes gestacional y la deficiencia de ácido


fólico son factores ambientales implicados en la aparición de cardiopatías
congénitas.

La tetralogía de Fallot consiste en una comunicación interventricular, con la aorta


situada sobre el tabique interventricular, hipertrofia del ventrículo izquierdo y
estenosis de la arteria pulmonar.

A un paciente de 60 años, fumador de larga data, que consulta por disfonía, se le


realiza una laringoscopia, observándose una lesión exofítica sobre la cuerda
vocal derecha. La biopsia informa "Carcinoma Laríngeo". Marque la opción
CORRECTA: *

La mucosa adyacente a la lesión infiltrante puede mostrar hiperplasia, displasia o


carcinoma in situ.

Los carcinomas glóticos tienen mal pronóstico ya que su sintomatología es tardía.

Frecuentemente el carcinoma se origina en los llamados nódulos laríngeos, nódulos


reactivos o pólipos, relacionados con grandes esfuerzos laríngeos.

El 95% corresponden histológicamente a adenocarcinomas.

https://docs.google.com/forms/d/e/1FAIpQLSdf7CxAkIJqFmGGOWxot1ysNUp9QfHW-0du8LMta0Rk6CsRgg/formResponse 12/16
09/10/2020 PRIMER EXAMEN PARCIAL - COMISIONES UDH

Usted se encuentra cursando cirugía general en la UDH. Le solicitan que realice


una clase sobre causas de hiperparatiroidismo primario. Durante la búsqueda y
lectura bibliográfica encuentra que las principales causas de esta afectación son
el adenoma paratiroideo, la hiperplasia paratiroidea y el carcinoma paratiroideo.
Durante el armado de la clase se pregunta cuáles son las principales
características que se tiene que tener en cuenta para diferenciar estas entidades
entre sí. Marque la opción CORRECTA. *

Tanto el carcinoma como la hiperplasia atípica no generan metástasis.

El carcinoma se diferencia del adenoma por su capacidad de invadir los tejidos


circundantes.

El carcinoma se diferencia del adenoma por sus características citológicas y número


de mitosis.

El adenoma se diferencia del carcinoma, porque generalmente afecta múltiples


glándulas.

Un paciente de 43 años con tuberculosis tratada hace 10 años presentó fiebre,


tos, expectoración y decaimiento general. En la Rx de tórax se observó una
imagen densa (nódulo pulmonar solitario). En la biopsia guiada por TAC, se
observó “fenómeno inflamatorio de tipo granulomatoso acompañado por un
conglomerado de filamentos delgados y tabicados, ramificaciones en ángulo
agudo, las técnicas de tinción especiales resultaron positivas para PAS y
GROCOTT. ¿Cuál es para usted el diagnóstico más probable? *

Aspergilosis

Candidiasis

Mucormicosis

Actinomicosis

https://docs.google.com/forms/d/e/1FAIpQLSdf7CxAkIJqFmGGOWxot1ysNUp9QfHW-0du8LMta0Rk6CsRgg/formResponse 13/16
09/10/2020 PRIMER EXAMEN PARCIAL - COMISIONES UDH

En referencia al "Feocromocitoma”, marque el enunciado CORRECTO: *

Su diagnóstico de malignidad se basa en las características cito-histológicas e


inmunohistoquímicas.

El 30% son unilaterales y malignos.

No están relacionados con mutación de la línea germinal que causa los síndromes
MEN de tipo II.

Son tumores poco frecuentes que generan hipertensión, corregible quirúrgicamente.

¿Cuál de las siguientes asociaciones entre tipo de Pericarditis y etiología es


CORRECTA? *

Pericarditis purulenta / radiación, neoplasias y fiebre reumática.

Pericarditis serosa y serofibrinosa / tuberculosis.

Pericarditis serosa / enfermedades autoinmunitarias e infecciones virales.

Pericarditis hemorrágica / infarto agudo de miocardio, síndrome de Dressler,


radiación.

Una paciente de 40 años concurre a la consulta porque se ha palpado un nódulo


en la tiroides. Usted solicita una punción por aspiración con aguja fina. A la
citología, observa múltiples células foliculares atípicas a especificar, por lo que
decide realizar una biopsia quirúrgica intraoperatoria. ¿Qué elementos de la
biopsia deberá tener en cuenta para poder realizar el diagnóstico correcto? *

Invasión capsular y vascular

Fibrosis

Células de Hürthle
https://docs.google.com/forms/d/e/1FAIpQLSdf7CxAkIJqFmGGOWxot1ysNUp9QfHW-0du8LMta0Rk6CsRgg/formResponse 14/16
09/10/2020 PRIMER EXAMEN PARCIAL - COMISIONES UDH

Atipia marcada

Respecto a la Osteoporosis, señale la opción INCORRECTA: *

Reducción de estrógenos séricos

Aumento de la expresión de RANK/RANKL

Aumento de la concentración de citoquinas proinflamatorias

Consumo de carnes rojas en exceso

Marque la INCORRECTA en referencia a la entidad infecciosa relacionada con el


SARS-COV2: *

Entre los trastornos fisiopatológicos graves observados en autopsias se encuentran


la endotelitis, trombosis y angiogénesis.

El SARS-COV2 pertenece a un grupo de coronavirus al igual que el MERS-COV y SARS-


COV1.

En la actualidad se desconoce el huésped intermediario de este agente etiológico.

En el contexto de la respuesta inmune a la infección por SARS-COV2 se identifican a


las IL-1β, IL-6 y TNFα como moléculas beneficiosas.

¿Cuál de los siguientes enunciados es CORRECTO respecto a los adenomas


hipofisarios? *

Son necesarias las técnicas inmunohistoquímicas para la adecuada clasificación y


diagnóstico de los adenomas hipofisarios.

La microscopía electrónica es necesaria para el diagnóstico y clasificación actual de


los adenomas de hipófisis.

El estudio inmunohistoquímico de TTF-1 es de utilidad en el diagnóstico de tumores


de adenohipófisis.

Es frecuente su malignización
https://docs.google.com/forms/d/e/1FAIpQLSdf7CxAkIJqFmGGOWxot1ysNUp9QfHW-0du8LMta0Rk6CsRgg/formResponse 15/16
09/10/2020 PRIMER EXAMEN PARCIAL - COMISIONES UDH
Es frecuente su malignización.

Usted es un residente en la especialidad de Anatomía Patológica y es llamado por


el equipo de ginecología para ir al quirófano por una consulta intraoperatoria. La
paciente tiene 38 años de edad, es obesa y presenta un blastoma anexial
derecho de 11x6x5 cm, que en la ecografía mostró áreas hiperecogénicas en su
interior. La superficie de la pieza que le entregan es lisa y congestiva, y al cortarla
nota múltiples áreas sólidas mientras rezuma un material amarillento viscoso con
algunos pelos. De acuerdo a la macroscopía de la pieza, que resulta muy
característica, usted ya tiene un diagnóstico presuntivo. Respecto de esta
entidad, responda: *

El CA-125 es diagnóstico en este tipo de lesiones.

Es un tumor del cordón sexual.

Su extirpación genera el descenso inmediato de β-HCG.

Su comportamiento biológico suele ser benigno.

Back Next

Never submit passwords through Google Forms.

This form was created inside of Facultad de Medicina. Report Abuse

Forms

https://docs.google.com/forms/d/e/1FAIpQLSdf7CxAkIJqFmGGOWxot1ysNUp9QfHW-0du8LMta0Rk6CsRgg/formResponse 16/16
2° RECUPERATORIO 1° EXAMEN PARCIAL
- 2°cuatrimestre 2020
*Obligatorio

Preguntas Múltiple choice

- Seleccione en cada una de las preguntas la opción CORRECTA o INCORRECTA según lo solicitado.
- RECUERDE marcar sólo 1 opción.

El feocromocitoma se asocia con todos los siguientes síndromes, excepto: *

Neoplasia endocrina múltiple 2b (MEN 2b).

Neoplasia endocrina múltiple 1 (MEN 1).

Síndrome paraganglioma-feocromocitoma hereditario.

Neoplasia endocrina múltiple 2a (MEN 2a).

Una mujer de 32 años, está cursando la semana 39 de gestación y detecta que


no percibe movimientos fetales desde hace un 1 día. El feto nace muerto por
parto vaginal y no presenta malformaciones externas. El examen microscópico
de la placenta detecta una corioamnionitis aguda. ¿Cuál es el agente infeccioso
más frecuentemente involucrado? *

Un parásito

Un virus

Una bacteria

Un hongo
/
Paciente de sexo masculino de 52 años de edad con antecedentes de dermatitis
recidivantes, y biopsias previas con diagnóstico de eczema. Actualmente
presenta lesiones eritematoescamosas, levemente sobreelevadas, de bordes
irregulares, situadas en espalda y dorso de brazos. En la biopsia por sacabocados
realizada por el dermatólogo se presenta un infiltrado en banda en dermis
papilar constituido predominantemente por linfocitos, y epidermis con mínima
acantosis y leve espongiosis, con algunos infiltrados de células con aspecto
linfoide, que en sectores formaban microabscesos. Se realizaron técnicas de
inmunohistoquímica con destacado predominio de marcación positiva con CD4
en las células del mencionado infiltrado. Responda, ¿cuál es el diagnóstico? *

Enfermedad de Bowen.

Micosis fungoide.

Dermatitis crónica.

Psoriasis.

¿Cuál es la asociación más significativa en todo el mundo del sarcoma de


Kaposi? *

Edad avanzada.

Síndrome de inmunodeficiencia adquirida (sida).

Infección por VIH fundamentalmente en los primeros estadios.

Trasplante.

/
Una mujer de 37 años ha notado una masa de tamaño creciente en la mama
izquierda durante los últimos 2 años. La paciente atribuye su crecimiento al roce
el corpiño en dicho sector. La ginecóloga le palpa una masa firme de 2cm de
diámetro. Se le efectúa una nodulectomía. Macroscópicamente se reconoce una
masa firme-elástica, blanquecina nacarada, de límites netos que la separan del
tejido adiposo circundante. Histológicamente se halla constituida por una
proliferación epitelial canalicular y un estroma mixoide. ¿Cuál es el diagnóstico
más probable? *

necrosis grasa por traumatismo

fibroadenoma

carcinoma medular

carcinoma lobulillar

¿Qué estudio complementario solicitaría para diagnosticar la etiología de una


paciente que presenta Síndrome de Cushing y cuya RMN de hipófisis y TAC de
suprarrenales son normales, y que niega uso de corticoides exógenos? *

Ecografía de vías biliares

Ecocardiograma transtorácico

Radiografía de tórax

Radiografía de abdomen

/
Paciente femenina de 25 años, concurre a la consulta con el informe de anatomía
patológica de la conización que le realizaron el mes pasado. Ésta informa células
escamosas inmaduras que abarcan 2/3 de la totalidad del epitelio cervical.
¿Cómo se clasifica este tipo de neoplasia intraepitelial cervical? *

CIN III

L-SIL

H-SIL

CIN I

Con respecto a la Patología Endocrina señale la opción CORRECTA: *

En la enfermedad de Graves se observan estructuras papilares dentro de los folículos


tiroideos revestidas por células con núcleos claros o vacíos.

Los únicos criterios fiables son la invasión de tejidos adyacentes y metástasis.

En la tiroiditis de Hashimoto se observan células metaplásicas de Hürthle y presencia


de infiltrado inflamatorio mixto compuesto por linfocitos con abundantes
polimorfonucleares.
/
Los trastornos hipofisarios se relacionan con aumento o descenso en los niveles de
hormonas hipofisarias o con efecto de masa.

Una mujer de 40 años lleva 3 meses con lumbalgia. Refiere tos crónica durante 2
años. En el examen físico se observa hipersensibilidad a la palpación en las
vértebras lumbares, sin calor, tumefacción ni eritema. La Rx. de columna revela
una fractura por compresión a nivel de L2. La TAC abdominal identifica un
absceso en el psoas derecho. ¿Cuál de los siguientes microorganismos es más
probable que se identifique como causa de estos hallazgos? *

Cryptococcus neoformans

Treponema pallidum

Mycobacterium tuberculosis

Staphylococcus aureus

En referencia a los tumores mucinosos de ovario, señale la opción CORRECTA. *

Se producen principalmente antes de la pubertad.

La mutación del prooncogen KRAS es una alteración genética presente en la mayoría


de estos tumores.

Representan el 80% de todas las neoplasias del ovario.

El 75% son tumores malignos.

¿Cuál de las siguientes características corresponde al carcinoma de endometrio


tipo I (endometrial)? *

d) Es más frecuente que el carcinoma de endometrio tipo II (seroso).

Es similar al carcinoma seroso de ovario.

Se da a edades mayores que el carcinoma de endometrio tipo II (seroso).

Derivar de la atrofia endometrial.


/
Una mujer de 22 años presenta desde hace un año un nódulo pardo claro en la
pierna izquierda. En el examen físico se trata de un nódulo firme e indoloro de
0,7 cm. Se extirpa y su estudio histológico revela una proliferación de células
fusiformes localizada en la dermis, con hiperplasia de la epidermis suprayacente
y elongación en sentido descendente de unas crestas epidérmicas
hiperpigmentadas. ¿Cuál es el diagnóstico más probable? *

Leiomioma

Carcinoma de células de Merkel

Dermatofibroma

Schwannoma

¿Qué molécula caracteriza a las células neoplásicas del carcinoma medular de


tiroides? *

Tiroglobulina.

Mieloperoxidasa.

Melan A.

Calcitonina.

En relación a la Patología Intersticial Pulmonar, señale la opción INCORRECTA: *

la correlación de la histología con los hallazgos clínicos, antecedentes, e imágen


tomográfica; es esencial para el diagnóstico.

tienen en común mostrar un patrón obstructivo en las pruebas respiratorias.

muchas de estas entidades son de etiología desconocida.

representan un grupo heterogéneo de enfermedades caracterizadas por inflamación y /


fibrosis del intersticio.

Osteoma Osteoide. Marquen la opción CORRECTA. *

Generalmente su tamaño es menor a 2cm y afecta láminas y pedículos vertebrales.

Un 80% sufre transformación maligna.

Imagen lítica mayor a 2 cm, asintomática, constituido por trabéculas óseas


reticulares separadas por tejido fibroso laxo.

Imagen lítica con “nido” central, dolorosa, constituido por trabéculas óseas reticulares
separadas por tejido fibroso laxo.

La principal causa de la miocardiopatía hipertrófica es: *

Mutación del gen de la B-miosina

Pericarditis constrictiva

Estenosis mitral por fiebre reumática

Hipertensión arterial

Un hombre de 75 años ha desarrollado una lumbalgia con dolor de ambas


caderas y del hombro derecho progresivos durante los últimos 3 años. Refiere
que ahora usa sombreros de mayor tamaño. Las radiografías muestran un
estrechamiento del espacio articular con esclerosis del hueso adyacente. La
biopsia de hueso de la cresta ilíaca revela pérdida de las trabéculas normales con
un patrón en mosaico y aumento del número de osteoclastos y osteoblastos.
¿Cuál de las siguientes complicaciones es más probable que presente el paciente
como consecuencia de este proceso? *

Espondilitis anquilosante

Osteoma osteoide

Displasia fibrosa
/
Osteosarcoma

Paciente masculino de 35 años sin antecedentes patológicos de importancia,


consulta por episodios reiterados de sintomatología inespecífica durante las
últimas semanas (aislado cuadro febril, fatiga y pérdida de peso) y actualmente
por molestias oculares con un significativo debilitamiento del pulso en las
extremidades superiores. Una angiotomografía reveló irregularidad del contorno
de la aorta torácica con estenosis luminal a nivel de las ramas principales del
cayado. Una biopsia de uno de los vasos comprometidos reveló un
engrosamiento pronunciado intimal asociado a destrucción y fibrosis de la capa
media con presencia de células gigantes mononucleares, infiltrado
linfoplasmocitario y fibrosis de la adventicia. Ante su sospecha, podría tratarse
de: *

Vasculopatía hipertensiva.

Panarteritis nudosa.

Arteritis de Takayasu.

Arteritis de Kawasaki.

En referencia a la "Patología Pericárdica", señale cuál de los siguientes


enunciados es CORRECTO. *

La Pericarditis Purulenta se produce únicamente como consecuencia de la


diseminación hematógena desde un foco piógeno a distancia.

La Pericarditis Serosa es característica en el contexto de una TBC.

Las metástasis de melanoma en pericardio pueden identificarse por


inmunohistoquímica (HMB45, S100, MelanA), confirmando el diagnóstico al obtener
positividad en 2 de ellas.

Los diferentes tipos de pericarditis evolucionan a la organización con fibrosis y su


consecuente pericarditis constrictiva.

/
Los pacientes con melanomas metastásicos fallecen casi todos a los 5 años,
excepto que presenten alguna alteración que permitan incluirlos en terapia de
blanco con buena respuesta. Marque ¿cuál es la mutación molecular presente
mas común en un melanoma que nos autoriza a incluirlo en plan de terapia de
blanco/target o especificamente dirigidas? *

p53

BRAF

CDK4

PTEN

En relación a los tumores cardiacos marce la opción CORRECTA. *

Ninguno de los enunciados anteriores es correcta.

El rabdomioma es el tumor secundario benigno más frecuente en pacientes


pediátricos.

El angiosarcoma es el tumor secundario maligno más frecuente.

El mixoma es el tumor primario maligno más frecuente.

Mujer de 30 años con antecedentes de radioterapia en cabeza y cuello, concurre


a realizar control ecográfico tiroideo periódico, durante el cual se detecta un
nódulo de 1,5cm de diámetro. Se realiza PAAF; observándose en el extendido
citológico planchas de “células con núcleos ovoides, solapados, algunos de los
cuales presentan surcos o hendiduras, y otros pseudoinclusiones o vacuolas
intranucleares”. ¿Cuál es para Ud. el diagnóstico más probable? *

Carcinoma papilar

Carcinoma anaplásico
/
Carcinoma medular

Hiperplasia nodular

Se presenta a la consulta una paciente de 65 años con una lesión escamosa de


coloración rojiza en la mejilla derecha que mide 1cm. de diámetro. Se realiza
biopsia y el estudio histopatológico de la misma revela: células epiteliales
grandes (poligonales) con anisocariosis y nucléolo evidente con distorsión de la
histoarquitectura normal de la epidermis, presencia de mitosis en todo el
espesor del epitelio y algunas áreas de disqueratosis. ¿Cuál es para Ud. el
diagnóstico más probable? *

Queratosis actínica

Queratosis seborreica

Queratoacantoma

Carcinoma epidermoide in situ

Un paciente obeso de 63 años de edad, ex tabaquista severo, con tos y


producción de esputo la mayor parte de los días en los últimos cuatro años,
hipertenso y con historia de episodios cardiovasculares, es ingresado a la
guardia de un sanatorio por una descompensación súbita y fallece luego de 2
horas de recibir una atención médica adecuada. Sus familiares solicitan una
autopsia médica que confirma un infarto masivo del miocardio. Entre los
hallazgos más relevantes consignados en el informe de la autopsia los patólogos
observaron que los macrófagos alveolares estaban cargados con pigmento
antracótico, y que en la tráquea y los bronquios el epitelio respiratorio tenía un
incremento de células caliciformes y disminución de ciliadas, con áreas de
metaplasia escamosa, hipertrofia de glándulas submucosas, infiltrado
inflamatorio predominantemente linfocitario y fibrosis; además muchos
bronquíolos estaban tapados por material mucoso. ¿Cuál de las siguientes
entidades corresponde a la descripción realizada? *

Bronquitis crónica.

Carcinoma escamoso.

Fibrosis pulmonar idiopática. /


Bronquiectasia.

En referencia a la entidad nosológica “Patología pleural”, indique la opción


CORRECTA *

Las pleuritis hemorrágicas se presentan generalmente en el contexto de las


colagenopatías.

Hasta el momento no hay explicación fisiopatológica certera para el derrame pleural


de origen cardíaco.

Las colecciones serosas no inflamatorias están asociadas a patología neoplásica


maligna.

El Lupus eritematoso sistémico y la artritis reumatoidea son causa de pleuritis con


exudado seroso y serofibrinoso.

Respecto de las "Neoplasias óseas", señale cual/es de los siguientes enunciados


es INCORRECTO: *

El osteoma es un tumor benigno, formador de hueso, que se origina de la superficie


de la corteza ósea.

La exostosis es una alteración consistente en crecimiento de hueso laminar con un


recubrimiento de cartílago hialino, que prolifera hasta generar células con marcada
atipia.

El osteosarcoma se caracteriza por una proliferación de células osteoblásticas


malignas, pleomórficas, y presencia de matriz osteoide o de hueso maduro.

El osteoblastoma se localiza predominantemente a nivel de la columna vertebral.

Marque la INCORRECTA en referencia a la entidad infecciosa relacionada con el


SARS-COV-2: *

En la fisiopatogenia viral se reconoce la unión de la proteína S con el receptor de


ACE2 (enzima convertidora de angiotensina 2) de la célula huésped.

En el contexto de la infección, la respuesta inmune innata y adaptativa se presentan


de manera controlada y acordes no generando daños tisulares a nivel local o
sistémico. /
La neutrofilia y la linfopenia son indicadores de mal pronóstico.

Los microtrombos, la capilaritis, los infiltrados subepiteliales y la bronconeumonía


bacteriana, son hallazgos posibles en autopsias de pacientes COVID (+).

Sobre el hiperpituitarismo y adenomas hipofisarios, marque la opción


INCORRECTA: *

La causa más frecuente de hiperpituitarismo es un adenoma en el lóbulo anterior.

Las anomalías genéticas asociadas a los adenomas hipofisarios se relacionan con


las mutaciones de la proteína G.

Los adenomas hipofisarios pueden ser funcionales, produciendo más de una


hormona.

La causa más frecuente de hiperpituitarismo son los carcinomas hipofisarios

Un hombre de 56 años se somete a una radiografía de tórax de rutina como


parte de un examen físico completo. La Rx muestra una lesión única de moneda
solitaria, localizada en el centro, con un patrón de calcificación de “palomitas de
maíz”. Se realiza una biopsia de pulmón que revela nódulos de cartílago maduro
benigno y epitelio respiratorio (ver imagen) ¿Cúal es el diagnóstico más
probable? *

Hamartoma pulmonar.

Leiomioma. /
Tumor carcinoide.

Fibroma pulmonar.

Señale la opción INCORRECTA con respecto al cáncer de mama. *

Los carcinomas asociados a expresión de receptores de estrógenos representan un


grupo infrecuente y agresivo de cáncer de mama, pero susceptible de tratamiento con
tamoxifeno.

Los carcinomas positivos para Her2neu, presentan una amplificación del gen.

BRCA1 es uno de los principales genes de susceptibilidad involucrados en cáncer de


mama familiar.

El carcinoma lobulillar presenta una morfología con discohesividad celular debido a


la pérdida de expresión de E-cadherina, y muestra un patrón de diseminación hacia
cavidad abdominal y leptomeninges.

Una mujer de 54 años presenta durante una semana febrícula y tos. Los síntomas
mejoran gradualmente en la semana siguiente, pero luego concurre a una nueva
consulta con fiebre más intensa, tos, disnea y malestar general. En el examen
físico presenta temperatura de 37,9°C y se auscultan rales crepitantes
inspiratorios. La Rx. de tórax muestra áreas alveolares parcheadas, pequeñas y
bilaterales de consolidación. Se le toma una biopsia transbronquial que
demuestra bronquíolos con tapones polipoides de tejido de granulación y tejido
fibroso laxo y un infiltrado intersticial de células mononucleares. Es tratada con
corticosteroides y el cuadro clínico mejora. ¿Cuál de los siguientes diagnósticos
es más probable? *

bronquiectasias

neumonitis intersticial descamativa

neumonía organizada criptogénica

proteinosis alveolar pulmonar


Paciente femenina de 40 años, consulta por dispareunia y metrorragia
intermenstrual. Como antecedente refiere que le hicieron una conización por
“cáncer de cuello uterino” hace 3 años. La anatomía patológica de la pieza
informa márgenes libres de lesión. Habiendo descartado una recidiva del
carcinoma de cuello de útero, ¿qué otra lesión podría esta paciente presentar
con mayor probabilidad? *

Carcinoma de vagina

Carcinoma endometrial

Cistoadenocarcinoma ovárico

Carcinoma escamoso de vulva

Determine a qué tipo de miocardiopatía se corresponden las siguientes


imágenes: *

Miocardiopatía Hipertrófica

Miocardiopatía Restrictiva

Miocardio No Compactado

Miocardiopatía Dilatada

/
La evolución progresiva de la necrosis por coagulación de los miocitos, la
aparición brusca de un infiltrado polimorfonuclear constituído por neutrófilos y la
pérdida de núcleos y estriaciones de los miocitos son cambios morfológicos que
pueden observarse durante la evolución de un infarto de miocardio. ¿A qué
período de tiempo corresponden los cambios mencionados? *

1 a 2 semanas.

3 a 7 días.

12 a 72 horas.

1 hora.

Siguiente

/
2° RECUPERATORIO 1° EXAMEN PARCIAL
- 2°cuatrimestre 2020
*Obligatorio

Casillas de verificación

- Marcar todas las opciones CORRECTAS.


- PRESTAR MUCHA ATENCIÓN ya que deben marcar TODAS LAS CORRECTAS - siempre CORRECTAS-, si
marcan correctas de menos o incorrectas, el sistema anula la pregunta.

En referencia a la “Patologia respiratoria” señale cual/es de los siguientes


enunciados es CORRECTO. Recuerde que deberá marcar todas las opciones
correctas considerando que el marcar correctas de menos o 1 respuesta
incorrecta, dará por anulado el ejercicio: *

En el síndrome de dificultad respiratoria aguda el cuadro histológico característico


consiste en vasos con paredes gruesas en el intersticio pulmonar.

La variante histológica más frecuente de cáncer de pulmón es el adenocarcinoma.

En las atelectasias por reabsorción, una fibrosis pulmonar focal o generalizada


impide la expansión completa del pulmón.

El enfisema es una enfermedad restrictiva crónica de las vías respiratorias.

La antracosis consiste en pigmento de carbón que es deglutido por los macrófagos


alveolares o intersticiales.

El asma es una enfermedad de ambientes industrializados con muchos


contaminantes aéreos que actúan como alergenos.

/
En referencia a la “Patología cardiovascular” señale cual/es de los siguientes
enunciados es correcto. Recuerde que deberá marcar todas las opciones
correctas considerando que al marcar correctas de menos o 1 respuesta
incorrecta, dará por anulado el ejercicio: *

La mayoría de los casos de cardiopatía isquémica se deben a ateroesclerosis arterial


coronaria.

En la miocardiopatía hipertrófica se produce una reducción del volumen sistólico por


alteración del llenado diastólico.

En la angina inestable no hay fenómenos de agregación plaquetaria involucrados.

En un infarto de miocardio las alteraciones típicas de necrosis coagulativa se hacen


detectables a las 48hs.

La hipertensión provoca hipertrofia por sobrecarga de volumen del ventrículo


izquierdo.

En la endocarditis infecciosa las vegetaciones de las válvulas presentan


microorganismos.

/
En referencia a los “Tumores malignos del estroma endometrial y miometrio”
señale cual/es de los siguientes enunciados es correcto. Recuerde que deberá
marcar todas las opciones correctas considerando que al marcar correctas de
menos o 1 respuesta incorrecta, dará por anulado el ejercicio: *

Los adenosarcomas predominan en mujeres entre la cuarta y la quintas décadas y


generalmente se consideran de bajo grado de malignidad.

Los sarcomas del estroma endometrial son tumores malignos de bajo grado, con baja
tasa de recidiva, cuya probabilidad puede ser estimada por la atipia citológica y el
índice mitótico del tumor.

Los tumores del estroma endometrial son relativamente infrecuentes, comprenden


menos del 5% de los cánceres de endometrio e incluyen neoplasias estromales puras
y asociadas con glándulas benignas (adenosarcomas).

La mayoría de los leiomiosarcomas tienen cariotipos normales, pero


aproximadamente el 40% tiene un anomalía cromosómica simple asociada con la
fusión del gen JAZF1.

Los adenosarcomas se presentan con mayor frecuencia como grandes crecimientos


polipoides endometriales que pueden prolapsar a través del orificio cervical. El
principal diagnóstico diferencial es con pólipos grandes benignos.

Para arribar al diagnóstico de leiomiosarcoma, la presencia de 10 o más mitosis por


10 campos de alta magnificación (400×) indica malignidad, particularmente si se
acompaña por atipia citológica y/o necrosis.

En referencia a la “Enfermedad de Graves”, señale cual/es de los siguientes


enunciados es CORRECTO. Recuerde que deberá marcar todas las opciones
correctas, considerando que el marcar correctas de menos o 1 respuesta
incorrecta, dará por anulado el ejercicio. *

El incremento difuso del tamaño de la tiroides, el exoftalmos y el mixedema pre-tibial


son parte de la tríada clínica clásica.

A nivel histológico las células foliculares epiteliales en pacientes sin tratamiento son
altas y estan más concentradas de lo habitual.

Tiene una incidencia máxima entre los 50 y los 60 años de edad.

Suele afectar hasta 10 veces más a mujeres que a hombres

/
Es la causa más frecuente de hipertiroidismo endógeno.

La presencia de células gigantes multinucleadas englobando lagunas o fragmentos


de coloide es un hallazgo histológico característico.

En referencia a los "Tumores óseos y Tumores de partes blandas", señale cual/es


de los siguientes enunciados es CORRECTO. Recuerde que deberá marcar todas
las opciones correctas, considerando que el marcar opciones correctas de
menos o 1 respuesta incorrecta, dará por anulado el ejercicio. *

Las localizaciones más frecuentes de los tumores de partes blandas se encuentran


en miembros inferiores, tronco y retroperitoneo.

La mayoría de los tumores de partes blandas se asocian a Sindromes genéticos.

El liposarcoma bien diferenciado o tumor lipomatoso atípico de las extremidades es


la variante de liposarcoma más frecuente.

Para el diagnóstico presuntivo de tumores óseos es importante conocer la edad del


paciente y contar con estudios por imágenes de la lesión.

El Mieloma Múltiple es una neoplasia de células plasmáticas que provoca


exclusivamente lesiones en sacabocado en la calota.

g) Las MTS óseas provienen con mayor frecuencia de tumores de mama, pulmón,
tracto urinario y próstata.

En referencia a la “PSORIASIS", señale cual/es de los siguientes enunciados es


CORRECTO. Recuerde que deberá marcar todas las opciones correctas,
considerando que el marcar opciones correctas de menos o 1 respuesta
incorrecta, dará por anulado el ejercicio. *

Es una dermatosis que produce escamas blanquecinas sobre placas salmón-rosado


bien delimitadas, en los planos de extensión (codo, rodillas), la nuca, cuero cabelludo
y pliegues interglúteos

En la histología hay hipergranulosis, disqueratosis, acantosis regular con


aplanamiento de las crestas epidérmicas interpapilares y un infiltrado inflamatorio
mononuclear en banda en la dermis papilar.

Las lesiones establecidas de psoriasis tienen un cuadro histológico característico.

Es una dermatosis inflamatoria crónica con una probable base autoinmunitaria

Aproximadamente el 15% de los pacientes con psoriasis tienen una artritis asociada.

Esta dermatosis estaría mediada por anticuerpos IgE. /


2° RECUPERATORIO 2° EXAMEN PARCIAL
- 2°cuatrimestre 2020
*Obligatorio

Preguntas Múltiple choice

- Seleccione en cada una de las preguntas la opción CORRECTA o INCORRECTA según lo solicitado.
- RECUERDE marcar sólo 1 opción.

Mujer de 43 años se presenta a la consulta con dolor lumbar en el ángulo


costovertebral, T Axilar 40°C, escalofríos y malestar general. Usted hace
diagnóstico clínico de Pielonefritis aguda. Respecto de esta patología, ¿cuál de
las siguientes afirmaciones es INCORRECTA? *

Cuando evoluciona a la curación nunca lo hace con secuelas cicatrizales.

Entre las condiciones predisponentes se encuentran el embarazo, el reflujo


vesicoureteral y la instrumentación del tracto urinario.

Histológicamente se caracteriza por inflamación supurativa destructiva en parches en


el intersticio, más necrosis tubular.

Puede complicarse con necrosis papilar, principalmente en diabéticos y pacientes


con obstrucción del tracto urinario.

/
Un hombre de 25 años ha padecido cefaleas durante los últimos 5 meses. En ese
tiempo, sus familiares han notado que el afectado no exhibía la misma agudeza
mental que en el pasado y que presentaba mayor labilidad emocional. En un
plazo de 2 semanas sufre cuatro convulsiones generalizadas. En el examen físico
no se observan edema de papila ni alteraciones motoras. La TAC craneal revela
una masa de 2 cm en el lóbulo frontal derecho. Se le realiza una biopsia
estereotáxica de esta lesión observándose sólo gliosis e indicios de hemorragias
recientes y antiguas. La masa es extirpada e histológicamente se observa un
conglomerado de vasos tortuosos de distinto tamaño rodeados de gliosis. ¿Cuál
es el diagnóstico más probable? *

malformación arteriovenosa

angiosarcoma

placa de esclerosis múltiple

aneurisma sacular roto

/
Un paciente de 2 años presentó desviación ocular (estrabismo), leucocoria y una
formación tumoral en el examen del fondo de ojo. Se realizó la enucleación,
observándose una masa tumoral ocupante de la cavidad vítrea con adhesión a la
pared ocular, compuesta por células pequeñas hipercromáticas y de escaso
citoplasma, que formaban rosetas, con amplias áreas de necrosis. ¿Cuál es para
Ud. el diagnóstico más probable? *

Fibroplasia retrolental

Persistencia del vítreo primario hiperplásico

Rabdomiosarcoma

Retinoblastoma

¿Cuáles de los siguientes son factores de riesgo para el carcinoma de vejiga? *

Tabaquismo, sexo masculino, radiación, edades entre 50 y 80 años, consumo


importante de ciclofosfamida.

Tabaquismo, sexo masculino, edades entre 50 y 80 años, dieta alta en fibras


vegetales, aumento de renina.

Tabaquismo, sexo femenino, radiación, edades entre 20 y 50 años, aumento de ADH,


consumo importante de ciclofosfamida.
/
Tabaquismo, sexo femenino, edades entre 20 y 50 años, aumento de ADH, exposición
a arilaminas.

En relación al cáncer gástrico, indique la opción CORRECTA: *

Los adenocarcinomas gástricos de tipo intestinal suelen infiltrar la pared produciendo


una linitis plástica.

Los adenocarcinomas suelen presentarse como pequeñas úlceras de bordes


deprimidos.

Los GIST se originan en el epitelio de revestimiento gástrico.

Los tumores GIST presentan positividad para CD117 o C-kit en inmunohistoquímica.

Paciente varón de 63 años, consulta al hematólogo luego de un examen de


laboratorio donde tenía elevada la serie mieloide. Al examen físico presenta
esplenomegalia y refiere sudoración nocturna. Si Ud sospechara en este
paciente una Leucemia mieloide crónica, ¿qué translocación; que tiene
implicancia pronóstica; podría solicitar para confirmar el cuadro? *

translocación (9,22) - cromosoma Philadelphia

translocación (9,14)

translocación (8,22)

translocación (8,14)

Un paciente pediátrico masculino de 8 años es intervenido quirúrgicamente por


presentar una lesión tumoral en el vermis cerebeloso. La anatomía patológica
revela que está constituida por células atípicas pequeñas de escaso citoplasma y
núcleos hipercromáticos, con mitosis abundantes, gránulos de neurosecreción y
marcación + con GFAP. ¿Cuál es su sospecha diagnóstica? *

Astrocitoma pilocítico.

Meningioma.
/
Ependimoma.

Meduloblastoma.

Un hombre de 65 años se notó un bulto en el cuello hace 2 meses. La exploración


revela un grupo de tres ganglios cervicales posteriores derechos definidos e
indoloros, y una masa de ganglios linfáticos axilares derechos aumentados de
tamaño. La TAC toracoabdominal muestra adenopatías mediastínicas y
hepatoesplenomegalia. El examen microscópico de un ganglio linfático cervical
revela abundantes células binucleadas CD15+ y CD30+ de gran tamaño con
nucléolos acidófilos prominentes, dispersas en un infiltrado linfocítico escaso.
¿Cuál de las siguientes alteraciones es más probable que se ponga de manifiesto
en el análisis molecular de esta lesión? *

infección por Helicobacter pylori en todas las células

reordenamientos del gen BCL6 en las células grandes

mutaciones del gen JAK2 en los linfocitos

integración clonal del virus de Epstein-Barr en las células grandes

Opción 5

Una mujer de 65 años presenta desde hace 3 años una masa de 4cm, móvil,
indolora, bien delimitada y de lento crecimiento en la parte lateral derecha de la
cara, anterior a la oreja y superior a la mandíbula. Se procede a la resección
quirúrgica de la lesión y el estudio histopatológico informa, “Células epiteliales
ductales inmersas en un estroma mixoide con islotes de tejido condroide y
hueso”. ¿Cuál es para usted el diagnóstico más probable? *

Tumor de Warthin

Carcinoma mucoepidermoide

Adenoma pleomorfo

Carcinoma epidermoide

/
Una paciente de 44 años, presenta adenopatías indoloras, leve hepato y
esplenomegalia. Se le extirpa una adenopatía cervical derecha, que mide 3cm de
diámetro. En la superficie de corte se observan nódulos blanquecinos.
Microscópicamente se observaron “conglomerados nodulares de linfocitos
atípicos pequeños (centrocitos) y otros más grandes (centroblastos)”. IHQ: CD10
(+) y la proteína BCL2 (+). ¿Cuál es para Ud. el diagnóstico más probable? *

Linfoma folicular

Linfoma de Hodgkin

Linfoma difuso de células grandes B

Linfoma de células T del adulto

De los tumores de intestino delgado que se enumeran a continuación. ¿Cuál se


halla asociado a la enteropatía por gluten? *

Leiomioma

Schwanoma

Tumor carcinoide

Linfoma

En referencia a la entidad nosológica “Tumores Seminomatosos”, indique la


afirmaciòn INCORRECTA: *

El seminoma clásico es el tumor más frecuente de testículo y predomina en adultos


de la 2da a 3ra década de la vida.

A pesar de ser quimiosensible tiene mal pronóstico por su diseminación precoz

En los tumores mixtos puede combinarse con el coriocarcinoma. /


15% de los seminomas son GCH + por la presencia de células trofoblàsticas
intratumorales.

Consulta un paciente de 60 años por dolor suprapúbico de larga data


relacionado a polaquiuria y tenesmo vesical. Refiere tener Hiperplasia prostática
benigna diagnosticada hace 5 años pero nunca se adecuó al tratamiento. No
refiere antecedentes de relevancia, no toma medicamentos y niega consumir
drogas, entre ellas, tabaco. Si Ud. le efectuara una cistoscopia, ¿qué esperaría
encontrar? *

Atrofia de la mucosa vesical con ulceraciones sangrantes.

Tumoración exofítica de coloración heterogénea, friable, fuertemente adherida a la


pared que ocluye la uretra casi en su totalidad.

Aumento de tamaño de la vesícula con trabeculaciones que forman criptas entre una
y una.

Lesión excéntrica que ocupa más del 50% de la vejiga, de superficie irregular y friable
que se une a la pared vesical por un tallo fibrovascular

En referencia a la “Patología tumoral peneana”, señale el enunciado CORRECTO:


*

Tanto la enfermedad de Bowen y la papulosis bowenoide como representantes del


carcinoma in situ peneano comparten su importante asociación con la infección por
el HPV (16).

El tabaquismo y la acumulación de esmegma por mal hábito higiénico, no se hallan


relacionados con el riesgo de desarrollar carcinoma peneano.

<de 1,5cm de invasión en profundidad no tiene riesgo de metástasis.

El carcinoma verrugoso es una variante endofítica indiferenciada del carcinoma


epidermoide.

¿Cuál de las siguientes características corresponde al Adenocarcinoma de


páncreas? *

Es más frecuente en edades tempranas /


La ictericia obstructiva está más asociada con la ubicación en la cola del páncreas.

Suele diagnosticarse tempranamente y tener un buen pronóstico.

Es más frecuente que las neoplasias del páncreas endócrino.

Un paciente de 55 años, consultó por pirosis. En la endoscopía se visualizó la


mucosa del tercio distal del esófago de color rojo intenso. La biopsia evidenció
epitelio pavimentoso con elongación de las papilas e hiperplasia de células
basales con infiltración inflamatoria a predominio de eosinófilos. ¿Cuál es para
Ud. el diagnóstico más probable? *

Esófago de Barrett

Esofagitis por reflujo

Esofagitis actínica

Esofagitis candidiásica

¿Cuál de los siguientes es un linfoma no Hodgkin indolente (de bajo grado)? *

Linfoma difuso de células B grandes.

Linfoma de Burkitt.

Linfoma folicular.

Mieloma múltiple.

¿Cuál es la leucemia más común en el adulto anciano? *

Leucemia mieloide crónica.

Leucemia linfática aguda.

Leucemia mieloide aguda.

Leucemia linfocítica crónica.

/
La mayoría de los casos de glomerulonefritis (nefropatía) membranosa tienen la
siguiente etiología: *

Infecciones virales

Idiopática o primaria

LES

Antecedentes de fármacos

Usted es residente de urologìa de 1er año y le muestra a su Jefe de Residentes el


siguiente informe “fragmentos irregulares de tejido prostàtico con proliferación
glandular, algunas de las cuales presentan dilataciones quísticas, con dos capas
celulares, con proliferación fibrosa o muscular del estroma”. ¿Cuál de los
siguientes cree Ud. que será el diagnóstico que se corresponda a ese informe? *

Adenocarcinoma de próstata

Prostatitis aguda

Hiperplasia prostàtica benigna

Prostatitis granulomatosa

Un hombre de 55 años observa un aumento de tamaño gradual del escroto a lo


largo del último año. Este crecimiento no es doloroso, aunque se asocia a
sensación de pesadez. No refiere dificultades sexuales. El examen físico no
revela lesiones en la piel escrotal ni masas evidentes, pero el escroto aparece
aumentado de tamaño, edematoso y blando en ambos lados. El resultado de la
transiluminación es positivo, permitiendo el paso de la luz a su través. ¿Cuál es el
diagnóstico más probable? *

varicocele
/
seminoma

orquitis

hidrocele

En cuanto a la criptorquidia, ¿cuál de las siguientes opciones es INCORRECTA? *

Implica riesgo aumentado de desarrollar neoplasia sólo en el testículo no


descendido.

En el examen microscópico del testículo afectado se observa importante hialinización


y engrosamiento de la membrana basal.

Cuando es bilateral puede causar esterilidad.

En el examen microscópico del testículo afectado pueden hallarse focos de


Neoplasia de Células Germinales Intratubular (IGCNU)

Una paciente de 54 años consulta por cefalea. Se le realiza una TAC de cerebro
observándose una lesión nodular, parasagital cerca de la hoz del cerebro y en
contacto con las meninges. Qué hallazgos histopatológicos Ud. esperaria
encontrar: *

Masas sólidas compactas de células ahusadas, con presencia de cuerpos de


psammoma.

Células atípicas que se disponen formando estructuras glandulares, algunas con


vacuolas citoplasmáticas.

Células escamosas atípicas con ocasionales perlas corneas.

Células pequeñas indiferenciadas, hipercromasia, moldeamiento y abundantes


mitosis.

Con respecto a la Patología de SNC señale la opción CORRECTA: *

Los tumores de células germinales cerebrales primarios se presentan con frecuencia


en lóbulos parietales y temporales del cerebro.

Los ependimomas mixopapilares suelen localizarse en el filum terminale de la


médula.

El meningioma fibroblástico presenta numerosos cuerpos de psamoma. /


Los tumores neuronales son más frecuentes que los tumores gliales.

En relación a la enfermedad de Parkinson marcar la INCORRECTA: *

Un hallazgo característico es la palidez de la sustancia negra.

Alrededor del 10-15% de los pacientes con enfermedad de Parkinson desarrollan una
demencia con cuerpos de Lewy.

Es una enfermedad neurodegenerativa caracterizada por un trastorno hipocinético del


movimiento, causado por pérdida de las neuronas dopaminérgicas en la sustancia
negra.

Es una enfermedad neurodegenerativa asociada con la agregación patológica de


proteínas tau.

La microfotografía presentada corresponde a un adenocarcinoma de próstata de


tipo acinar. Según el Sistema de Gleason, ¿qué patrón histológico de
diferenciación glandular le asignaría Usted a las células tumorales que se
observan en la microfotografía? *

No se puede asignar ningún patrón histológico según el Sistema de Gleason.

Patrón histológico 4 según el Sistema de Gleason.


/
Patrón histológico 5 según el Sistema de Gleason.

Patrón histológico 3 según el Sistema de Gleason.

En referencia a la Enfermedad ulcerosa péptica, ¿Cuál de las siguientes


afirmaciones es CORRECTA? *

Morfológicamente se caracteriza por lesión excavada en la cual se observa


nodularidad en los bordes y pliegues asimétricos.

El hipoparatiroidismo se relaciona con la hiperproducción de gastrina, aumentando la


secreción ácida, la cual favorece la enfermedad ulcerosa péptica.

Morfológicamente se caracteriza por lesión excavada redondeada de bordes lisos y


pliegues que convergen radialmente hacia la lesión.

Es infrecuente su asociación con la gastritis crónica inducida por H.Pylori.

Una persona de sexo femenino de 45 años presenta valores aumentados de


fosfatasa alcalina, transaminasas y colesterol, y presencia de anticuerpos
antimitocondriales en plasma. En la biopsia se evidencian los “espacios portas
expandidos por un infiltrado linfocitario y de células plasmáticas, rodeando a una
reacción granulomatosa destructiva centrada en un conducto biliar”. ¿A qué
patología podría corresponder? *

Colangitis esclerosante primaria.

Colangiocarcinoma.

Cirrosis biliar primaria. /


Hemocromatosis.

Paciente femenina de 23 años, concurre a la consulta luego de presentar cuadro


de dolor abdominal de 3 meses de evolución que se acompañó de diarrea y
distensión abdominal. Refiere asociar los episodios de dolor y diarrea a los
alimentos que consume en especial fideos o pan. Ud., sospechando celiaquía, le
solicita una VEDA (videoendoscopia digestiva alta) para toma de biopsia. ¿Qué
espera encontrar en ésta? *

Pared intestinal engrosada y correosa como consecuencia del edema transmural, con
abundantes neutrófilos que infiltran y dañan las criptas del epitelio.

Pérdida de la superficie mucosa y del borde en cepillo con aumento del número de los
linfocitos T CD8+ intraepiteliales e hiperplasia de las criptas.

Infiltración leucocitaria intraepitelial tanto linfocítica como polimorfonucleares con


atrofia de enterocitos.

Aumento de tamaño de las placas de Peyer en íleon terminal, formando elevaciones


bien delimitadas de hasta 8cm de diámetro, con infiltrado polimorfonuclear dentro de
la lámina propia superficial con macrófagos que contienen bacterias.

Un niño de 6 años se queja de dolor progresivo en la ingle derecha desde hace 1


semana. El examen físico revela ganglios linfáticos tumefactos y dolorosos en la
región inguinal derecha. Se realiza la biopsia de un ganglio linfático inguinal, que
macroscópicamente se halla aumentado de tamaño, de coloración gris rojiza y
aspecto congestivo. En el examen histológico exhibe centros germinales
agrandados, de tamaño variable, conteniendo numerosas mitosis. Se encuentra
moderado a intenso infiltrado de neutrófilos en ubicación parafolicular y
sinusoidal. ¿Cuál es la causa más probable de estas alteraciones histológicas?. *

sarcoidosis

linfoma folicular

leucemia linfoblástica aguda

linfadenitis aguda inespecífica

/
Un hombre de 62 años en estudio por molestias abdominales presenta en la TAC
un tumor en polo superior de riñón derecho; se realiza nefrectomía, durante la
cual se ve que la lesión invade la vena renal. Microscópicamente la masa se halla
formada por “células con abundante citoplasma claro y membranas celulares
destacadas”. El genoma del tumor muestra pérdida homocigótica del gen de Von
Hippel Lindau. ¿Cuál es para Ud. el diagnóstico más probable? *

Carcinoma de células renales variante papilar

Carcinoma de células claras de riñón

Nefroblastoma

Angiomiolipoma

Un niño de 6 años, comienza con hematuria, HTA y edemas. En la orina se


observa una proteinuria leve. Se presentan los hallazgos de la biopsia renal y la
inmunofluorescencia. ¿Cuál es el diagnóstico más probable? *

nefropatía por cambios mínimos

enfermedad de Berger

síndrome de membrana basal delgada

glomerulopatía membranosa

/
En referencia a los “Tumores del Sistema Nervioso Central”, cuál de las siguientes
afirmaciones es CORRECTA: *

Algunos tumores gliales con hallazgos histológicos de bajo grado infiltran regiones
extensas en consecuencia son de mal pronóstico.

En los niños la ubicación más frecuente son los hemisferios cerebrales.

En los adultos la ubicación más frecuente es en fosa posterior.

El astrocitoma pilocítico es considerado un tumor infiltrante, grado II/IV, por lo tanto


su ki67 es elevado.

¿Por qué vía suele hacer metástasis el carcinoma renal de células claras con
mayor frecuencia? *

Vía linfática a ganglios lumbo-aórticos.

Vía linfática a ganglios periumbilicales.

Vía hematógena a los pulmones.

Vía hematógena al hígado.

Un varón de 70 años presenta dolor en la columna lumbar de varios meses de


evolución. La Rx muestra dos áreas radio-opacas a nivel de la columna y en el
laboratorio las fosfatasas séricas ácida y alcalina se presentan elevadas. La
presunción diagnóstica más probable es que el cuadro corresponda a: *

Metástasis de carcinoma de vejiga

Mieloma múltiple

Metástasis ósea osteoclástica de carcinoma de próstata

Metástasis ósea osteoblástica de carcinoma de próstata


/
2° RECUPERATORIO 2° EXAMEN PARCIAL
- 2°cuatrimestre 2020
*Obligatorio

Casillas de verificación

- Marcar todas las opciones CORRECTAS.


- PRESTAR MUCHA ATENCIÓN ya que deben marcar TODAS LAS CORRECTAS - siempre CORRECTAS-, si
marcan correctas de menos o incorrectas, el sistema anula la pregunta.

En referencia a los “Sindromes Mielodisplásicos” (SMD), señale cuales de los


siguientes enunciados es CORRECTO. Recuerde que deberá marcar todas las
opciones correctas, considerando que el marcar correctas de menos o 1
respuesta incorrecta, dará por anulado el ejercicio. *

El recuento de blastos y el grado de citopenia periférica son los factores pronósticos


más importantes.

El SMD primario tiene una edad media de inicio a los 70 años y casi la mitad de ellos
se diagnostica de manera incidental.

La pancitopenia y la susceptibilidad a las infecciones es una característica observada


frecuentemente en pacientes con SMD.

La progresión a LMA y el pronóstico ominoso son más frecuentes en los SMD


idiopáticos que en los SMD 2rios.

En los SMD 1° la progresión a LMA se da en más del 90%.

En los SMD los blastos mieloides constituyen menos del 20% de la celularidad
medular global.

/
En referencia a la “Enfermedad Inflamatoria Intestinal”, señale cual/es de los
siguientes enunciados es CORRECTO. Recuerde que deberá marcar todas las
opciones correctas, considerando que el marcar correctas de menos o 1
respuesta incorrecta, dará por anulado el ejercicio. *

La enfermedad de Crohn puede tener afectación transmural con serositis.

La enfermedad de Crohn puede generar un sindrome de malabsorción de grasas y


vitaminas.

La enfermedad de Crohn muestra un compromiso difuso y continuo desde el colon


hasta el íleon.

El infiltrado inflamatorio en la enfermedad de Crohn puede presentar granulomas.

La colitis ulcerosa sólo compromete mucosa y submucosa.

La colitis ulcerosa se asocia a la formación de fístulas y adherencias.

La colitis ulcerosa se asocia a un riesgo incrementado de cáncer de colon.

En referencia a los "Tumores del SNC”, señale cuales de los siguientes


enunciados es CORRECTO. Recuerde que deberá marcar todas las opciones
correctas, considerando que el marcar opciones correctas de menos o marcar 1
respuesta incorrecta, dará por anulado el ejercicio. *

Los gliomas (astrocitomas, oligodendrogliomas y ependimomas) conforman el grupo


más frecuente de tumores cerebrales primarios.

En el oligodendroglioma la presencia de mutaciones IDH1, IDH2 se asocia a mejor


pronóstico.

El glioblastoma multiforme se caracteriza por presentar áreas de necrosis y


proliferación vascular.

Se denominan ependimomas mixopapilares a los tumores ubicados en el filum


terminale de la médula y que contienen elementos papilares sobre un fondo mixoide.

El glioblastoma se caracteriza por presentar células bipolares con prolongaciones


largas y finas que son GFAP (+) acompañadas de fibras de Rosenthal y cuerpos
granulares eosinófilos.
/
Los oligodendrogliomas anaplásicos con codeleción 1p/19q tienen menor respuesta
a la quimio y radioterapia y consecuentemente peor pronóstico.

En referencia a la “Patología vesical”, señale cuales de los siguientes enunciados


es CORRECTO. Recuerde que deberá marcar todas las opciones correctas,
considerando que el marcar opciones correctas de menos o marcar 1 respuesta
incorrecta, dará por anulado el ejercicio. *

El 95% de los tumores vesicales poseen origen mesenquimatoso.

Las cistitis agudas se caracterizan macro y microscópicamente por presentar una


mucosa hiperémica e infiltrado de neutrófilos con o sin exudado.

Existen 2 lesiones precursoras del carcinoma urotelial invasivo: la del carcinoma


urotelial papilar (que puede ser de bajo o alto grado) y la del carcinoma in situ
urotelial plano (sistemáticamente de alto grado)

Las lesiones precursoras más frecuentes del carcinoma urotelial invasivo son los
tumores papilares no invasivos que se originan a partir de la hiperplasia papilar
urotelial.

La anomalía congénita más frecuente y grave es el reflujo vesicoureteral.

La cistitis aguda es una entidad no relacionada con la pielonefritis bacteriana.

En referencia a la “Patología Neoplásica Testicular”, señale cuales de los


siguientes enunciados es CORRECTO. Recuerde que deberá marcar todas las
opciones correctas, considerando que el marcar opciones correctas de menos o
marcar 1 respuesta incorrecta, dará por anulado el ejercicio. *

La mayoría de los tumores de células germinales del testículo son agresivos, capaces
de una diseminación rápida y extensa.

El linfoma difuso de linfocitos B grandes (LDCGB) debe ser considerarse un


diagnóstico diferencial ante las presencia de una masa testicular en pacientes
mayores de 60 años.

Todos los seminomas presentan HCG elevada.

A diferencia de otros tumores malignos de la economía, la mayoría de los tumores


malignos testiculares de células germinales no se originan de una lesión precursora.

El tumor testicular más frecuente en menores de 3 años de edad es el tumor del seno
endodérmico.
/
Las células clásicas de los seminomas son grandes con un gran núcleo central, con
uno a dos nucleolos prominentes.

Los coriocarcinomas provocan con frecuencia aumento del tamaño testicular.

Atrás Enviar

/
11/27/2020 SEGUNDO EXAMEN PARCIAL - COMISIONES UDH

SEGUNDO EXAMEN PARCIAL -


COMISIONES UDH
*Obligatorio

Preguntas Múltiple choice

- Seleccione en cada una de las preguntas la opción CORRECTA o INCORRECTA según lo solicitado.
- RECUERDE marcar sólo 1 opción.

En referencia al “Adenocarcinoma de Próstata”, señale cual de los siguientes


enunciados es INCORRECTO. *

El hallazgo de PIN de alto grado puede estar asociado con adenocarcinoma (hasta en
un 80% de los casos).

En la gran mayoría de los casos surge en la zona central a nivel periuretral.

El cáncer de próstata presenta pérdida de E-cadherina al igual que el carcinoma


invasivo de células en anillo de sello gástrico y el carcinoma lobulillar de mama.

La gradación según el sistema de Gleason es importante en el cáncer de próstata, ya


que el grado y el estadio son los mejores factores predictivos del pronóstico.

https://docs.google.com/forms/d/e/1FAIpQLSfn8OCA4-e09mNqNHTan7VNxgUctUy-vhP0FakJ1vxZpQZjJA/formResponse 1/20
11/27/2020 SEGUNDO EXAMEN PARCIAL - COMISIONES UDH

Un hombre de 57 años presenta una masa indolora, móvil, firme, de 4.5cm en la


región parotídea derecha, que ha aumentado de tamaño en el último año. El
aspecto de la mucosa bucal es normal. No refiere dificultad en la fonación ni en la
masticación. Se procede a la resección quirúrgica de la lesión.
Macroscópicamente y al corte, se observan espacios quísticos con secreciones
mucinosas. Histológicamente estos espacios quísticos separan lobulillos de
epitelio neoplásico, que consta de una capa doble de células epiteliales
eosinófilas situadas sobre un estroma linfoide reactivo. ¿Cuál es el diagnóstico
más probable? *

Adenoma pleomorfo

Carcinoma mucoepidermoide

Linfoma no Hodgkin

Tumor de Warthin

En referencia a la entidad “Seminoma”, ¿Cuáles serían para Ud. los elementos


que permiten justificar el diagnóstico? *

Macroscópicamente: estructura nodular pequeña, con bordes mal definidos y


extensas áreas de hemorragia y necrosis. Microscópicamente: células epiteliales
formando láminas irregulares, con grandes núcleos hipercromáticos. corioCA

Macroscópicamente: estructura nodular de aspecto sólido, blanco-grisácea,


homogénea, delimitada, con tractos blanquecinos. Microscópicamente: células
uniformemente grandes de núcleo redondo, nucléolo prominente y citoplasma claro,
septos con infiltrado de linfocitos.

Macroscópicamente: estructura nodular de gran tamaño, de aspecto heterogéneo con


áreas sólidas y otras quísticas. Microscópicamente: células neurales, musculares,
cartilaginosas y de diferentes tejidos maduros inmersas en un estroma fibroso. teratoma

Macroscópicamente: estructura nodular pequeña de aspecto mucinoso homogéneo y


https://docs.google.com/forms/d/e/1FAIpQLSfn8OCA4-e09mNqNHTan7VNxgUctUy-vhP0FakJ1vxZpQZjJA/formResponse 2/20
11/27/2020 SEGUNDO EXAMEN PARCIAL - COMISIONES UDH
Macroscópicamente: estructura nodular pequeña de aspecto mucinoso, homogéneo y
amarillento. Microscópicamente: células neoplásicas cúbicas dispuestas en red.
saco vit

Una paciente de 54 años consulta por cefalea. Se le realiza una TAC de cerebro
observándose una lesión nodular, parasagital cerca de la hoz del cerebro y en
contacto con las meninges. Qué hallazgos histopatológicos Ud. esperaría
encontrar: * meningioma

Células escamosas atípicas con ocasionales perlas corneas. MTS ca escamoso

Células pequeñas indiferenciadas, hipercromasia, moldeamiento y abundantes


mitosis. meduloblastoma

Masas sólidas compactas de células ahusadas, con presencia de cuerpos de


psammoma.

Células atípicas que se disponen formando estructuras glandulares, algunas con


vacuolas citoplasmáticas. MTS adenocarcinoma

Llega a la consulta una persona con su hijo de 3 años. Se lo nota preocupado al


padre porque refiere que su hijo hace un mes que no quiere jugar, que lo nota
cansado, lo ve algo más pálido. En la anamnesis se resalta que está con
sangrados leves-moderados, y en el último año tuvo 5 veces otitis, de causa
infecciosa. Se sospecha de una patología hemolinfoide, ¿de cuál probablemente
se trate? *

Leucemia/linfoma linfoblástico agudo

Leucemia mieloide aguda

Leucemia mieloide crónica

Síndrome mielodisplásicos

https://docs.google.com/forms/d/e/1FAIpQLSfn8OCA4-e09mNqNHTan7VNxgUctUy-vhP0FakJ1vxZpQZjJA/formResponse 3/20
11/27/2020 SEGUNDO EXAMEN PARCIAL - COMISIONES UDH

En los últimos 6 meses, una mujer de 65 años ha notado una masa de


crecimiento lento en la uretra, que no causa gran molestia. Al examen físico se ve
una masa rojiza de 1,5cm que protruye por el meato uretral externo. ¿Cuál será
para Ud. el diagnóstico más probable cuando se biopsie la lesión? *

Rabdomiosarcoma embrionario

Hemangioma

Carúncula uretral

Carcinoma urotelial

En referencia al “Carcinoma de vesícula biliar” señale cual de las siguientes


opciones es INCORRECTA: *

La colecistitis aguda litiásica es la indicación más frecuente para las


colecistectomías de urgencia.

El patrón de crecimiento más frecuentemente observado es el endofítico infiltrante


con una zona difusamente indurada.

El factor de riesgo más importante para el carcinoma vesicular es el hábito enólico.

La gran mayoría de los cánceres vesiculares se detectan tardíamente por sus


síntomas inespecíficos por lo cual su pronóstico es malo.

Un paciente de 45 años está siendo estudiado por litiasis urinaria. En la TAC


abdominal se observan múltiples dilataciones quísticas de los conductos
colectores en la médula renal. ¿Cuál es para Ud. el diagnóstico más probable? *

Enfermedad poliquística del adulto


https://docs.google.com/forms/d/e/1FAIpQLSfn8OCA4-e09mNqNHTan7VNxgUctUy-vhP0FakJ1vxZpQZjJA/formResponse 4/20
11/27/2020 SEGUNDO EXAMEN PARCIAL - COMISIONES UDH
p q

Riñón en esponja medular

Quistes simples

Nefronoptisis esporádica

Un hombre de 48 años procedente de Hong Kong lleva 4 meses con dificultad


para respirar a través de la nariz y refiere dolor facial. El examen físico reveló una
masa que ocupa la cavidad nasal derecha. La TAC craneal muestra una masa
nasofaríngea de 5cm que erosiona el hueso adyacente. Se procede a la
extirpación y el estudio histopatológico informa que se observan “células
epiteliales grandes con núcleos vesiculosos, nucleolos destacados y bordes
celulares imprecisos dispuestos tipo sincicio. Se visualizan linfocitos maduros
dispuestos en el seno de la neoplasia”. ¿Cuál de los siguientes factores
etiológicos es más probable que haya tenido una mayor implicación en la
aparición de esta lesión? *

Infección por virus Epstein-Barr

Rinitis alérgica

Vasculitis asociada a ANCA

Síndrome de Sjögren

Con respecto a la patología del SNC, señale la opción INCORRECTA: *

El glioblastoma multiforme presenta histología similar a los astrocitomas


anaplásicos y se les agrega necrosis y proliferación vascular o de células
endoteliales.

El glioblastoma multiforme se presentan como masas infiltrantes, hemorrágicas y


necróticas.

El astrocitoma pilocítico suele presentarse en ancianos.

En el glioblastoma multiforme las células tumorales se agrupan alrededor de áreas


necróticas dando imágenes en "empalizada".

https://docs.google.com/forms/d/e/1FAIpQLSfn8OCA4-e09mNqNHTan7VNxgUctUy-vhP0FakJ1vxZpQZjJA/formResponse 5/20
11/27/2020 SEGUNDO EXAMEN PARCIAL - COMISIONES UDH

En referencia a los “Tumores Testiculares”, cuál de las siguientes afirmaciones es


INCORRECTA: *

Los tumores derivados del estroma y de los cordones sexuales (células no


germinales) son los más frecuentes y la gran mayoría de ellos tienen comportamiento
biológico agresivo.

El 60% de los tumores testiculares está compuesto por màs de un patròn “puro”.

Entre los factores de riesgo debe considerarse el Síndrome de disgenesia testicular.

El linfoma testicular es el tumor testicular más frecuente en ancianos.

https://docs.google.com/forms/d/e/1FAIpQLSfn8OCA4-e09mNqNHTan7VNxgUctUy-vhP0FakJ1vxZpQZjJA/formResponse 6/20
11/27/2020 SEGUNDO EXAMEN PARCIAL - COMISIONES UDH

Una paciente de 28 años consultó por presentar anorexia, pérdida de peso,


adinamia e ictericia de varias semanas de evolución. Se le realizó una TAC en la
que se observó una masa hepática única, de bordes irregulares de
aproximadamente 7cm de diámetro ubicada en el lóbulo hepático derecho con
una marcada cicatriz central. Se realiza resección quirúrgica. Histológicamente
se observaron “células poligonales atípicas bien diferenciadas, que crecen
formando nidos o cordones separados por haces densos de colágeno”. ¿Cuál es
para Ud. el diagnóstico más probable? *

Colangiocarcinoma

Hepatocarcinoma fibrolamelar

https://docs.google.com/forms/d/e/1FAIpQLSfn8OCA4-e09mNqNHTan7VNxgUctUy-vhP0FakJ1vxZpQZjJA/formResponse 7/20
11/27/2020 SEGUNDO EXAMEN PARCIAL - COMISIONES UDH

Adenoma hepático

Hiperplasia nodular focal

Paciente de 40 años, femenina, a la que se le realiza fibroendoscopía digestiva


alta y se observa lesión ulcerada de 4,5cm en antro, de bordes irregulares
anfractuosos y fondo sucio. Con los datos aportados cuál es la opción más
CORRECTA: *

Úlcera por estrés/histológicamente: mucosa adyacente normal, sufusión de sangre


en mucosa y submucosa acompañado de reacción inflamatoria.

Úlcera neoplásica/histológicamente: proliferación de células cilíndricas atípicas


formando glándulas que infiltran el estroma.

Úlcera péptica/histológicamente: proliferación glandular atípica infiltrante.

Úlcera péptica/histológicamente: mucosa con material fibrinoleucocitario en la


superficie, tejido de granulación y fibrosis en la base.

En referencia a los “Síndromes Mieloproliferativos”, señale la opción


INCORRECTA: *

Todos los Sindromes Mieloproliferativos pueden transformarse en Leucemias agudas


y una fase que se caracteriza por la fibrosis medular asociada a anemia,
trombocitopenia y esplenomegalia.

La Policitemia Vera y la Mielofibrosis primaria se asocian a mutaciones puntuales


activadoras de la tirocina cinasa JAK2.

En mayores de 60 años la mielofibrosis primaria es el síndrome mieloproliferativo


más frecuente, superando a la policitemia vera y a la trombosis esencial.

En más del 90% de los casos de Leucemia Mieloide Crónica el gen BCR-ABL se crea
por una translocación (9;22)

https://docs.google.com/forms/d/e/1FAIpQLSfn8OCA4-e09mNqNHTan7VNxgUctUy-vhP0FakJ1vxZpQZjJA/formResponse 8/20
11/27/2020 SEGUNDO EXAMEN PARCIAL - COMISIONES UDH

Un paciente de 23 años de edad, inmunodeprimido y con otitis media aguda


presenta alteraciones de la conducta, convulsiones y fiebre alta de 48hs de
evolución. Se solicita una RMN con contraste en la que se observa una zona
redondeada hipodensa rodeada de un halo hipercaptante en lóbulo frontal. El
paciente entra en shock y fallece. Se muestra la imagen macroscópica de un
corte coronal del cerebro e histológicamente se observa “abundante material
necrótico, densos infiltrados de tipo polimorfonuclear y gliosis reactiva”. ¿Cuál
es para Ud. el diagnóstico más probable? *

Quiste coloide

Tuberculoma

Glioblastoma

Absceso piógeno

https://docs.google.com/forms/d/e/1FAIpQLSfn8OCA4-e09mNqNHTan7VNxgUctUy-vhP0FakJ1vxZpQZjJA/formResponse 9/20
11/27/2020 SEGUNDO EXAMEN PARCIAL - COMISIONES UDH

Paciente de 65 años de edad en tratamiento con corticoides sistémicos, consulta


por presentar disfagia. Se realiza fibroendoscopia digestiva alta en la cual se
observan pseudomembranas blanquecinas adherentes en la mucosa esofágica.
Se toma biopsia en la que se informa “elementos filamentosos no ramificados,
PAS (+) en el epitelio escamoso asociado a infiltrado inflamatorio
polimorfonuclear leve”. ¿Cuál es para Ud. el diagnóstico más probable? *

Esofagitis por Citomegalovirus

Esofagitis moniliásica

Esofagitis por sustancias químicas

Esofagitis por reflujo gastroesofágico

En referencia a la “Patología tumoral peneana”, señale el enunciado CORRECTO:


*

Tanto la enfermedad de Bowen y la papulosis bowenoide como representantes del


carcinoma in situ peneano comparten su importante asociación con la infección por
el HPV (16).

El tabaquismo y la acumulación de esmegma por mal hábito higiénico, no se hallan


relacionados con el riesgo de desarrollar carcinoma peneano.

El carcinoma verrugoso es una variante endofítica indiferenciada del carcinoma


epidermoide.

<de 1,5cm de invasión en profundidad no tiene riesgo de metástasis.

https://docs.google.com/forms/d/e/1FAIpQLSfn8OCA4-e09mNqNHTan7VNxgUctUy-vhP0FakJ1vxZpQZjJA/formResponse 10/20
11/27/2020 SEGUNDO EXAMEN PARCIAL - COMISIONES UDH

En referencia a la entidad “Uropatía obstructiva”. Indique la opción INCORRECTA:


*

La hidronefrosis es un factor predisponente para padecer urolitiasis.

La hidronefrosis es la complicación más frecuente en pacientes con hiperplasia


prostática benigna.

Las lesiones obstructivas no tratadas generalmente provocan atrofia renal


permanente.

La hiperplasia prostática benigna genera hidronefrosis unilateral favoreciendo la


formación de cálculos en el uréter homolateral.

En referencia a la entidad nosológica “Pielonefritis aguda”, indique la afirmación


INCORRECTA: *

Puede complicarse con necrosis papilar, principalmente en diabéticos y pacientes


con obstrucción del tracto urinario.

Entre las condiciones predisponentes se encuentran el embarazo, el reflujo


vesicoureteral y la instrumentación del tracto urinario.

Histológicamente se caracteriza por inflamación supurativa destructiva en parches en


el intersticio, más necrosis tubular.

La principal característica macroscópica son las cicatrices corticomedulares


irregulares.

https://docs.google.com/forms/d/e/1FAIpQLSfn8OCA4-e09mNqNHTan7VNxgUctUy-vhP0FakJ1vxZpQZjJA/formResponse 11/20
11/27/2020 SEGUNDO EXAMEN PARCIAL - COMISIONES UDH

Un paciente de 43 años de edad, con hepatitis crónica por virus C, consultó por
presentar edema generalizado. En los exámenes de laboratorio se constató
proteinuria masiva, hipoproteinemia e hipoalbuminemia. Se le realizó biopsia
renal. Observe la imagen y defina qué elementos morfológicos son los que
permiten justificar el diagnóstico. *

glomérulos con áreas de esclerosis, hialinosis focal y segmentaria.


Inmunofluorescencia positiva focal para IgM y C3.

glomérulos con engrosamiento difuso de las paredes capilares. Inmunofluorescencia


positiva para IgG y C3 granular con patrón difuso intramembranoso y depósitos
subepiteliales en la microscopía electrónica.

glomérulos con proliferación mesangial. Inmunofluorescencia positiva para IgA, IgG,


IgM y C3, en mesangio.

glomérulos aumentados de tamaño, con proliferación difusa de células endoteliales,


mesangiales y epiteliales, con infiltrados polimorfonucleares y algunos monocitos.
Inmunofluorescencia positiva para IgG y C3 granular en mesangio y depósitos
subepiteliales grandes en forma de joroba en la microscopía electrónica.

https://docs.google.com/forms/d/e/1FAIpQLSfn8OCA4-e09mNqNHTan7VNxgUctUy-vhP0FakJ1vxZpQZjJA/formResponse 12/20
11/27/2020 SEGUNDO EXAMEN PARCIAL - COMISIONES UDH

Paciente de 10 años consultó por inestabilidad en la marcha de 1 mes de


evoluciòn. En la TAC con contraste se observó una lesión hiperintensa quística
con un nódulo mural en la fosa posterior de la línea media. Se procede a la
resección quirúrgica. Informe histopatológico: Células bipolares con
prolongaciones "pilosas" largas y finas que son positivas para GFAP y que forman
una densa trama fibrilar. Además pueden observarse fibras de Rosenthal. ¿Cuál
es para Ud. el diagnóstico más probable? *

Oligodendroglioma

Astrocitoma anaplásico

Glioblastoma multiforme

Astrocitoma pilocítico

En referencia a las “Enfermedades Neurodegenerativas”, señale el enunciado


INCORRECTO: *

El hallazgo característico de la Enfermedad de Parkinson es la despigmentación de la


substancia nigra y del locus coeruleus y presencia de cuerpos de Lewy.

La Enfermedad de Alzheimer se caracteriza a nivel patológico por presentar en su


fase final, placas neuríticas y ovillos neurofibrilares.

La causa más frecuente de demencia es la Enfermedad de Parkinson.

El proceso patológico común a la mayor parte de las enfermedades


neurodegenerativas es la acumulación de agregados de proteínas diferenciándose en
la localización anatómica y en otras alteraciones celulares específicas.

https://docs.google.com/forms/d/e/1FAIpQLSfn8OCA4-e09mNqNHTan7VNxgUctUy-vhP0FakJ1vxZpQZjJA/formResponse 13/20
11/27/2020 SEGUNDO EXAMEN PARCIAL - COMISIONES UDH

Con respecto al adenocarcinoma ductal infiltrante de páncreas, señale la


respuesta INCORRECTA: *

Se originan a partir de lesiones precursoras no invasivas y bien diferenciadas que


afectan a los conductos de pequeño calibre y se denominan neoplasia intraepitelial
pancreática (PanIN).

La supervivencia a los 5 años es cercana al 5%.

El oncogen SMAD4 es el que más frecuentemente se afecta, descubriéndose


mutaciones puntuales en el 90-95% de los casos.

Los carcinomas del cuerpo y la cola del páncreas no comprimen la vía biliar por lo
que permanecen silentes por relativos períodos de tiempo, alcanzando un tamaño
importante, hallándose diseminados al momento del diagnóstico.

¿Cuál es para usted el factor pronóstico más importante a tener en cuenta para
la evolución del carcinoma urotelial o transicional de vejiga? *

La invasión de la capa muscular

Crecimiento papilar

Necrosis tumoral

Numerosas mitosis

https://docs.google.com/forms/d/e/1FAIpQLSfn8OCA4-e09mNqNHTan7VNxgUctUy-vhP0FakJ1vxZpQZjJA/formResponse 14/20
11/27/2020 SEGUNDO EXAMEN PARCIAL - COMISIONES UDH

Un niño de 6 años se queja de dolor progresivo en la ingle derecha desde hace 1


semana. El examen físico revela ganglios linfáticos tumefactos y dolorosos en la
región inguinal derecha. Se realiza la biopsia de un ganglio linfático inguinal, que
macroscópicamente se halla aumentado de tamaño, de coloración gris rojiza y
aspecto congestivo. En el examen histológico exhibe centros germinales
agrandados, de tamaño variable, conteniendo numerosas mitosis. Se encuentra
moderado a intenso infiltrado de neutrófilos en ubicación parafolicular y
sinusoidal. ¿Cuál es la causa más probable de estas alteraciones histológicas? *

linfadenitis aguda inespecífica

leucemia linfoblástica aguda

sarcoidosis

linfoma folicular

¿Cuál de los siguientes cuadros responde con mayor fidelidad a la descripción


macroscópica de los riñones en la Arteriosclerosis hialina? *

Ligeramente disminuidos de tamaño y peso. Superficie externa finamente granular.


Corteza adelgazada.

Disminuidos de tamaño y peso. Superficie externa lisa, de color rojo vinoso. Corteza
engrosada.

Marcadamente aumentados de tamaño y peso. Superficie externa lisa, pálida.


Corteza adelgazada.

Aumentados de tamaño y peso. Superficie externa de color rojo vinoso, con cicatrices
profundas en forma de “C”. Corteza engrosada.

https://docs.google.com/forms/d/e/1FAIpQLSfn8OCA4-e09mNqNHTan7VNxgUctUy-vhP0FakJ1vxZpQZjJA/formResponse 15/20
11/27/2020 SEGUNDO EXAMEN PARCIAL - COMISIONES UDH

¿Cuál de los siguientes es un linfoma no Hodgkin indolente (de bajo grado)? *

Linfoma folicular

Linfoma de Burkitt

Mieloma múltiple

Linfoma difuso de células B grandes

En referencia a los infartos cerebrales, cual de las siguientes afirmaciones es


INCORRECTA: *

Los infartos lacunares suelen ser pequeños y múltiples y se relacionan con la HTA.

Los infartos cerebrales se deben siempre a fenómenos oclusivos arteriales.

La extensión del infarto dependerá del grado de obstrucción, calibre del vaso, estado
y grado de anastomosis de los vasos arteriales circundantes.

Los infartos cerebrales por oclusión se relacionan con ateroémbolos provenientes de


la carótida interna.

La fase leucémica del linfoma cutáneo de células T se llama: *

Síndrome de Sezary

Micosis fungoide

Leucemia de células pilosas

Leucemia de células T adultas

https://docs.google.com/forms/d/e/1FAIpQLSfn8OCA4-e09mNqNHTan7VNxgUctUy-vhP0FakJ1vxZpQZjJA/formResponse 16/20
11/27/2020 SEGUNDO EXAMEN PARCIAL - COMISIONES UDH

En relación con la Patología leucocitaria indique la opción CORRECTA: *

La histiocitosis sinusal se caracteriza por un estrechamiento de la luz de los


sinusoides en los ganglios linfáticos.

En una hiperplasia folicular, a nivel de los centros germinales, los macrófagos con
cuerpos tingibles contienen núcleos de linfocitos T que han sufrido apoptosis.

La hiperplasia paracortical desencadena respuestas inmunitarias mediadas por


linfocitos B.

La leucocitosis eosinófila (eosinofilia) se observa en trastornos alérgicos, reacciones


adversas medicamentosas, algunos procesos malignos etc.

Paciente de 26 años que consulta por presentar lesión ulcerada y sobreinfectada


en la piel del pene de 1 año de evoluciòn. Se toma biopsia la cual informa:
“proliferación de células pavimentosas con núcleos irregulares e hipercromáticos
con pérdida de la polaridad nuclear. Se observan puentes intercelulares y focos
de queratinizaciòn en forma de perla còrnea con importante infiltrado
polimorfonuclear.”. ¿Cuál es para Ud. el diagnóstico màs probable? *

Papulosis bowenoide

Condiloma acuminado

Carcinoma epidermoide invasivo

Enfermedad de Bowen

https://docs.google.com/forms/d/e/1FAIpQLSfn8OCA4-e09mNqNHTan7VNxgUctUy-vhP0FakJ1vxZpQZjJA/formResponse 17/20
11/27/2020 SEGUNDO EXAMEN PARCIAL - COMISIONES UDH

Un paciente de 2 años presentó desviación ocular (estrabismo), leucocoria y una


formación tumoral en el examen del fondo de ojo. Se realizó la enucleación,
observándose una masa tumoral ocupante de la cavidad vítrea con adhesión a la
pared ocular, compuesta por células pequeñas hipercromáticas y de escaso
citoplasma, que formaban rosetas, con amplias áreas de necrosis. ¿Cuál es para
Ud. el diagnóstico más probable? *

Rabdomiosarcoma

Fibroplasia retrolental

Retinoblastoma

Persistencia del vítreo primario hiperplásico

Un paciente de 68 años de edad, con trastornos miccionales y diagnóstico de


Hiperplasia fibroadenomatosa de la próstata, es intervenido quirúrgicamente
resecándose varios fragmentos irregulares de tejido blanquecino con sectores
de aspecto criboso que en conjunto pesan 140gr. ¿Cuáles serían para Ud. los
elementos histológicos que permiten justificar el diagnóstico? *

Proliferación de células epiteliales que forman, en sectores, glándulas y en otros


cordones comprimidos por el aumento del número de células del estroma.

Proliferación de glándulas, algunas de las cuales presentan dilataciones quísticas,


con dos capas celulares, con proliferación fibrosa o muscular del estroma.

Proliferación de glándulas de aspecto criboso, con una sola capa de células,


rodeadas de estroma fibromuscular con cambios desmoplásicos.

Proliferación de glándulas pequeñas con una sola capa de células y núcleos grandes
vacuolados con nucléolo prominente con escaso estroma interpuesto.

https://docs.google.com/forms/d/e/1FAIpQLSfn8OCA4-e09mNqNHTan7VNxgUctUy-vhP0FakJ1vxZpQZjJA/formResponse 18/20
11/27/2020 SEGUNDO EXAMEN PARCIAL - COMISIONES UDH

Varón de 65 años comienza desde hace 3 meses con debilidad, fatiga,


linfadenopatía generalizada, hepatoesplenomegalia, cuadro de anemia
moderada con trombocitopenia y marcada leucocitosis (con un 80% de
linfocitos). En la biopsia ganglionar se constata reemplazo difuso de la
arquitectura por una población de células linfoides pequeñas con núcleo
redondeado con cromatina condensada y escaso citoplasma. La biopsia de
médula ósea también muestra agregados intersticiales de linfocitos pequeños. El
estudio inmunohistoquímico de las células neoplásicas revela positividad de
membrana citoplasmática con CD20, CD23 y CD5. El diagnóstico más probable
es: *

Linfoma folicular

Linfoma linfoblástico

Leucemia linfocítica crónica

Leucemia de células vellosas

https://docs.google.com/forms/d/e/1FAIpQLSfn8OCA4-e09mNqNHTan7VNxgUctUy-vhP0FakJ1vxZpQZjJA/formResponse 19/20
11/27/2020 SEGUNDO EXAMEN PARCIAL - COMISIONES UDH

Un varón de 80 años ha presentado dificultad para iniciar la micción y nicturia en


el último año. Además, ha sufrido lumbalgia progresiva en los últimos 6 meses. Al
tacto rectal se encuentra una próstata dura e irregular. La gammagrafía ósea
revela áreas de captación en la vértebras torácicas y lumbares. Las pruebas de
laboratorio revelan: fosfatasa alcalina sérica de 300 U/l y antígeno prostático
específico de 70 ng/ml. La tasa de nitrógeno ureico en sangre es de 44 mg/dl y la
de creatinina sérica de 3,8 mg/dl. Se obtienen muestras transrectales de biopsia
de todos los lóbulos prostáticos. El estudio histológico de las mismas revela que
más del 90% del tejido está constituido por una infiltración de cordones y
láminas de células con núcleos pleomórficos hipercromáticos, con nucléolos
evidentes y escaso citoplasma. ¿Cuál de las siguientes clasificaciones es la mejor
para la enfermedad de este paciente? *

Gleason 5, 5

Gleason 1,1

Gleason 3, 3

Gleason 2, 2

Atrás Siguiente

Nunca envíes contraseñas a través de Formularios de Google.

El formulario se creó en Facultad de Medicina. Denunciar abuso

Formularios

https://docs.google.com/forms/d/e/1FAIpQLSfn8OCA4-e09mNqNHTan7VNxgUctUy-vhP0FakJ1vxZpQZjJA/formResponse 20/20
SEGUNDO EXAMEN PARCIAL -
COMISIONES URIBURU TARDE (1, 2, 5, 6)
*Obligatorio

Preguntas Múltiple choice

- Seleccione en cada una de las preguntas la opción CORRECTA o INCORRECTA según lo solicitado.
- RECUERDE marcar sólo 1 opción.

¿Cuáles son las principales características histológicas que deben observarse en


un tumor astrocitario para considerárselo Glioblastoma? *

Incremento de la densidad celular, con variable pleomorfismo nuclear y presencia de


mitosis.

Degeneración quística.

Hiperplasia endotelial o microvascular, necrosis tumoral y pseudoempalizadas


nucleares.

Positividad con GFAP (Proteína gliofibrilar ácida).

Llega a la guardia un estudiante de Medicina de sexo masculino de 25 años de


edad quien en un autoexamen testicular se ha palpado una masa no dolorosa en
el testículo izquierdo. Se realizan exámenes complementarios con altas
sospechas de un tumor de células germinales testiculares. No impresiona tener
linfadenopatías ni síntomas en otros órganos. El médico de guardia sugiere una
biopsia sin pensarlo tanto, mientras el estudiante no está de acuerdo. ¿Por qué
no estaría indicada la biopsia testicular? *

Debido a que este tipo de tumores tienen tan buen pronóstico que los pacientes no
reciben tratamiento.

Debido a que se espera la diseminación a ganglios inguinales para hacerla.

Debido al riesgo de diseminación del tumor por la punción.

Debido a que todos los tumores de células germinales tienen un pronóstico similar.

/
Una paciente de 44 años, presenta adenopatías indoloras, leve hepato y
esplenomegalia. Se le extirpa una adenopatía cervical derecha, que mide 3cm de
diámetro. En la superficie de corte se observan nódulos blanquecinos.
Microscópicamente se observaron “conglomerados nodulares de linfocitos
atípicos pequeños (centrocitos) y otros más grandes (centroblastos)”. IHQ: CD10
(+) y la proteína BCL2 (+). ¿Cuál es para Ud. el diagnóstico más probable? *

Linfoma folicular

Linfoma de Hodgkin

Linfoma de células T del adulto

Linfoma difuso de células grandes B

Un muchacho de 18 años presenta hipoacusia, zumbidos y comienza con vértigo;


en el estudio de potenciales evocados se sugiere una lesión del nervio acústico;
radiológicamente se demuestra agrandamiento del poro acústico interno; en la
TAC se observa un tumor en el ángulo pontocerebeloso en relación con el VIII
par. Histológicamente se observan “células ahusadas dispuestas con dos
patrones, unos más celulares con cuerpos de Verocay y otros hipocelulares
(áreas de “Antoni A” y “Antoni B”). El estudio inmunofenotípico de las células
neoplásicas demuestra proteína S100 (+).” ¿Cuál es para Ud. el diagnóstico más
probable? *

Neurofibrosarcoma

Schwannoma

Astrocitoma

Meningioma

/
Un paciente masculino de 65 años con antecedentes de adenocarcinoma de
colon es sometido a una segmentectomía hepática por una lesión focal. Se
muestra la pieza quirúrgica. *

Hemangioma.

Metástasis de adenocarcinoma de colon.

Adenoma hepático.

Hiperplasia nodular focal.

Paciente de 43 años de edad, obeso, que se presenta a la consulta por padecer


pirosis post ingesta y en decúbito. Refiere que el síntoma mejora con la ingesta
de sustancias alcalinas. Se realiza una fibroendoscopia digestiva alta en la cual se
observan en el tercio inferior del esófago, múltiples erosiones hiperémicas y
edematosas. Se toma biopsia que informa “leve hiperplasia de la zona basal
epitelial, elongación de las papilas de la lámina propia con infiltrado eosinófilo
intraepitelial disperso”. ¿Cuál es para Ud. el diagnóstico más probable? *

Esófago de Barrett

Esofagitis moniliásica

Esofagitis por Citomegalovirus

Esofagitis por reflujo gastroesofágico

/
Una mujer de 65 años presenta desde hace 3 años una masa de 4cm, móvil,
indolora, bien delimitada y de lento crecimiento en la parte lateral derecha de la
cara, anterior a la oreja y superior a la mandíbula. Se procede a la resección
quirúrgica de la lesión y el estudio histopatológico informa, “Células epiteliales
ductales inmersas en un estroma mixoide con islotes de tejido condroide y
hueso”. ¿Cuál es para usted el diagnóstico más probable? *

Tumor de Warthin

Carcinoma mucoepidermoide

Adenoma pleomorfo

Carcinoma epidermoide

Una mujer de 25 años consulta a su médico por presentar un ganglio linfático


cervical indoloro, incluso a la palpación, que no disminuye de tamaño desde hace
3 semanas. El profesional al revisarla encuentra otras adenopatías de menor
tamaño próximas a la referida y también en el sector supraclavicular del mismo
lado. Sin demora, solicita una biopsia escisional de uno de los ganglios
comprometidos que el patólogo informa como "Linfoma de Hodgkin, variante
esclerosis nodular". Indique cuál es la opción INCORRECTA: *

Con el tratamiento adecuado y oportunamente instaurado, la entidad suele ser de


muy buen pronóstico.

Las células linfoides neoplásicas son positivas para CD45, CD15 y CD30, y negativas
para CD3 y CD20.

No suele asociarse a infección por el virus Epstein-Barr.

Es una de las formas clásicas de la enfermedad, y la más frecuente entre ellas.

En relación a la “Cáncer de vesícula biliar” indique la opción INCORRECTA: *

La sobreexpresión de Her-2/neu en el carcinoma vesicular se constituye en un blanco


terapéutico.

Los fenómenos de calcificación distrófica en la pared vesicular aumentan el riesgo de


carcinoma vesicular.

Al momento del diagnóstico, la mayoría de los carcinomas vesiculares, han infiltrado


el hígado, conductos biliares y ganglios portahepáticos.

El carcinosarcoma de vesícula biliar es el tumor maligno más frecuente de la vía biliar


extrahepática.

/
En referencia a la Enfermedad ulcerosa péptica, ¿Cuál de las siguientes
afirmaciones es CORRECTA? *

Es infrecuente su asociación con la gastritis crónica inducida por H.Pylori.

Morfológicamente se caracteriza por lesión excavada redondeada de bordes lisos y


pliegues que convergen radialmente hacia la lesión.

El hipoparatiroidismo se relaciona con la hiperproducción de gastrina, aumentando la


secreción ácida, la cual favorece la enfermedad ulcerosa péptica.

Morfológicamente se caracteriza por lesión excavada en la cual se observa


nodularidad en los bordes y pliegues asimétricos.

Un paciente de 42 años de edad consulta por dolor lumbar y hematuria de 2 días


de evolución. En los estudios radiológicos se observa una lesión de límites netos,
con focos de calcificación y contenido líquido de 6cm de diámetro ubicada a
nivel de uno del polo renal inferior izquierdo. Se realiza cirugía.
Macroscópicamente se observa un quiste de pared fina con áreas calcificadas y
contenido serohemático. Histológicamente se observa una “pared constituida
por tejido conectivo denso y depósitos cálcicos, tapizada por epitelio plano
simple”. ¿Cuál es para Ud. el diagnóstico más probable? *

Carcinoma de células renales

Quiste simple renal

Hamartoma

Poliquistosis renal

Un hombre de 63 años presenta en el laboratorio un análisis de PSA 8ng/mL. Se le


realizan biopsias cuyo resultado del examen microscópico muestra “estructuras
glandulares dispuestas apiñadas, revestidas por una sola hilera de células
epiteliales cuboides con núcleo ovoide con nucléolo prominente”. ¿Cuál de las
siguientes complicaciones es más probable que desarrolle este paciente? *

Hidronefrosis

Esterilidad

Lesiones vertebrales destructivas

Infección recurrente del tracto urinario

/
Su tío, lo consulta a usted, estudiante de medicina, para que le explique el
resultado del informe anatomopatológico de la punción-biopsia de próstata que
le realizaron en el Servicio de Urología del hospital en el que se atiende. El
informe dice lo siguiente: Adenocarcinoma de Próstata - Gleason combinado 8
(5+3). ¿Cuál de las siguientes opciones es para Ud. la más correcta en cuanto al
pronóstico de la enfermedad de su tío? *

tiene un tumor maligno poco diferenciado y su pronóstico es malo.

tiene un tumor maligno moderadamente diferenciado y su pronóstico es bueno.

tiene un tumor maligno bien diferenciado y su pronóstico es excelente.

tiene un tumor maligno con 8 mitosis (5+3) por campo y su pronóstico es malo.

Un hombre de 27 años ha notado un aumento de tamaño progresivo del escroto


con sensación de pesadez durante el último año. En la exploración física se
observa que el testículo derecho es dos veces mayor de lo normal, firme y
ligeramente doloroso. La ecografía identifica una masa testicular derecha de 3,5
cm sólida. La TAC abdominal revela un aumento de tamaño de los ganglios
linfáticos paraaórticos. En la Rx. tórax se identifican múltiples nódulos
pulmonares. En las pruebas de laboratorio se destaca un incremento de las
concentraciones séricas de gonadotropina coriónica humana y α-fetoproteína.
¿Cuál de las siguientes neoplasias es más probable que tenga este paciente? *

Tumor de células germinales mixto

Coriocarcinoma

Tumor de células de Leydig

Seminoma espermatocítico puro

En referencia a la entidad “Linfadenitis”, señale la opción INCORRECTA: *

La histiocitosis sinusal es particularmente prominente en los ganglios linfáticos en el


curso de las infecciones virales.

La linfadenitis crónica inespecífica puede generar diversos patrones morfológicos


como la hiperplasia folicular, hiperplasia paracortical y la histiocitosis sinusal.

Las adenomegalias inducidas por infecciones piógenas en ocasiones dejan cicatrices


residuales.

La hiperplasia folicular se debe a estímulos que activan la respuesta inmune humoral


y morfológicamente deberá diferenciarse del linfoma folicular.
/
Paciente de 46 años de edad consulta por convulsiones repetidas y cefaleas de
15 días de evolución. Se solicita RMN con contraste: “masa hipercaptante con
focos de microcalcificación ubicada a nivel del lóbulo frontal”. Se realiza cirugía.
Histológicamente se observa una “proliferación de células uniformes de núcleo
redondo e hipercromàtico rodeado de escaso citoplasma con halos
perinucleares acompañadas por una red de capilares anastomosados. Pueden
visualizarse focos microscópicos calcificados”. ¿Cuál es para Ud. el diagnóstico
más probable? *

Astrocitoma anaplàsico (GIII/IV)

Oligodendroglioma clásico (GII/IV)

Xantoastrocitoma pleomòrfico (GII/IV)

Glioblastoma multiforme (GIV/IV)

En referencia a la entidad nosológica “Carcinoma Urotelial papilar de bajo grado”,


indique la afirmación CORRECTA: *

Las células neoplásicas que lo componen presentan alta cohesividad y se hallan


distribuidas homogéneamente manteniendo la polaridad.

Este tipo de tumores se caracterizan por la alta frecuencia de recidivas.

Sin tratamiento el 75% progresa a carcinoma in situ.

Las células neoplásicas presentan marcada atipia con figuras mitóticas frecuentes a
predominio basal.

Con respecto a la Patología de SNC señale la opción CORRECTA: *

Los ependimomas mixopapilares suelen localizarse en el filum terminale de la


médula.

Los tumores neuronales son más frecuentes que los tumores gliales.

El meningioma fibroblástico presenta numerosos cuerpos de psamoma.

Los tumores de células germinales cerebrales primarios se presentan con frecuencia


en lóbulos parietales y temporales del cerebro.

/
El cromosoma Filadelfia se caracteriza por: *

t (8; 14)

t (9:22)

t (14; 8)

t (22; 9)

Una niña de 3 años de edad es llevada por su madre a la consulta médica por
presentar palidez, cansancio, numerosas petequias en el tronco, adenopatías
cervicales y fiebre de varios días de evolución. Un exámen de laboratorio revela
una disminución significativa de eritrocitos, leucocitos y plaquetas a nivel
plasmático por lo que se decide realizar una PAMO. Este estudio describe una
médula ósea hipercelular constituída por una abundante proliferación de
linfoblastos con cromatina nuclear condensada, pequeños nucleólos y
citoplasma agranular escaso que desplazan a los elementos celulares normales.
Además se observa que la membrana nuclear de estas células presenta
hendiduras profundas que le confieren un aspecto convoluto. ¿Cual es su
sospecha diagnóstica? *

Linfoma de Hodgkin.

Leucemia mieloide aguda.

Leucemia mieloide crónica.

Leucemia/linfoma linfoblástico agudo.

Respecto a la pancreatitis hemorrágica aguda, señale la opción INCORRECTA. *

Consiste en la destrucción brusca y difusa del tejido pancreático por acción de


enzimas líticas.

En su fisiopatología intervienen procesos de proteolisis y lipolisis.

Se relaciona frecuentemente con consumo de alcohol y patología de la vía biliar,


como por ejemplo colelitiasis.

La hemorragia observada en estos procesos es consecuencia exclusiva del accionar


de tripsinas y lipasas.

/
Paciente de 78 años de edad ha sufrido un accidente cerebro vascular y la TAC
realizada en su internación informa "probable infarto". ¿Con qué entidad
patológica se asocian con mayor frecuencia los infartos encefálicos? *

Ateroesclerosis

Plaquetopenia

Deficiencia de los factores de coagulación

Tumores metastásicos cerebrales

Fallece una mujer de 60 años a raíz de un problema durante la anestesia para una
cirugía estética. Se realizó la autopsia, en la que se observaron “ambos riñones
ligeramente disminuidos de tamaño y peso, con la superficie externa ligeramente
irregular. Histológicamente se observaron algunos glomérulos en oblea,
moderada fibrosis intersticial, atrofia tubular, arteriolas pequeñas con paredes
engrosadas, hialinizadas, con reducción de la luz y ateriolas interlobares y
arcuatas con hipertrofia de la media, duplicación de la lámina elástica y fibrosis
intimal”. ¿Cuál es para Ud. el diagnóstico más probable? *

Nefroesclerosis benigna

Necrosis cortical difusa

Pielonefritis crónica

Nefroesclerosis maligna

/
La microfotografía presentada corresponde a tejido nervioso de un paciente con
una meningoencefalitis vírica. Las flechas indican una célula de Purkinje con
inclusiones citoplasmáticas redondeadas a ovales y eosinófilas. ¿La presencia de
estas inclusiones se considera patognomónica de cuál de las siguientes
entidades? *

Virus del herpes simple tipo 1.

Rabia.

Citomegalovirus.

Arbovirus.

/
Un paciente de 4 años de edad es traído a la consulta por sus padres
presentando edema de ambos párpados y tobillos. En los exámenes de
laboratorio el paciente presenta proteinuria masiva, hipoproteinemia e
hipoalbuminemia. Se le realiza una punción biopsia renal. Observe la imagen y
defina qué elementos morfológicos son los que permiten justificar el
diagnóstico. *

glomérulos aumentados de tamaño, con proliferación difusa de células endoteliales,


mesangiales y epiteliales, con infiltrados polimorfonucleares y algunos monocitos.
Inmunofluorescencia positiva para IgG y C3 granular en mesangio y depósitos
subepiteliales grandes en forma de joroba en la microscopía electrónica.

glomérulos de estructura conservada sin alteraciones túbulo intersticiales. La


inmunofluorescencia es negativa para los distintos tipos de inmunoglobulinas y para
el complemento. Borramiento uniforme y difuso de los podocitos en la microscopía
electrónica

glomérulos con áreas de esclerosis, hialinosis focal y segmentaria.


Inmunofluorescencia positiva focal para IgM y C3 con pérdida de las prolongaciones
en la microscopía electrónica.

glomérulos con proliferación mesangial. Inmunofluorescencia positiva para IgA, IgG,


IgM y C3, en mesangio.

¿Cuál de los siguientes marcadores inmunohistoquímicos que se utilizan


habitualmente para el estudio de patologías hemolinfoides NO es considerado
como marcador "T"? *

CD5.

CD8.

CD3.

CD21.

/
Una mujer de 57 años refiere dolor miccional durante los últimos 5 meses y ayer
observó que había manchado su ropa interior con sangre. Al examen físico se
evidencia un nódulo rojizo doloroso de 1cm en el labio posterior de la uretra
externa. Se procede a la resección quirúrgica. ¿Qué hallazgo patológico es más
probable que se encuentre en el estudio de esta lesión? *

Carcinoma epidermoide.

Células multinucleadas.

Tejido de granulación.

Infiltración de células plasmáticas.

Se realiza biopsia de adenopatía en paciente masculino de 68 años con Linfoma


de Hodgkin cuyo resultado informa: infiltrado denso linfocitario con pérdida de
histoarquitectura, con variantes mononucleares y células de Reed-Stenberg
positivas para PAX5, CD15 y CD30. ¿De qué subtipo de Linfoma de Hodgkin se
trata? *

Rico en linfocitos

Celularidad mixta

Esclerosis nodular

Depleción linfocítica

Un hombre de 70 años que presenta paresia en miembros inferiores, con


alteraciones en su coagulograma y PSA elevado, cuyos estudios radiográficos
señalaron una lesión hiperdensa a nivel del cuerpo de L4 que comprime la cola
de caballo es sometido a una punción bajo guía tomográfica, sospechándose la
posibilidad de una lesión metastásica. El examen microscópico de la muestra
obtenida consistió en células atípicas indiferenciadas dispuestas
predominantemente en grupos compactos, con ausencia luces. Ante la alta
sospecha que el tumor primario pudiere tratarse de un adenocarcinoma de
origen prostático, RESPONDA: *

El diagnóstico de metástasis requerirá la ayuda de alguna marcación


inmunohistoquímica, como PSA, para determinar el origen de las células neoplásicas.

Considera que la muestra es insuficiente para determinar el diagnóstico de


metástasis y evalúa la necesidad de solicitar una nueva muestra de la lesión.

En los adenocarcinomas prostáticos las citoqueratinas de alto peso molecular no


marcan debido a la ausencia de células basales, y también por ello su ausencia en /
una metástasis ayuda a confirmar el diagnóstico y su procedencia.

La imagen observada al microscopio y el PSA sérico elevado son suficientes para


diagnosticar metástasis de carcinoma de origen prostático.

Señale la opción CORRECTA acerca del Tumor del Saco Vitelino o Tumor del Seno
Endodérmico: *

Es muy común como forma pura en el adulto.

Es el tumor de testículo más frecuente después de los 30 años de edad.

Suele aumentar la α-fetoproteína sérica.

Microscópicamente generalmente se halla constituido por playas de células


pequeñas, redondas y azules.

Asiste a la consulta una mujer de 55 años que se descubre un nódulo al frotar la


lengua contra la cara interna de la mejilla. En el examen físico se constata en la
línea de mordida, la presencia de un nódulo firme indoloro rosado, de 0,6cm
recubierto por mucosa bucal. Se realiza la escisión de la lesión y el estudio
histopatológico revela “hiperplasia fibrosa focal”. ¿Cuál es para usted el
diagnóstico más probable? *

Granuloma piógeno

Leucoplasia

Fibroma

Sialodenitis

En referencia a la entidad nosológica “Lesión tubular aguda de origen


isquémico”, indique la afirmación INCORRECTA: *

Se aprecia una necrosis extensa, difusa, en los segmentos del túbulo contorneado
proximal y necrosis del asa de Henle ascendente.

El evento central está dado el aumento del efecto vasoconstrictor de la endotelina y


disminución del óxido nítrico y prostaciclina.

Presenta cambios estructurales propios de la injuria reversible (como pérdida del


ribete en cepillo) e irreversible (como necrosis y apoptosis)

Puede cursar clínicamente con volumen urinario normal o incluso aumentado.

/
En referencia a la entidad “Uropatía obstructiva”. Indique la opción INCORRECTA:
*

El tamaño renal dependerá del grado, nivel y duración de la obstrucción.

La capacidad de concentración tubular y de filtración glomerular, no se ven afectadas


en la uropatía obstructiva.

La uropatía obstructiva y su consecuencia la hidronefrosis, generan atrofia renal


progresiva.

La obstrucción bilateral completa de presentación aguda obliga a instaurar un


tratamiento de forma rápida.

Paciente masculino de 61 años, concurre a la consulta por presentar dolor


suprapúbico, tenesmo vesical y polaquiuria con ardor al orinar, de 2 meses de
evolución. Refiere dolor lumbar que él asocia con sus problemas de ciático y le
entrega un estudio de laboratorio que informa PSA 9ng/mL. La biopsia prostática
evidencia abundantes glándulas con dilataciones quísticas revestidas por dos
capas de epitelio. ¿A qué patología prostática correspondería este cuadro
clínico? *

Neoplasia intraepitelial prostática (PIN) de alto grado

Prostatitis aguda

Hiperplasia prostática benigna

Adenocarcinoma de próstata bien diferenciado

Una mujer de 85 años, que vive en una residencia de ancianos, es hospitalizada


por desarrollo de niveles fluctuantes de consciencia, con cefaleas y confusión en
los últimos dos días. En el examen físico está consciente, pero desorientada e
irritable. Los parámetros vitales son temperatura de 36,9°C y TA de 130/85
mmHg. Presenta edema de papila derecho. La TAC cerebral demuestra
acumulación de sangre en el espacio subdural derecho. ¿Cuál de las siguientes
lesiones vasculares es más probable que genere estos hallazgos? *

trombosis de la arteria cerebral media

laceración de la arteria meníngea media

desgarro de las venas comunicantes cerebrales

rotura de un aneurisma sacular

/
SEGUNDO EXAMEN PARCIAL -
COMISIONES URIBURU TARDE (1, 2, 5, 6)
*Obligatorio

Casillas de verificación

- Marcar todas las opciones CORRECTAS.


- PRESTAR MUCHA ATENCIÓN ya que deben marcar TODAS LAS CORRECTAS - siempre CORRECTAS-, si
marcan correctas de menos o incorrectas, el sistema anula la pregunta.

En referencia a las “Proliferaciones leucocitarias reactivas”, señale cuales de los


siguientes enunciados es CORRECTO. Recuerde que deberá marcar todas las
opciones correctas, considerando que el marcar correctas de menos o 1
respuesta incorrecta, dará por anulado el ejercicio. *

En el contexto de una apendicitis aguda, la adenitis mesentérica observada consta de


ganglios macroscópicamente ingurgitados que histológicamente presentan grandes
centros germinales reactivos acompañados de figuras mitóticas e infiltrado
polimorfonuclear.

Las reacciones leucemoides son patognomónicas de las leucemias mieloides.

La histiocitosis sinusal es el patrón morfológico característico observado en la


linfadenitis aguda inespecífica.

Los cuerpos de Döhle y las granulaciones tóxicas son cambios morfológicos


observados en los neutrófilos de pacientes con sepsis o trastornos inflamatorios
graves como enfermedad de Kawasaki.

Las infecciones crónicas generan leucocitosis por aumento de producción de


factores de crecimiento hematopoyético en médula ósea.

En situaciones de stress, los glucocorticoides disminuyen la extravasación de


leucocitos hacia los tejidos.

La hiperplasia folicular debe ser diferenciada morfológicamente del linfoma folicular


para lo cual una de las características es la conservación de la arquitectura
ganglionar acompañada de una importante variación del tamaño y la forma de los
folículos.

/
En referencia a la "Patología neoplásica prostática", señale cual/es de los
siguientes enunciados es CORRECTO/S. Recuerde que deberá marcar todas las
opciones correctas, considerando que al marcar opciones correctas de menos o
1 respuesta incorrecta, el sistema dará por anulado el ejercicio. *

La gradación según el sistema de Gleason es importante en el cáncer de próstata, ya


que el grado y el estadío son los mejores factores predictivos del pronóstico.

La diseminación hematógena en el carcinoma de próstata se da fundamentalmente a


huesos del esqueleto axial generando metástasis típicamente osteoblásticas.

En la gran mayoría de los casos de los carcinomas de próstata surge en la zona


central a nivel periuretral.

La determinación sérica de PSA total por sí sola, es diagnóstico de cáncer de


próstata.

Los pacientes que padecen HPB refieren signos y síntomas urinarios como
polaquiuria, tenesmo, nicturia y dificultad para iniciar e interrumpir el chorro de orina.

El hallazgo de PIN de alto grado puede estar asociado con adenocarcinoma (hasta en
un 80% de los casos)

El cáncer de próstata presenta pérdida de E-cadherina al igual que el carcinoma


invasivo de células en anillo de sello gástrico y el carcinoma lobulillar de mama.

En referencia a la "Patología neoplásica del SNC", señale cual/es de los siguientes


enunciados es CORRECTO/S. Recuerde que deberá marcar todas las opciones
correctas, considerando que al marcar opciones correctas de menos o 1
respuesta incorrecta, el sistema dará por anulado el ejercicio. *

En el oligodendroglioma la presencia de mutaciones IDH1, IDH2 se asocia a mejor


pronóstico.

Los oligodendrogliomas anaplásicos con codeleción 1p/19q tienen menor respuesta


a la quimio y radioterapia y consecuentemente peor pronóstico.

Se denominan ependimomas mixopapilares a los tumores ubicados en el filum


terminale de la médula y que contienen elementos papilares sobre un fondo mixoide.

Los gliomas (astrocitomas, oligodendrogliomas y ependimomas) conforman el grupo


más frecuente de tumores cerebrales primarios.

El glioblastoma multiforme se caracteriza por presentar áreas de necrosis y


proliferación vascular.

El glioblastoma se caracteriza por presentar células bipolares con prolongaciones


largas y finas que son GFAP (+) acompañadas de fibras de Rosenthal y cuerpos
granulares eosinófilos.

El meduloblastoma aparece predominantemente en niños y en cerebelo.

/
En referencia al "Cáncer colorrectal", señale cual/es de los siguientes enunciados
es CORRECTO/S. Recuerde que deberá marcar todas las opciones correctas,
considerando que al marcar opciones correctas de menos o 1 respuesta
incorrecta, el sistema dará por anulado el ejercicio. *

No se observa mayor riesgo de carcinoma colorrectal en pacientes que padecen


poliposis juvenil y el Síndrome de Peutz Jeghers.

El Síndrome de Lynch tiene estrecha relación con mutación de los genes MSH2 y
MLH1 e inestabilidad de microsatélites.

Las características microscópicas generales de los adenocarcinomas de colon


derecho e izquierdo son similares.

El estadío B1 de Dukes/Astler-Coller se corresponde a un tumor que invade la


muscular propia pero no la atraviesa y en ausencia de metástasis ganglionares y a
distancia.

El seguimiento ideal de un paciente con poliposis adenomatosa familiar (PAF) es por


control endoscópico cada 5 años ya que el riesgo de desarrollar adenocarcinoma
colorrectal es bajo.

La gran mayoría de los adenocarcinomas colónicos son de carácter esporádico vía


APC/Wnt y cumplen con la secuencia clásica de adenoma-carcinoma.

En referencia a las “Neoplasias renales", señale cual/es de los siguientes


enunciados es CORRECTO/S. Recuerde que deberá marcar todas las opciones
correctas, considerando que al marcar opciones correctas de menos o 1
respuesta incorrecta, el sistema dará por anulado el ejercicio. *

El carcinoma renal papilar suele ser unifocal a diferencia del carcinoma renal de
células claras que es multifocal.

Una característica particularmente problemática de los carcinomas renales es su


tendencia a metastatizar difusamente antes de dar lugar a síntomas y signos locales.

Si se comparan 2 pacientes con carcinoma renal de células claras y el mismo estadío


de la enfermedad; el que tiene células más diferenciadas tiene mayor probabilidad de
estar vivo a 5 años.

Los carcinomas papilares con trisomías 12, 16, 22 y 14 tienen un comportamiento


biológico más agresivo.

En el carcinoma de células claras, la mutación para VHL se asocia con las formas
esporádicas y las familiares en casi su totalidad.

El oncocitoma y el carcinoma renal cromófobo derivan de las células intercaladas de


los túbulos colectores y su pronóstico es excelente.

Las células espumosas intersticiales y los cuerpos de psamoma son características


histológicas del carcinoma renal cromófobo.
/
EXAMEN FINAL 26 DE NOVIEMBRE 2020
*Obligatorio

Instrucciones
RECUERDE:
- En las preguntas tipo múltiple choice, 1 sola opción es la correcta.
- En las preguntas tipo casilla de verificación, deberá identificar todos los enunciados CORRECTOS teniendo en cuenta
que el marcar enunciados correctos de menos o enunciados incorrectos, la pregunta se da automáticamente por anulada.

Un niño de 12 años tuvo faringitis sin tos, con exudados amigdalinos y fiebre de 38,3°C hace 3
semanas. Cultivo faríngeo positivo para Streptococcus β-hemolítico del grupo A. Durante el
seguimiento el niño no tiene fiebre. La frecuencia cardíaca es de 85 latidos x’; la frecuencia
respiratoria de 18 respiraciones x’ y la TA de 90/50mmHg. En la auscultación se detecta un
soplo mitral diastólico y hay estertores difusos en ambos pulmones. En los 2 días siguientes
presenta varios episodios de fibrilación auricular acompañados de signos de insuficiencia aguda
ventricular izquierda. ¿Cuál de las siguientes alteraciones que se están produciendo en el
corazón del niño es más probable que sea la causa de la insuficiencia cardíaca del ventrículo
izquierdo? *
1 punto

Miocarditis
Fibrosis de la válvula mitral
Amiloidosis
Pericarditis fibrinosa

¿Cuál de los siguientes enunciados es CORRECTO respecto a los adenomas hipofisarios? *


1 punto

Son necesarias las técnicas inmunohistoquímicas para la adecuada clasificación y diagnóstico de los
adenomas hipofisarios.
El estudio inmunohistoquímico de TTF-1 es de utilidad en el diagnóstico de tumores de adenohipófisis.
Es frecuente su malignización.
La microscopía electrónica es necesaria para el diagnóstico y clasificación actual de los adenomas de
hipófisis.

Una mujer de 26 años tiene disnea progresiva con fatiga desde hace 2 años. En el examen
físico se constata edema en miembros inferiores, ingurgitación yugular y hepatomegalia. La
auscultación del tórax revela campos pulmonares limpios. La TAC muestra un aumento de la
silueta del corazón derecho. Un cateterismo cardíaco registra un aumento de la presión de la
arteria pulmonar sin gradientes en la válvula pulmonar ni presencia de comunicaciones
auriculoventriculares. Se toma una biopsia transbronquial y se demuestran lesiones arteriales
plexiformes. ¿Cuál de las siguientes mutaciones es más probable que cause la enfermedad
pulmonar? *
1 punto

renina
fibrilina 1
receptor morfogénico óseo 2 (BMPR2)
péptido natriurético de tipo B (BNP)
En referencia a las "Lesiones pigmentarias de la piel", señale cual/es de los siguiente/s
enunciado/s es CORRECTO/S. Recuerde que deberá marcar todas las opciones correctas,
considerando que al marcar opciones correctas de menos o 1 respuesta incorrecta, el sistema
dará por anulado el ejercicio. *
1 punto

Los nevos intradérmicos son lesiones maculares, planas, que crecen formando nidos o cordones en la
dermis, pudiendo adoptar sus células más profundas una forma ligeramente fusada, fenómeno denominado
neurotización.
El concepto de melanoma ulcerado empeora el pronóstico.
Los bordes de los melanomas son irregulares en contraste con los bordes redondeados, lisos y uniformes de
los nevos melanocíticos.
El Melanoma se diferencia histológicamente de los nevos por presentar atipía y carecer de rasgos de
“maduración" o "neurotización" en profundidad.
El melanoma nodular posee un marcado crecimiento vertical prácticamente desde su inicio, que se vincula
con su invasividad precoz.
El melanoma extensivo superficial se caracteriza por un extenso periodo de crecimiento radial, por lo que, a
diferencia del resto de los melanomas, éste se considera una variante benigna.

Respecto de la "Patología neoplásica que compromete el corazón", señale cuál de los


siguientes enunciados es CORRECTO: *
1 punto

Los angiosarcomas son las neoplasias malignas que más frecuentemente comprometen el corazón.
Los mixomas suelen localizarse a nivel ventricular y pueden ser sésiles o pediculados.
Una complicación potencialmente grave del mixoma es que, debido a su consistencia blanda, pueda
fragmentarse y generar un émbolo neoplásico.
Los rabdomiosarcomas son las neoplasias primarias del corazón que son más frecuentemente halladas en
pacientes pediátricos.

Un hombre de 32 años con antecedentes de enfermedad de Crohn ha notado la aparición de


un nódulo rojo doloroso de 5cm., rodeado de una zona pálida en la parte inferior de la pierna
izquierda durante la semana previa. Su temperatura es de 37,3°C. Esta lesión se resuelve en
las 3 semanas siguientes, pero aparece otra en la pantorrilla contralateral. En la biopsia se
observa “infiltrado inflamatorio mixto en la dermis con neutrófilos, células redondas y células
gigantes, que afectan al tejido adiposo, además de intenso edema”. Las lesiones se resuelven
sin cicatriz, pero durante el año siguiente aparecen más. ¿Cuál es el diagnóstico más
probable? *
1 punto

Eritema nudoso
Molusco contagioso
Impétigo
Dermatitis herpetiforme

En relación a los tumores carcinoides pulmonares, indique la respuesta INCORRECTA: *


1 punto

Son neoplasias epiteliales malignas de alto grado, con una estrecha relación al consumo de cigarrillos,
constituídos por células redondas, ovaladas o fusiformes con escaso citoplasma, bordes mal definidos y
cromatina nuclear finamente granulada.
Histológicamente están constituidos por estructuras celulares organoides, trabeculares o en empalizada,
separadas por un estroma fibrovascular delicado, similares a las lesiones descriptas en el tubo digestivo.
Son tumores que crecen a modo de masas digitiformes o polipoideas esféricas, proyectadas hacia la luz del
bronquio y revestidos habitualmente por una mucosa intacta.
La mayoría se encuentran confinados a los bronquios principales, mientras que otros atraviesan la pared
bronquial para abrirse en abanico hacia los tejidos adyacentes, produciendo una característica lesión en
ojal.

Si bien la mayoría de los adenocarcinomas esofágicos se asocian a esófago de Barrett, la


mayoría de las personas con esófago de Barrett no desarrolla tumores esofágicos. Esta
entidad se caracteriza por la presencia de metaplasia intestinal dentro de la mucosa escamosa
del esófago. ¿Cuál es su localización anatómica más frecuente? *
1 punto

Todas son correctas


Tercio medio del esófago
Tercio superior del esófago
Tercio inferior del esófago

Un hombre de 55 años observa un aumento de tamaño gradual del escroto a lo largo del
último año. Este crecimiento no es doloroso, aunque se asocia a sensación de pesadez. No
refiere dificultades sexuales. El examen físico no revela lesiones en la piel escrotal ni masas
evidentes, pero el escroto aparece aumentado de tamaño, edematoso y blando en ambos
lados. El resultado de la transiluminación es positivo, permitiendo el paso de la luz a su través.
¿Cuál es el diagnóstico más probable? *
1 punto

Hidrocele
Orquitis
Varicocele
Seminoma

Se recibe una pieza de colecistectomía cerrada y tensa, con serosa lisa, brillante, rojiza, en
sectores congestiva. A la apertura, se constata la presencia de abundante bilis turbia y varios
litos, facetados, amarillentos. Mucosa rojiza, edematosa, en algunos sectores de aspecto
ulcerado y hemorrágico. A la microscopía se observa escaso infiltrado inflamatorio a predominio
polimorfonuclear. ¿Cuál de los siguientes, considera Usted, es el diagnóstico más probable? *
1 punto

Colecistitis crónica
Colecistitis aguda alitiásica
Colecistitis aguda gangrenosa
Colecistitis aguda litiásica

Un paciente de 61 años, ex tabaquista, con pérdida de peso en los últimos meses e ictérico
presenta una obstrucción de la vía biliar extrahepática por un tumor que compromete la cabeza
del páncreas. Se lo extrae a través de una duodenopancreatectomía cefálica. Al corte, el
examen macroscópico de la pieza mostró un área irregular, de límites imprecisos y con aumento
de la consistencia respecto del tejido normal adyacente. Al microscopio, se observó la presencia
de estructuras tubulares con cierta irregularidad, patrón de crecimiento infiltrante, invasión
perineural y áreas de extensa fibrosis. ¿Cuál es el diagnóstico histopatológico de la lesión
descripta? *
1 punto
Adenocarcinoma ductal
Carcinoma de células acinares
Pancreatoblastoma
Glucagonoma

Mujer de 42 años con nódulo único de 3,5 x 3 cm en lóbulo tiroideo izquierdo, cuya punción con
aguja fina determinó que la lesión era altamente sospechosa de una lesión neoplásica de las
células del epitelio folicular, por lo que se le realiza una extirpación quirúrgica de la glándula. Al
corte, la pieza mostró una lesión única, nodular, bien delimitada por una cápsula delicada que la
separaba del parénquima tiroideo adyacente. Al analizarla al microscopio, se identificaron
pequeños folículos con escaso coloide, en cuyas células de revestimiento no se observaron
alteraciones nucleares típicas. La totalidad de la cápsula estaba indemne. En un sector alejado
de la lesión se observó un grupo de células similares a las localizadas en el interior del nódulo
dentro de una vénula. Responda, ¿cuál es su diagnóstico? *
1 punto

Carcinoma medular
Hiperplasia folicular
Adenoma folicular
Carcinoma folicular

Un varón de 55 años sufrió una súbita pérdida de conocimiento hace 4 meses. En el examen
físico actual presenta hemiplejía. La TAC cerebral demuestra un gran espacio quístico en la
región parietal derecha. La angiografía por RM revela oclusión de una arteria cerebral periférica
en la unión sustancia gris-sustancia blanca cercana a la lesión. ¿Cuál de los siguientes
procesos patológicos subyacentes es más probable que presente? *
1 punto

insuficiencia renal crónica con hipertensión


cardiopatía isquémica con trombosis ventricular izquierda
adenocarcinoma de colon con síndrome de Trousseau
SIDA con recuento de linfocitos T CD4+ bajo

Indique cuál de las siguientes descripciones se corresponde a una tiroiditis en el contexto del
postparto. *
1 punto

Presencia de masa en el cuello indolora acompañada inicialmente por tirotoxicosis que luego de unos meses
evoluciona a normalización de los niveles de hormonas tiroides en sangre. Histológicamente hay presencia
de un infiltrado a predominio linfocitario asociado a centros germinales en el parénquima tiroideo.
Glándula firme, con aumenta de volumen unilateral o bilateral. Histológicamente se constatan ruptura de los
folículos con extravasación de coloide acompañado de la presencia de macrófagos formando una
prominente reacción granulomatosa.
Fibrosis extensiva de la gandula tiroides y tejidos adyacentes del cuello, se constata una masa fija y dura.
Aumento difuso y simétrico indoloro de la glándula, histológicamente caracterizado por la presencia de un
infiltrado inflamatorio mononuclear acompañado de centros germinales y células plasmáticas. Los folículos
tiroideos son atróficos con presencia de células eosinófilas con citoplasma granular.

Un hombre de 25 años sufre súbitamente un paro cardíaco y es reanimado. En la exploración


los parámetros vitales son normales. El ecocardiograma muestra un ventrículo izquierdo normal
y un ventrículo derecho dilatado, con un notable adelgazamiento parietal. 6 meses después
fallece y en la autopsia se observa extensa sustitución fibroadiposa del miocardio, sin
inflamación (imagen). ¿Cuál de las siguientes causas es más probable que explique estos
hallazgos? *
1 punto

Cardiomiopatía arritmogénica
Miocardiopatía chagásica
Miocardiopatía hipertrófica de base genética
Síndrome del QT largo

En referencia a los "Tumores Tiroideos", señale cual/es de los siguiente/s enunciado/s es


CORRECTO/S. Recuerde que deberá marcar todas las opciones correctas, considerando que al
marcar opciones correctas de menos o 1 respuesta incorrecta, el sistema dará por anulado el
ejercicio. *
1 punto

El carcinoma papilar tiroideo es el tipo histológico menos frecuente y su diagnóstico histológico se basa
especialmente en la observación de estructuras papilares.
El resultado de una PAAF (punción aspiración con aguja fina) de nódulo tiroideo se informa como una lesión
folicular, pero no permite diferenciar un adenoma de un carcinoma folicular.
El 80% de los carcinomas medulares forman parte de Síndromes NEM siendo multicéntricos y bilaterales.
El carcinoma papilar tiene mal pronóstico cuando se diagnostica con presencia de metástasis ganglionares
únicas en ganglios cervicales regionales.
La presencia de mutaciones BRAF en los carcinomas papilares se relaciona con peor pronóstico.
El carcinoma medular es una neoplasia neuroendócrina cuyo origen son las células parafoliculares por lo
cual el dosaje de calcitonina es útil para el diagnóstico y seguimiento post quirúrgico.

En referencia a la "Patología tumoral ósea y de partes blandas", señale cuál de los siguientes
enunciados es INCORRECTO. *
1 punto

Para el diagnóstico presuntivo de tumores óseos es importante conocer la edad del paciente y contar con
estudios por imágenes de la lesión.
La mayoría de los tumores de partes blandas se asocian a síndromes genéticos.
El liposarcoma bien diferenciado o tumor lipomatoso atípico de las extremidades es la va riante de
liposarcoma más frecuente.
Las localizaciones más frecuentes de los tumores de partes blandas se encuentran en miembros inferiores,
tronco y retroperitoneo.

Paciente femenina de 43 años de edad con antecedente de infección de vías urinarias bajas,
diabética. Presenta fiebre, dolor costal, disuria. La paciente se descompensa por su estado
diabético y fallece. En la autopsia, al examen macroscópico del riñón se evidencia superficie
con abscesos delimitados y amarillentos. Microscópicamente se ve necrosis licuefactiva que
afecta el parénquima renal, abscesos y neutrófilos intratubulares. La descripción es compatible
con: *
1 punto

Pielonefritis aguda
Carcinoma de células renales
Necrosis tubular aguda
Intoxicación por AINES

Un hombre de 60 años presenta tos sin expectoración y lumbalgia durante los últimos 4 meses,
hematuria macroscópica desde hace 1 mes. Es tabaquista de 300paquetes/año. Al examen
físico se evidencia: TA 175/110mmHg e hipersensibilidad a la percusión de la parte alta de la
espalda. Análisis de orina: sangre 3+, sin cilindros ni cristales. La TAC de tórax identifica un
nódulo sólido de 4cm en lóbulo inferior derecho pulmonar y lesiones líticas de 1-2cm en
vértebras torácicas. ¿En cual de las siguientes estructuras urinarias es más probable que se
haya originado esta neoplasia? *
1 punto

Cúpula vesical
Cáliz
Uréter
Corteza renal

En referencia al "Carcinoma in situ de mama”, señale cual/es de los siguientes enunciados es


CORRECTO/S. Recuerde que deberá marcar todas las opciones correctas, considerando que al
marcar opciones correctas de menos o 1 respuesta incorrecta, el sistema dará por anulado el
ejercicio. *
1 punto

Histológicamente el carcinoma ductal in situ se divide en dos subtipos arquitecturales principales:


comedociano y no comedociano.
Tanto el CLIS como el CA invasor lobulillar suelen expresar en la mayoría de los casos receptores de
estrógeno y progesterona.
El CDIS comedoniano presenta células con necrosis central y atipía de alto grado, pudiéndose visualiza r a la
mamografía como calcificaciones que varían desde patrones lineales a ramificadas.
A diferencia del CA invasor lobulillar, el CLIS se caracteriza por la expresión de E-cadherina.
Los CDIS son imposibles de diagnosticar mediante una mamografía.
El riesgo de recidiva del CDIS depende de: alto grado nuclear, extensión de la enfermedad y compromiso de
márgenes quirúrgicos.

Respecto a la glomeruloesclerosis focal y segmentaria, señale la opción CORRECTA. *


1 punto

Microscópicamente, se observa compromiso de la totalidad del glomérulo, con colapso de asas capilares y
esclerosis, con adherencias a cápsula de Bowman.
Con microscopía electrónica se puede observar borramiento difuso de los podocitos.
El trasplante renal se considera tratamiento curativo y efectivo.
Con técnicas de inmunofluorescencia se pueden observar depósitos de IgG, característicos de esta entidad.

Un hombre de 75 años ha desarrollado una lumbalgia con dolor de ambas caderas y del hombro
derecho progresivos durante los últimos 3 años. Refiere que ahora usa sombreros de mayor
tamaño. Las radiografías muestran un estrechamiento del espacio articular con esclerosis del
hueso adyacente. La biopsia de hueso de la cresta ilíaca revela pérdida de las trabéculas
normales con un patrón en mosaico y aumento del número de osteoclastos y osteoblastos.
¿Cuál de las siguientes complicaciones es más probable que presente el paciente como
consecuencia de este proceso? *
1 punto

Espondilitis anquilosante
Osteoma osteoide
Displasia fibrosa
Osteosarcoma

En referencia a la “Patología vesical”, señale cuales de los siguientes enunciados es


CORRECTO. Recuerde que deberá marcar todas las opciones correctas, considerando que al
marcar opciones correctas de menos o 1 respuesta incorrecta, el sistema dará por anulado el
ejercicio. *
1 punto

La cistitis aguda es una entidad no relacionada con la pielonefritis bacteriana.


La anomalía congénita más frecuente y grave es el reflujo vesicoureteral.
El 95% de los tumores vesicales poseen origen mesenquimatoso.
En los carcinomas uroteliales la invasión de la muscular de la mucosa tiene importancia pronóstica.
Las cistitis agudas se caracterizan macro y microscópicamente por presentar una mucosa hiperémica e
infiltrado de neutrófilos con o sin exudado.
Las lesiones precursoras más frecuentes del carcinoma urotelial invasivo son los tumores papilares no
invasivos que se originan a partir de la hiperplasia papilar urotelial.

Paciente masculino de 55 años, actualmente trabaja en el rubro de la construcción como


capataz. Refiere molestias en la garganta y pérdida de voz progresiva. Tabaquista desde los 17
años. En la laringoscopía se observó tumoración exofítica ulcerada laríngea, con compromiso
de movilidad de cuerda vocal derecha. La tumoración se extiende a supraglotis comprometiendo
las cuerdas vocales falsas y el repliegue aritenoepiglótico derecho, hay compromiso de la luz
glótica. Usted razona lo siguiente (marque la opción CORRECTA): *
1 punto

Esta entidad es más frecuente en adultos jóvenes con papilomatosis laríngea.


Este tipo de lesiones se relaciona con grandes esfuerzos laríngeos.
Muy probablemente se trate de un carcinoma escamoso infiltrante.
Muy probablemente se trate de un adenocarcinoma infiltrante.

Un paciente de 23 años de edad, inmunodeprimido y con (otitis media aguda presenta


alteraciones de la conducta, convulsiones y fiebre alta de 48hs de evolución. Se solicita una
RMN con contraste en la que se observa una zona redondeada hipodensa rodeada de un halo
hipercaptante en lóbulo frontal. Se le solicita una biopsia estereotáxica de la lesión.
Histológicamente se observa “abundante material necrótico, densos infiltrados de tipo
polimorfonuclear y gliosis reactiva”. ¿Cuál es para Ud. el diagnóstico más probable? *
1 punto
Absceso piógeno
Quiste coloide
Tuberculoma
Glioblastoma

Paciente de 46 años de edad consulta por convulsiones repetidas y cefaleas de 15 días de


evolución. Se solicita RMN con contraste: “masa hipercaptante con focos de microcalcificación
ubicada a nivel del lóbulo frontal”. Se realiza cirugía. Histológicamente se observa una
“proliferación de células uniformes de núcleo redondo e hipercromático rodeado de escaso
citoplasma con halos perinucleares acompañadas por una red de capilares anastomosados.
Pueden visualizarse focos microscópicos calcificados”. ¿Cuál es para Ud. el diagnóstico más
probable? *
1 punto

Xantoastrocitoma pleomòrfico (GII/IV)


Oligodendroglioma clásico (GII/IV)
Astrocitoma anaplàsico (GIII/IV)
Glioblastoma multiforme (GIV/IV)

Paciente de 57 años de edad tabaquista de larga data, consultó por hemiparesia izquierda. Se
realizó RMN con contraste observándose una lesión tumoral hipercaptante de 3,5cm de
diámetro ubicada en los ganglios de la base que comprometía la cápsula interna. En la biopsia
estereotáxica se observa “proliferación de células atípicas de aspecto epitelial, formando
aisladas perlas córneas, dispuestas en masas que infiltraban el tejido nervioso en forma
irregular”. IHQ: citoqueratinas(+); GFAP(proteína gliofibrilar ácida) (-), NF(neurofilamentos) (-) y
NSE(enolasa-neuro específica) (-). ¿Cuál es para Ud. el diagnóstico más probable? *
1 punto

Astrocitoma difuso (GII/IV)


Metástasis de carcinoma epidermoide pulmonar
Metástasis de adenocarcinoma de próstata
Metástasis de carcinoma de células pequeñas pulmonar

Un paciente de 28 años de edad acudió a la consulta por un agrandamiento indoloro de su


testículo derecho. Se le realizó un estudio serológico que evidenció aumento de la beta-HCG y
LDH, estando la alfa-fetoproteína en valores normales. Un mes más tarde se le practicó una
orquiectomía derecha, y la pieza quirúrgica presentó, al corte, una masa homogénea, sólida y
ligeramente blanquecina con algunas lobulaciones. Su examen histológico revela la presencia
de células neoplásicas grandes, con citoplasma claro y núcleos con nucléolos evidentes, con
aisladas células gigantes de tipo sinciciotrofoblasto, separadas por tabiques de tejido conectivo
en donde se hallan linfocitos y algunos plasmocitos. Respecto de este caso: *
1 punto

El aumento de la beta-HCG en el contexto de un seminoma se vincula a un muy mal pronóstico.


La descripción coincide con el diagnóstico de un seminoma de tipo espermatocítico.
Los agrandamientos testiculares vinculados a un tumor se presentan comúnmente con dolor.
La descripción histológica de la pieza quirúrgica no corresponde a un carcinoma embrionario.

En referencia a los “Síndromes Mieloproliferativos”, señale la opción INCORRECTA: *


1 punto
La Policitemia Vera y la Mielofibrosis primaria se asocian a mutaciones puntuales activadoras de la tirocina
cinasa JAK2.
La mielofibrosis primaria se caracteriza por la presencia de la mutación MPL en la mayoría de los casos.
Todos los Sindromes Mieloproliferativos pueden transformarse en Leucemias agudas y una fase que se
caracteriza por la fibrosis medular asociada a anemia, trombocitopenia y esplenomegalia.
En más del 90% de los casos de Leucemia Mieloide Crónica el gen BCR-ABL se crea por una translocación
(9;22).

Una primigrávida de 36 años desarrolla edema periférico al final del 2° trimestre de embarazo.
En el examen físico tiene una TA de 155/95mmHg. El análisis de orina revela una proteinuria de
2+, sin presencia de sangre, glucosa o cuerpos cetónicos. El bebé nace de término pero con
bajo peso. La TA de la paciente se normaliza y ya no presenta proteinuria. ¿Cuál de los
siguientes hallazgos patológicos es más probable que se presente al estudiar la placenta? *
1 punto

Mola hidatiforme parcial


Múltiples infartos
Vellosidades hidrópicas
Corioamnionitis

En referencia a las "Patología Intersticial pulmonar", señale cual/es de los siguientes


enunciados es CORRECTO. Recuerde que deberá marcar todas las opciones correctas,
considerando que al marcar opciones correctas de menos o 1 respuesta incorrecta, el sistema
dará por anulado el ejercicio. *
1 punto

La fibrosis pulmonar idiopática puede presentar cuadros de exacerbación aguda que histopatológicamente
se traducen en daño alveolar difuso o neumonía en organización.
El patrón de neumonía intersticial usual se observa exclusivamente en la fibrosis pulmonar idiopática.
El patrón histológico de la neumonía intersticial usual consiste en áreas parcheadas en las cuales las zonas
afectadas se caracterizan por lesiones en diferentes momentos evolutivos con focos fibroblásticos, fibrosis
densa y panalización.
La fibrosis pulmonar idiopática es un síndrome clínico-patológico caracterizado por fibrosis intersticial
progresiva sin destrucción del parénquima pulmonar.
El fenómeno fibrosante produce en su evolución, hipertensión pulmonar y cor pulmonale.
Tanto en la neumonía intersticial usual como en la neumonía intersticial no específica, las lesiones
predominan en lóbulos inferiores.

La microfotografía presentada corresponde a un adenocarcinoma de próstata de tipo acinar.


Según el Sistema de Gleason, ¿qué patrón histológico de diferenciación glandular le asignaría
Usted a las células tumorales que se observan en la microfotografía? *
1 punto
Patrón histológico 5 según el Sistema de Gleason
Patrón histológico 3 según el Sistema de Gleason
Patrón histológico 4 según el Sistema de Gleason
No se puede asignar ningún patrón histológico según el Sistema de Gleason

En referencia a la entidad “Linfoma de Hodgkin”, señale la opción CORRECTA: *


1 punto

El tipo “Esclerosis Nodular” es el más frecuente de los Linfomas Hodgkin y se caracteriza por presentar
histológicamente nódulos circunscriptos por bandas colágenas, células lacunares y expresa CD15, CD30 Y
PAX5.
Se describe a la célula de Reed Sternberg típica como una célula pequeña mononuclear y citoplasma
granular.
El tipo “Celularidad Mixta” es el menos frecuente de los tipos de Linfoma Hodgkin y no se ha encontrado
asociación con el VEB.
En el tipo “Predominio Linfocitario” se observa el 90% de positividad para VEB.

Paciente de 60 años, diabética, hipertensa y obesa, consulta a su ginecólogo por presentar


metrorragia de 2 meses de evolución. Luego de examinarla, se solicita una ecografía que
informa útero globoso, en el que se observa una masa de bordes indefinidos y heterogéneos de
aproximadamente 1x3x1.5cm. Se realiza una biopsia que confirma el diagnóstico de carcinoma
endometrial tipo 1. ¿Con la alteración de que gen se asocia la aparición de este tumor? *
1 punto

Rb
CHD4
BRCA1
PTEN

En relación al melanoma marque la opción CORRECTA: *


1 punto

Son tumores de potencial maligno indeterminado que pueden recidivar pero no hacen metástasis.
Son tumores malignos con diferenciación melanocítica y que presenta la mutación activadora del BRAF.
Son tumores malignos con diferenciación melanocítica y que carecen de la mutación activadora del BRAF.
Es un tumor benigno y por lo tanto no tiene capacidad de hacer metástasis.

Señale cuál de los siguientes enunciados es INCORRECTO: *


1 punto

La HTA aumenta el riesgo ateroesclerótico, causa hipertrofia e insuficiencia cardíaca, demencia multiinfarto,
disección aórtica e insuficiencia renal.
El 90-95% de los casos de HTA son idiopáticos y corresponden a la denominada hipertensión arterial
esencial.
En la hipertensión maligna puede observarse arteriolitis necrosante constituida por depósitos fibrinoides y
necrosis de la pared vascular.
La arterioloesclerosis hiperplásica se caracteriza por la calcificación de la membrana elástica interna de las
arterias musculares.

Una mujer de 65 años presenta episodios de gastritis y anemia que no remiten a pesar del
tratamiento médico. Sus valores de laboratorio revelan gastrina en sangre levemente elevada,
por lo que se decide estudiarla más profundamente. El gastroenterólogo le realiza una
videoendoscopia digestiva alta, toma muestras de cada sector de la mucosa gástrica y envía los
fragmentos para su análisis histopatológico. Éstos revelan una marcada disminución de las
células oxínticas a nivel del cuerpo junto a un infiltrado inflamatorio constituido
predominantemente por linfocitos, macrófagos y plasmocitos, con muy escasos y aislados
neutrófilos; hay relativa conservación de la estructura tisular normal en el sector antropilórico.
Teniendo en cuenta los datos consignados, ¿cuál es el diagnóstico más adecuado para esta
paciente? *
1 punto

Gastritis crónica
Enfermedad de Ménétrier
Gastritis linfocítica
Gastritis aguda

En referencia a la enfermedad de Paget ósea, señale cuál de los siguientes enunciados es


INCORRECTO. *
1 punto

Esta enfermedad es monostótica en el 15% de los pacientes y poliostótica en el resto.


Empieza por lo general en la edad adulta (media de 70 años al momento del diagnóstico) y aumenta
progresivamente de frecuencia a partir de esta edad.
Puede dividirse en tres fases secuenciales: una fase osteolítica inicial, una fase osteoclástica/osteoblástica y
por último una fase osteoesclerótica de agotamiento inactiva final.
La mayoría de los pacientes suelen presentar dolores óseos generalizados y se descubren por fracturas
patológicas.

Un niño de 6 años se queja de dolor progresivo en la ingle derecha desde hace 1 semana. El
examen físico revela ganglios linfáticos tumefactos y dolorosos en la región inguinal derecha.
Se realiza la biopsia de un ganglio linfático inguinal, que macroscópicamente se halla
aumentado de tamaño, de coloración gris rojiza y aspecto congestivo. En el examen histológico
exhibe centros germinales agrandados, de tamaño variable, conteniendo numerosas mitosis. Se
encuentra moderado a intenso infiltrado de neutrófilos en ubicación parafolicular y sinusoidal.
¿Cuál es la causa más probable de estas alteraciones histológicas? *
1 punto

linfadenitis aguda inespecífica


leucemia linfoblástica aguda
sarcoidosis
linfoma folicular
A una paciente de 41 años con dolor abdominal, se le realizó una ecografía ginecológica
presentando en el ovario derecho una lesión sólida, bien delimitada, de 6 cm de diámetro. Se la
extirpó, observándose una tumoración blanquecino-amarillenta encapsulada, dura, constituida
por una proliferación del estroma fibroso con algunas células con abundante contenido lipídico.
¿Cuál es el diagnóstico más probable? *
1 punto

Fibrotecoma
Cistoadenoma seroso
Cistoadenofibroma
Disgerminoma

Atrás

Enviar
Nunca envíes contraseñas a través de Formularios de Google.
Este formulario se creó en Facultad de Medicina. Notificar uso inadecuado

Formularios
30/10/2020 EXAMEN FINAL 30 DE OCTUBRE 2020

EXAMEN FINAL 30 DE OCTUBRE 2020


- Complete todos los datos personales.
- Al finalizar la evaluación presione enviar. Una vez enviado el examen ya no se podrán
modificar los datos, ni las respuestas.
- El formulario permanecerá habilitado de 14 a 16hs. Si el alumno, se conectara después del
horario de inicio, el tiempo disponible para la realización del examen será menor.
- Usted podrá resolver el examen 1 sola vez, finalizado el mismo y enviado, el sistema no
permitirá iniciarlo nuevamente.
- Si existe más de un examen enviado, se considerará el primero que haya sido resuelto y
enviado.
- El sistema registra la totalidad de las preguntas una vez que el alumno envía el formulario.
No realiza registros parciales de cada pregunta realizada.

- Los estudiantes que en la fecha y horario asignados para rendir el examen tengan
problemas técnicos -interrupción o fallas de la conectividad a internet, falta de energía
eléctrica, problemas en las herramientas de administración de la evaluación- deberán
informar, de inmediato y en el transcurso del período de evaluación, la dificultad a través del
correo electrónico de docencia (docenciaP-atologia2@gmail.com), adjuntando alguna
constancia (por ejemplo, una captura o foto de la pantalla).

*Obligatorio

¿Cuáles son las principales características histológicas que deben observarse en

.. un tumor astrocitario para considerárselo Glioblastoma? *

@ Hiperplasia endotelial, necrosis tumoral y pseudoempalizadas nucleares.

https://docs.google.com/forms/d/e/1 FAlpQLSfbrPFrxgbkbaExvYNVxWyoxy9-oiXHDgz09DD7fs8ySngwbNviewform 1/21


30/10/2020 EXAMEN FINAL 30 DE OCTUBRE 2020

o Degeneración quística.

o Positividad con GFAP (Proteína gliofibrilar ácida).

Incremento de la densidad celular, con variable pleomorfismo nuclear y presencia de


o mitosis.

Producto de la autopsia realizada a un joven de 20 años con diagnóstico de


muerte súbita cardíaca usted analiza el corazón obteniendo los siguientes datos.
Macroscópicamente se observa "hipertrofia del VI y del septum interventricular
que abomba hacia el infundíbulo de salida de la cavidad izquierda.
Conjuntamente la aurícula izquierda se valora aumentada de tamaño". A nivel
histológico se observa "hipertrofia miocardiocítica masiva con marcado
desarreglo miocítico acompañado de fibrosis intersticial y signos de isquemia
crónica". De acuerdo a esto usted infiere que podría tratarse de ... *

Q una displasia arritmogénica del ventrículo derecho

Q una miocardiopatía restrictiva

@ una miocardiopatía hipertrófica

Q una miocardiopatía dilatada

Una paciente de 72 años de edad con diagnóstico previo de hiperplasia de


células escamosas de la vulva presentó una lesión sobreelevada costrosa de
aproximadamente 0.8cm de diámetro en el labio mayor derecho de la vulva. Se
realiza biopsia, observándose "cordones de células epiteliales con marcada
anisocariosis, disqueratosis y presencia de perlas córneas". ¿Cuál es para Ud. el
diagnóstico más probable? *

Q Liquen escleroso

@ Carcinoma epidermoide queratinizante

Q Enfermedad de Paget extramamaria

.. Q Carcinoma basaloide

https://docs.google.com/forms/d/e/1 FAlpQLSfbrPFrxgbkbaExvYNVxWyoxy9-oiXHDgz09DD7fs8ySngwbNviewform 2/21


30/10/2020 EXAMEN FINAL 30 DE OCTUBRE 2020

Con respecto a los caracteres de las Glomerulopatías a nivel de microscopía


electrónica, indique cuál de las siguientes relaciones de correspondencia es
CORRECTA: *

@ Glomerulopatía membranosa: Depósitos subepiteliales.

o Enfermedad de cambios mínimos: Jorobas subepiteliales.

o Glomerulonefritis post-estreptocóccica: Desaparición de los pedicelos; Ausencia de


depósitos.

o Nefropatía por lgA: No hay depósitos; se ven roturas de la membrana basal; fibrina.

Una paciente de 20 años de edad consultó por presentar anorexia, pérdida de


peso, adinamia e ictericia de varias semanas de evolución. Se le realizó una TAC
en la que se observó una masa única, de bordes irregulares de aproximadamente
7cm de diámetro ubicada en el lóbulo hepático derecho con trombosis portal. Se
toma biopsia de la lesión. Histológicamente se observaron células poligonales
atípicas bien diferenciadas, que crecen formando nidos o cordones separados
por haces densos de colágeno. ¿Cuál es para Ud. el diagnóstico más probable? *

Q Hiperplasia nodular focal

Q Colangiocarcinoma

@ Hepatocarcinoma fibrolaminar

Q Adenoma hepático

..
https://docs.google.com/forms/d/e/1 FAlpQLSfbrPFrxgbkbaExvYNVxWyoxy9-oiXHDgz09DD7fs8ySngwbNviewform 3/21
30/10/2020 EXAMEN FINAL 30 DE OCTUBRE 2020

Paciente de sexo femenino de 36 años de edad que presenta un nódulo


dominante en lóbulo tiroideo derecho. Se realiza una ecografía y se informa
nódulo hipoecoico, sólido-quístico, de bordes difusos, con microcalcificaciones
y aumento del flujo sanguíneo intranodular, que mide 45 mm de diámetro. Los
extendidos citológicos obtenidos mediante punción-aspiración con aguja fina
del nódulo muestran células con núcleos de aspecto ópticamente vacío, con
hendiduras nucleares, surcos intranucleares (grooves) y seudoinclusiones
nucleares. Dichas células se disponen conformando pequeños folículos. ¿Cuál de
los siguientes considera Usted es el diagnóstico más probable? *

O Carcinoma medular de tiroides.

O Adenoma folicular de tiroides.

@ Carcinoma papilar de tiroides.

O Carcinoma folicular de tiroides.

Un hombre de 65 años comenzó con debilidad, fatiga, linfadenopatía


generalizada, hepatoesplenomegalia, cuadro de anemia moderada con
trombocitopenia y marcada leucocitosis (con un 80% de linfocitos). Se efectúo
biopsia ganglionar constatándose "reemplazo difuso de la arquitectura
ganglionar por una población de células linfoides pequeñas con núcleo
redondeado con cromatina condensada y escaso citoplasma. La biopsia de
médula ósea también muestra agregados intertrabeculares de linfocitos
pequeños". El estudio inmunofenotípico de las células tumorales reveló que son

.. ciclina D1 (-), PanB (+),CDS(+) y CD23 (+).¿Cuáles para Ud. el diagnóstico más
probable? *

https://docs.google.com/forms/d/e/1 FAlpQLSfbrPFrxgbkbaExvYNVxWyoxy9-oiXHDgz09DD7fs8ySngwbNviewform 4/21


30/10/2020 EXAMEN FINAL 30 DE OCTUBRE 2020
U umoma ro11cu1ar

O Linfoma de células del manto

O Linfoma inmunoblástico

@ Leucemia linfocítica crónica

¿Cuál de las siguientes características se encuentra asociada a la Enfermedad de


Crohn? *

o Se constituye como una enteritis regional con frecuente afectación ileal.

o La complicación más frecuente es el megacolon tóxico.

La distribución de las lesiones es difusa conformando un cuadro de rectocolitis-


úlcero-hemorrágica.

o No se observa la presencia de granulomas y/o fístulas.

Femenina de 64 años, concurre a la consulta por presentar metrorragia de 2 días


de evolución. Refiere un episodio similar hace dos meses. La ecografía
ginecológica informa endometrio de 13mm heterogéneo. Se realiza aspirado
manual endouterino cuya anatomía patológica informa glándulas hiperplásicas
de patrón complejo con células atípicas. Con estos datos aportados ¿Cuál es el
diagnóstico más certero? *

O Adenocarcinoma de endometrio tipo 2

@ Hiperplasia endometrial atípica

Q Adenocarcinoma de endometrio tipo 1 moderadamente diferenciado

O Adenocarcinoma de endometrio tipo 1 bien diferenciado

..
https://docs.google.com/forms/d/e/1 FAlpQLSfbrPFrxgbkbaExvYNVxWyoxy9-oiXHDgz09DD7fs8ySngwbNviewform 5/21
30/10/2020 EXAMEN FINAL 30 DE OCTUBRE 2020

Un varón de 80 años ha presentado dificultad para iniciar la micción y nicturia en


el último año. Además, ha sufrido lumbalgia progresiva en los últimos 6 meses. Al
tacto rectal se encuentra una próstata dura e irregular. La gammagrafía ósea
revela áreas de captación en la vértebras torácicas y lumbares. Las pruebas de
laboratorio revelan: fosfatasa alcalina sérica de 1200 U/1 y antígeno prostático
específico de 70 ng/ml. La tasa de nitrógeno ureico en sangre es de 44 mg/dl y la
de creatinina sérica de 3,8 mg/dl. Se obtienen muestras transrectales de biopsia
de todos los lóbulos prostáticos. El estudio histológico de las mismas revela que
más del 90% del tejido está constituido por una infiltración de cordones y
láminas de células con núcleos pleomórficos hipercromáticos, con nucléolos
evidentes y escaso citoplasma. ¿Cuál de las siguientes clasificaciones es la mejor
para la enfermedad de este paciente? *

o Gleason 3,3

o Gleason 2,2

@ Gleason 5,5

o Gleason 1,1

En referencia a los "Tumores del Sistema Nervioso Central", cuál de las siguientes
afirmaciones es CORRECTA: *

o El astrocitoma pilocítico es considerado un tumor infiltrante, grado 11/IV, por lo tanto


su ki67 es elevado.

En los niños la ubicación más frecuente son los hemisferios cerebrales .

.. o Algunos tumores gliales con hallazgos histológicos de bajo grado infiltran regiones
extensas en consecuencia son de mal pronóstico.

https://docs.google.com/forms/d/e/1 FAlpQLSfbrPFrxgbkbaExvYNVxWyoxy9-oiXHDgz09DD7fs8ySngwbNviewform 6/21


30/10/2020 EXAMEN FINAL 30 DE OCTUBRE 2020

Q En los adultos la ubicación más frecuente es en fosa posterior.

Paciente de sexo masculino de 15 años de edad que presenta una lesión


osteoblástica voluminosa ubicada en metáfisis de tibia proximal, con destrucción
de la cortical ósea y compromiso de tejidos blandos adyacentes. Al microscopio,
se observa marcado pleomorfismo celular con elementos fusiformes y
producción de abundante material osteoide eosinófilo que se dispone en forma
de encaje. Referente a esta patología, señale la opción CORRECTA. *

El tipo más frecuente asienta en metáfisis de huesos largos, es primario,


intramedular, osteoblástico y de alto grado.

o El tipo más frecuente asienta en diáfisis de huesos largos, es secundario,


intramedular, osteoblástico y de alto grado.

o El tipo más frecuente asienta en diáfisis de huesos largos, es primario, intracortical,


osteoblástico, y de bajo grado.

o El tipo más frecuente asienta en metáfisis de huesos largos, es secundario,


intracortical, osteoblástico y de bajo grado.

Un hombre de 50 años presenta náuseas persistentes y vómitos ocasionales. El


examen físico es normal. Se le realiza endoscopia alta y en el fundus gástrico se
observa una pequeña área con ausencia de pliegues. El estudio histopatológico
informa "proliferación neoplásica de células epiteliales atípicas que infiltran la
lámina propia, sin extenderse a la submucosa". Una biopsia hecha al paciente 5
años antes revelaba inflamación crónica con presencia de Helicobacter Pylori.
¿Cuál de las siguientes observaciones es más aplicable a esta neoplasia? *

Q Tiene el aspecto macroscópico de la linitis plástica.

..
@ La supervivencia a 5 años después de la resección es >90%.

Q No desarrolla metástasis en los ganglios regionales .

https://docs.google.com/forms/d/e/1 FAlpQLSfbrPFrxgbkbaExvYNVxWyoxy9-oiXHDgz09DD7fs8ySngwbNviewform 7/21


30/10/2020 EXAMEN FINAL 30 DE OCTUBRE 2020

U Microscópicamente muestra un patrón de células en anillo de sello.

En cuanto a la criptorquidia, ¿cuál de las siguientes opciones es INCORRECTA? *

o En el examen microscópico del testículo afectado se observa importante hialinización


y engrosamiento de la membrana basal.

o Cuando es bilateral puede causar esterilidad.

o Implica riesgo aumentado de desarrollar neoplasia sólo en el testículo no


descendido.

'i'
En el examen microscópico del testículo afectado pueden hallarse focos de
~ Neoplasia de Células Germinales lntratubular (IGCNU)

Mujer de 34 años de edad, sin antecedentes médicos relevantes, entrenando en


el gimnasio, sufre una brusca y fuerte cefalea y posteriormente pierde el
conocimiento. En la guardia le realizan una TAC que revela extensa hemorragia
subaracnoidea en la base del cerebro. ¿Cuál de las siguientes cree que ha sido la
causa? *

@ ruptura de la arteria meníngea media

O hemorragia hipertensiva

O como consecuencia de una angiopatía amiloide

Q ruptura de aneurisma congénito del polígono de Willis

..
https://docs.google.com/forms/d/e/1 FAlpQLSfbrPFrxgbkbaExvYNVxWyoxy9-oiXHDgz09DD7fs8ySngwbNviewform 8/21
30/10/2020 EXAMEN FINAL 30 DE OCTUBRE 2020

Un hombre de 56 años se somete a una radiografía de tórax de rutina como


parte de un examen físico completo. La Rx muestra una lesión única de 3cm
solitaria, redondeada de aspecto numular y radiopaca, de localización central,
con un patrón de calcificación de "palomitas de maíz". Se realiza una biopsia de
pulmón que revela islotes de cartílago maduro benigno revestido por epitelio
respiratorio. ¿Cuál es el diagnóstico más probable? *

@ Hamartoma pulmonar

Q Secuestro extralobar

Q Fibroma pulmonar

Q Tumor carcinoide

En relación al melanoma marque la opción CORRECTA *

o Son tumores de potencial maligno indeterminado que pueden recidivar pero no hacen
metástasis.

Son tumores malignos con diferenciación melanocítica y que presenta la mutación


activadora del BRAF.

o Son tumores malignos con diferenciación melanocítica y que carecen de la mutación


activadora del BRAF.

o Es un tumor benigno y por lo tanto no tiene capacidad de hacer metástasis .

..
https://docs.google.com/forms/d/e/1 FAlpQLSfbrPFrxgbkbaExvYNVxWyoxy9-oiXHDgz09DD7fs8ySngwbNviewform 9/21
30/10/2020 EXAMEN FINAL 30 DE OCTUBRE 2020

Respecto a las cardiopatías congénitas, señale la opción INCORRECTA: *

Las malformaciones que causan una derivación del flujo sanguíneo desde el lado
Q derecho al lado izquierdo del corazón se caracterizan por una hipoxia y consecuente
cianosis que se expresa clínicamente en forma temprana.

o La existencia de comunicación interauricular incrementa el riesgo de sufrir


potenciales infartos en distintos órganos debidos a una embolia paradójica.

La tetralogía de Fallot consiste en una comunicación interventricular, con la aorta


situada sobre el tabique interventricular, hipertrofia del ventrículo izquierdo y
estenosis de la arteria pulmonar.

La infección congénita por rubéola, la diabetes gestacional y la deficiencia de ácido


Q fálico son factores ambientales implicados en la aparición de cardiopatías
congénitas.

Paciente de sexo femenino consulta por amenorrea, galactorrea y trastornos de


la visión de aproximadamente un mes de evolución. Se solicita TAC en la que se
observa una lesión tumoral de aproximadamente 1cm de diámetro ubicada en la
región selar. Se realiza extirpación quirúrgica de la lesión. Histológicamente se
observa "células epiteliales poligonales con núcleos pequeños, de tamaño
uniforme y citoplasma claro en ausencia del entramado de reticulina". ¿Cuál es
para Ud. el diagnóstico más probable? *

O Coristoma de neurohipófisis

O Craneofaringioma

O Quiste de la hendidura de Rathke

.. @ Adenoma de hipófisis

https://docs.google.com/forms/d/e/1 FAlpQLSfbrPFrxgbkbaExvYNVxWyoxy9-oiXHDgz09DD7fs8ySngwbNviewform 10/21


30/10/2020 EXAMEN FINAL 30 DE OCTUBRE 2020

Un paciente de 58 años, en tratamiento anticoagulante por un reemplazo valvular


aórtico previo, tenía emisión de orina teñida con sangre, que continuó luego de
ajustar su tiempo de protrombina. En la cistoscopía la mucosa vesical estaba
congestiva, con una lesión sobreelevada polipoide irregular en el techo. Se
realizó la extirpación por vía endoscópica; estaba compuesto por ejes conectivo
vasculares papilares anchos, fusionados, revestidos por epitelio urotelial con
proliferación irregular de células epiteliales poliédricas, estratificadas en más de
10 capas, sin conservación de la capa de células en paraguas; con hipercromasia
nuclear, marcada anisocariosis, frecuentes mitosis y presencia de nucléolos, sin
infiltración de la lámina propia. ¿Cuál es para Ud. el diagnóstico más probable? *

@ Carcinoma urotelial de alto grado

O Carcinoma urotelial de bajo grado

O Papiloma invertido

O Papiloma vesical

Una paciente de 44 años, presenta adenopatías indoloras, leve hepato y


esplenomegalia. Se le extirpa una adenopatía cervical derecha, que mide 3cm de
diámetro. En la superficie de corte se observan nódulos blanquecinos.
Microscópicamente se observaron "conglomerados nodulares de linfocitos
atípicos pequeños (centrocitos) y otros más grandes (centroblastos)". IHQ: CD10
(+) y la proteína BCL2 (+).¿Cuáles para Ud. el diagnóstico más probable? *

@ Linfoma de Hodgkin

.. O
O
Linfoma de células T del adulto

Linfoma folicular

https://docs.google.com/forms/d/e/1 FAlpQLSfbrPFrxgbkbaExvYNVxWyoxy9-oiXHDgz09DD7fs8ySngwbNviewform 11/21


30/10/2020 EXAMEN FINAL 30 DE OCTUBRE 2020

O Linfoma difuso de células grandes B

La ecografía obstétrica de una paciente de 27 años con 18 semanas de


gestación, reveló un feto con ambos riñones agrandados, asimétricos, con
superficie irregular. Histológicamente se observaron túbulos inmaduros y
presencia de islotes de mesénquima indiferenciados con focos de cartílago.
¿Cuál es para Ud. el diagnóstico más probable? *

O Hidronefrosis bilateral

O Tumor de Wilms bilateral

@ Displasia renal multiquística

O Poliquistosis renal

Un niño de 8 años sufrió una fractura expuesta de tibia y peroné derechos al


caerse de un árbol; se realizó reducción manual de la fractura y se suturó la piel
del sitio donde el hueso la había atravesado, pero no se hizo nada más. Un año
más tarde, continúa con dolor en la rodilla derecha, y se ha desarrollado un tracto
sinusal de drenaje en la parte lateral externa inferior de la pierna. En una
radiografía de la pierna se observan secuestro óseo e involucro. ¿Cuál de los
siguientes cree Ud. que sería el diagnóstico más probable? *

@ Osteomielitis crónica

O Callo fracturario

O Artritis reumatoidea juvenil

O Sarcoma de Ewing

..
https://docs.google.com/forms/d/e/1 FAlpQLSfbrPFrxgbkbaExvYNVxWyoxy9-oiXHDgz09DD7fs8ySngwbNviewform 12/21
30/10/2020 EXAMEN FINAL 30 DE OCTUBRE 2020

¿Cuál de los siguientes enunciados es INCORRECTO en referencia a los


Mesoteliomas? *

El mesotelioma maligno difuso invade por contigüidad a diafragma o a pared torácica


y a través de las cisuras al pulmón.

El mesotelioma maligno epitelioide debe diferenciarse (a nivel inmunohistoquímico)


Q del carcinoma escamoso dado que ambos comparten una localización y clínica
semejante.

El mesotelioma maligno localizado presenta un crecimiento nodular circunscripto y


o mejor pronóstico con respecto al tipo difuso.

El 90% de los mesoteliomas malignos difusos presentan antecedentes de exposición


o al asbesto con un período de latencia entre 20-40 años.

Un paciente de 31 años de edad consultó por notar un aumento de tamaño del


testículo izquierdo. En la ecografía se observó una lesión tumoral de 4,5cm de
diámetro. Se realizó la orquiectomía. El estudio histopatológico informa
"Seminoma clásico". ¿Cuál de las siguientes descripciones es la correcta con
respecto a esta entidad? *

playas de células grandes, redondeadas con citoplasma claro, núcleo central con
nucléolos evidentes, separadas por delicados septos conectivos con infiltrados
linfocitarios.

Estructuras glomeruloides compuestas por células cuboideas, algunas de las cuales


o contienen glóbulos hialinos eosinofílicos.

o Células redondeadas con abundante citoplasma granular eosinofílico que contiene


cristaloides.

.. o Células grandes con núcleos lobulados e hipercromáticos con marcada atipía,


rodeadas de abundante citoplasma eosinófilo .

https://docs.google.com/forms/d/e/1 FAlpQLSfbrPFrxgbkbaExvYNVxWyoxy9-oiXHDgz09DD7fs8ySngwbNviewform 13/21


30/10/2020 EXAMEN FINAL 30 DE OCTUBRE 2020

Paciente de 57 años de edad tabaquista de larga data, consultó por hemiparesia


izquierda. Se realizó RMN con contraste observándose una lesión tumoral
hipercaptante de 3,5cm de diámetro ubicada en los ganglios de la base que
comprometía la cápsula interna. En la biopsia estereotáxica se observa
"proliferación de células atípicas de aspecto epitelial, formando aisladas perlas
córneas, dispuestas en masas que infiltraban el tejido nervioso en forma
irregular". IHQ: citoqueratinas {+); GFAP(proteína gliofibrilar ácida)(-),
NF(neurofilamentos) (-) y NSE(enolasa- neuro específica)(-). ¿Cuál es para Ud. el
diagnóstico más probable? *

Q Astrocitoma difuso {Gil/IV)

@ Metástasis de carcinoma epidermoide pulmonar

Q Metástasis de adenocarcinoma de próstata

Q Metastasis de carcinoma de células pequeñas pulmonar

Un paciente de 68 años, tuvo dolor lumbar y hematuria. En la ecografía se


observó en riñón izquierdo una masa de 5cm de diámetro en contacto con el
hilio. Se le realizó una nefrectomía, observándose una "lesión tumoral bien
delimitada, de coloración blanco-grisácea, constituida por láminas de células
epiteliales con núcleos ligeramente anisocarióticos, hipercromáticos y
citoplasma eosinófilo pálido (positivo con técnica de Hierro coloidal de Hale) con
halo perinuclear, estando las células de mayor tamaño concentradas alrededor
de los vasos sanguíneos. La vena renal se hallaba invadida por la neoplasia".
¿Cuál es para Ud. el diagnóstico más probable? *

.. Q
@
Angiomiolipoma

Carcinoma renal cromófobo

https://docs.google.com/forms/d/e/1 FAlpQLSfbrPFrxgbkbaExvYNVxWyoxy9-oiXHDgz09DD7fs8ySngwbNviewform 14/21


30/10/2020 EXAMEN FINAL 30 DE OCTUBRE 2020

O Oncocitoma renal

O Carcinoma renal transicional

Una mujer de 50 años presenta dolor en la región que rodea a la rodilla derecha
durante los últimos 2 meses. No tuvo ningún traumatismo, no se palpan masas ni
presenta signo de flogosis en la zona. La resonancia magnética identifica una
masa de 5 cm, más o menos bien delimitada, superior e inferior a la rótula. La
masa se encuentra dentro de los tejidos blandos, sin compromiso óseo. Se
obtiene una biopsia de la masa, cuyo estudio histológico revela un patrón
bifásico de células fusiformes y epiteliales con formación de glándulas. El análisis
del cariotipo de estas células neoplásicas identifica una translocación t(X;18)
(p11;q11). ¿Cuál es el diagnóstico más probable? *

o Osteoblastoma

o Osteosarcoma

® Sarcoma sinovial

o Leiomiosarcoma

Un paciente de 53 años concurre a su médico por presentar lesiones ampollares


en tronco, axila, cara y úlceras poco profundas en la mucosa bucal. Al examen
físico se constata Signo de Nikolsky (+) (al frotar enérgicamente la zona indemne
de la piel se desprende la epidermis). El examen histopatológico de la lesión
ampollar demuestra que la acantólisis suprabasal produce una ampolla
intraepidérmica en la que se identifican células epidérmicas discohesivas. ¿Cuál
es para Ud. el diagnóstico más probable? *

@ Pénfigo Vulgar

O
..
Dermatitis herpetiforme de Dühring - Brocq

O Penfigoide ampolloso

https://docs.google.com/forms/d/e/1 FAlpQLSfbrPFrxgbkbaExvYNVxWyoxy9-oiXHDgz09DD7fs8ySngwbNviewform 15/21


30/10/2020 EXAMEN FINAL 30 DE OCTUBRE 2020

U Eritema polimorfo

Un hombre de 55 años presenta episodios de dolor torácico subesternal grave al


realizar tareas que demandan un esfuerzo moderado. Los episodios han
aumentado en frecuencia y gravedad en el último año, mejorando con
nitroglicerina sublingual. No tiene fiebre y la frecuencia cardíaca es de 78 latidos
x', regular. No se auscultan soplos ni ritmo de galope. Las pruebas de laboratorio
muestran: creatinina 1,1 mg/dl; glucosa 130 mg/dl y colesterol sérico total de 223
mg/dl. ¿Cuál de estas lesiones cardíacas es más probable que presente este
paciente? *

Q estenosis aórtica calcificada

@ ateroesclerosis coronaria

O pericarditis serosa

Q miocardiopatía restrictiva

Una paciente de 40 años concurre a la consulta porque se ha palpado un nódulo


en la tiroides. Usted solicita una punción por aspiración con aguja fina. A la
citología, observa múltiples células foliculares atípicas a especificar, por lo que
decide realizar una biopsia. ¿Qué elementos de la biopsia deberá tener en cuenta
para poder realizar el diagnóstico correcto? *

Ü Células de Hürthle

@ Invasión capsular y vascular

Q Fibrosis

.. Q Atipía marcada

https://docs.google.com/forms/d/e/1 FAlpQLSfbrPFrxgbkbaExvYNVxWyoxy9-oiXHDgz09DD7fs8ySngwbNviewform 16/21


30/10/2020 EXAMEN FINAL 30 DE OCTUBRE 2020

Un hombre de 40 años, fumador de 2 atados de cigarrillos diario, consulta por


tos, hemoptisis y adelgazamiento. En la fibrobroncoscopía se observa una lesión
peribronquial extensa que se biopsia. En el informe histopatológico informa
"células redondas, azules, con escaso citoplasma, límites celulares mal definidos,
cromatina nuclear finamente granulada y nucléolos ausentes. Alto índice
mitótico y focos de necrosis. Las paredes vasculares presentan un llamativo
refuerzo basófilo". IHQ: Cromogranina {+), sinaptofisina {+), CD57 {+).¿Cuáles
para Ud. el diagnóstico más probable? *

@ Carcinoma de células pequeñas

O Carcinoma de células grandes

O Carcinoma epidermoide

O Adenocarcinoma

Usted se encuentra cursando cirugía general en la UDH. Le solicitan que realice


una clase sobre causas de hiperparatiroidismo primario. Durante la búsqueda y
lectura bibliográfica encuentra que las principales causas de esta afectación son
el adenoma paratiroideo, la hiperplasia paratiroidea y el carcinoma paratiroideo.
Durante el armado de la clase se pregunta cuáles son las principales
características que se tiene que tener en cuenta para diferenciar estas entidades
entre sí. (Marque la opción CORRECTA). *

Q El adenoma se diferencia del carcinoma, porque general afecta múltiples glándulas.

Q El carcinoma se diferencia de la hiperplasia, porque en general no hace metástasis.

.. Q El carcinoma se diferencia del adenoma por sus características citológicas .

El carcinoma se diferencia del adenoma por su capacidad de invadir los tejidos

https://docs.google.com/forms/d/e/1 FAlpQLSfbrPFrxgbkbaExvYNVxWyoxy9-oiXHDgz09DD7fs8ySngwbNviewform 17/21


30/10/2020 EXAMEN FINAL 30 DE OCTUBRE 2020
1.,111.,u11ua1 nt::;:,.

¿Cuál de las siguientes asociaciones entre tipo de Pericarditis y etiología es


CORRECTA? *

o Pericarditis fibrinosa y serofibrinosa / tuberculosis.

o Pericarditis purulenta/ radiación, neoplasias y fiebre reumática.

Pericarditis hemorrágica / infarto agudo de miocardio, síndrome de Dressler,


radiación .

o Pericarditis serosa/ enfermedades autoinmunitarias e infecciones virales .

En referencia al "Carcinoma in situ de mama", señale cuál de los siguientes


enunciados es INCORRECTO. *

o Histológicamente el carcinoma ductal in situ se divide en dos subtipos


arquitecturales principales: comedociano y no comedociano.

o Tanto el CLIS como el CA invasor lobulillar suelen expresar en la mayoría de los casos
receptores de estrógeno y progesterona.

o El riesgo de recidiva del CDIS depende de: alto grado nuclear, extensión de la
enfermedad y compromiso de márgenes quirúrgicos

Los CDIS son difíciles de diagnosticar mediante una mamografía .

..
https://docs.google.com/forms/d/e/1 FAlpQLSfbrPFrxgbkbaExvYNVxWyoxy9-oiXHDgz09DD7fs8ySngwbNviewform 18/21
30/10/2020 EXAMEN FINAL 30 DE OCTUBRE 2020

Usted es un residente en la especialidad de Anatomía Patológica y es llamado por


el equipo de ginecología para ir al quirófano por una consulta intraoperatoria. La
paciente tiene 38 años de edad, es obesa y presenta un blastema anexial
derecho de 11x6x5 cm, que en los estudios ultrasonográficos mostró áreas
hiperecogénicas en su interior. La superficie de la pieza que le entregan es lisa y
congestiva, y al cortarla nota múltiples áreas sólidas mientras rezuma un material
amarillento viscoso con algunos pelos. De acuerdo a la macroscopía de la pieza,
que resulta muy característica, usted ya tiene un diagnóstico presuntivo que
debe comunicar a los ginecólogos y consignar formalmente por escrito.
Respecto de esta entidad, responda: *

Q Es un tumor del cordón sexual

Q Su comportamiento biológico suele ser benigno

@ Su extirpación genera el descenso inmediato de í3-HCG

Q En general, se asocia a niveles elevados de CA-125

Un paciente HIV (+) con intensa disfagia fue sometido a una endoscopia en la que
se observaron múltiples ulceraciones superficiales a lo largo del esófago. Se
tomó biopsia de una lesión, observándose inclusiones nucleares y
citoplasmáticas en las células del estroma de la úlcera y del endotelio capilar.
¿Cuál es para Ud. el diagnóstico más probable de esta esofagitis? *

o por sustancias químicas

o por cándidas

.. o
@
por reflujo

por citomegalovirus

https://docs.google.com/forms/d/e/1 FAlpQLSfbrPFrxgbkbaExvYNVxWyoxy9-oiXHDgz09DD7fs8ySngwbNviewform 19/21


30/10/2020 EXAMEN FINAL 30 DE OCTUBRE 2020

Un hombre de 32 años con antecedentes de enfermedad de Crohn ha notado la


aparición de un nódulo rojo doloroso de 5cm., rodeado de una zona pálida en la
parte inferior de la pierna izquierda durante la semana previa. Su temperatura es
de 37,3ºC. Esta lesión se resuelve en las 3 semanas siguientes, pero aparece otra
en la pantorrilla contralateral. En la biopsia se observa "infiltrado inflamatorio
mixto en la dermis con neutrófilos, células redondas y células gigantes, que
afectan al tejido adiposo, además de intenso edema". Las lesiones se resuelven
sin cicatriz, pero durante el año siguiente aparecen más. ¿Cuál es el diagnóstico
más probable? *

Q Molusco contagioso

Q Dermatitis herpetiforme

@ Eritema nudoso

Q Impétigo

En referencia a la "Patología Intersticial pulmonar", señale cuál de los siguientes


enunciados es INCORRECTO. *

o El evento fisiopatogénico central es el fenómeno inflamatorio y de reparación luego


de la injuria de la pared alveolar.

En la asbestosis el patrón fibrosante es similar a la neumonía intersticial usual con


o focos fibroblásticos y diversos grados de fibrosis acompañados por cuerpos de
asbesto.

En el patrón de neumonía intersticial no específica es frecuente observar focos

.. Q
fibroblásticos, membranas hialinas y micropanalización.

La neumonía intersticial descamativa es una lesión asociada al tabaquismo y se


caracteriza por espacios alveolares con macrófagos cargados de pigmento tabáquico

https://docs.google.com/forms/d/e/1 FAlpQLSfbrPFrxgbkbaExvYNVxWyoxy9-oiXHDgz09DD7fs8ySngwbNviewform 20/21


30/10/2020 EXAMEN FINAL 30 DE OCTUBRE 2020

pardo-dorado.

Paciente de 56 años con cuadro de eritrodermia exfoliativa generalizada. Se


realiza biopsia de piel que informa la "presencia en la dermis superior de un
infiltrado constituido predominantemente por células linfoides pequeñas con
núcleos cerebriformes, que además se hallan formando algunos pequeños
microabscesos intraepidérmicos". En el hemograma se observa leucocitosis y
linfocitosis, con presencia de células idénticas a las descriptas en la piel.¿Cuál es
para usted el diagnóstico más probable? *

@ Síndrome de Sezary

O Eritrodermia psoriásica

O Eritema multiforme

O Leucemia linfocítica crónica

Enviar

Este formulario se creó en Facultad de Medicina. Notificar uso inadecuado

Google Formularios

..
https://docs.google.com/forms/d/e/1 FAlpQLSfbrPFrxgbkbaExvYNVxWyoxy9-oiXHDgz09DD7fs8ySngwbNviewform 21/21
DEPARTAMENTO DE PATOLOGÍA - PATOLOGIA II MODALIDAD VIRTUAL
SEGUNDO EXAMEN PARCIAL - TURNO JUEVES TARDE (14 - 18hs.) - 3 de julio del 2020.

1 - Se recibe una pieza de colecistectomía cerrada y tensa, con serosa lisa, brillante, rojiza, en sectores congestiva. A
la apertura, se constata la presencia de abundante bilis turbia y varios litos, fascetados, amarillentos. Mucosa rojiza,
edematosa, en algunos sectores de aspecto ulcerado y hemorrágico. A la microscopía se observa escaso infiltrado
inflamatorio a predominio polimorfonuclear. ¿Cuál de los siguientes, considera Usted, es el diagnóstico más probable?
a) Colecistitis aguda.
b) Colecistitis aguda gangrenosa.
c) Colecistitis aguda colesterolósica.
d) Colecistitis aguda litiásica.
- Kumar, A. and Aster. (2015). 9na.ed. Robbins y Cotran "Patología estructural y funcional". Tubo digestivo. Ed.Elsevier.
pp. (877-878)
- TEÓRICO: Vesícula biliar, diapositiva 32-37

2 - A un paciente de 70 años de edad en estudio por alteración en el hábito intestinal se le diagnostica por colonoscopía
un tumor exofítico de 5cm de diámetro en colon sigmoides; la biopsia informa adenocarcinoma bien diferenciado
infiltrante; no se observan otras lesiones en la mucosa del resto del colon. El paciente es intervenido quirúrgicamente.
De las lesiones enumeradas a continuación, ¿Cuál cree Ud. que puede haber sido precursora del adenocarcinoma
actual con mayor probabilidad?

a) Pólipo juvenil o hamartomatoso


b) Adenoma velloso
c) Pólipo hiperplásico
d) Pólipo carcinoide
- Kumar, A. and Aster. (2015). 9na.ed. Robbins y Cotran "Patología estructural y funcional". Tubo digestivo. Ed.Elsevier.
pp. (807-809)

3 - En relación con las Glomerulopatías, indique la opción CORRECTA:


a) La glomerulonefritis postinfecciosa suele manifestarse como un Sindrome Nefrótico
b) La glomerulonefritis postestretococcica se debe a inmunocomplejos
c) La glomerulonefritis postestreptococcica no se relaciona con la exotoxina piógena estreptocóccica B (SpeB)
d) La glomerulonefritis postestreptocóccica evidencia depósitos de IgA a nivel mesangial
- Kumar, A. and Aster. (2015). 9na.ed. Robbins y Cotran "Patología estructural y funcional". Riñón. Ed.Elsevier. pp.
(909-910)
4 - Paciente femenina de 43 años de edad c antecedente de infección de vías urinarias bajas, diabética. Presenta fiebre,
dolor costal, disuria. La paciente se descompensa por su estado diabético y fallece. En la autopsia al examen
macroscópico del riñón, este, evidencia superficie con abscesos delimitados y amarillentos, microscópicamente se ve
necrosis licuefactiva que afecta el parénquima renal, abscesos y neutrófilos intratubulares. La descripción es
compatible con:
a) Necrosis tubular aguda
b) Intoxicación por AINE
c) Carcinoma de células renales
d) Pielonefritis aguda
- Kumar, A. and Aster. (2015). 9na.ed. Robbins y Cotran "Patología estructural y funcional". Riñón. Ed.Elsevier. pp.
(931)

5 - Llega a la guardia un estudiante de Medicina de sexo masculino de 25 años de edad quien en un autoexamen
testicular se ha palpado una masa no dolorosa en el testículo izquierdo. Se realizan exámenes complementarios con
altas sospechas de un tumor de células germinales testiculares. No impresiona tener linfadenopatías ni síntomas en
otros órganos. El médico de guardia sugiere una biopsia sin pensarlo tanto, mientras el estudiante no está de acuerdo.
¿Por qué no estaría indicada la biopsia testicular?
a) Debido a que tienen tan buen pronóstico estos tumores que no reciben tratamiento estas personas.
b) Debido a que todos los tumores de células germinales tienen un pronóstico similar.
c) Debido al riesgo de diseminación del tumor por la punción.
d) Debido a que se espera la diseminación a ganglios inguinales para hacerla.
- Kumar, A. and Aster. (2015). 9na.ed. Robbins y Cotran "Patología estructural y funcional". Vías urinarias bajas y
aparato genital masculino. Ed.Elsevier. pp. (975-979)

6 - Paciente de 65 años, con antecedentes de fimosis, presenta una lesión ulcerada e indurada de 1,5cm en la superficie
interna del prepucio, cerca del surco coronal. Asimismo, se palpan ganglios inguinales. Se realizó una biopsia,
observándose “Proliferaciòn células epiteliales atípicas con marcada anisocariosis, mitosis atìpicas, hipercromasia,
cromatina irregular y nucleólo evidente, citoplasma eosinófilo con focos de disqueratosis y aisladas perlas còrneas que
infiltran la membrana basal”. ¿Cuál es para Ud. el diagnóstico más probable?

a) Eritroplasia de Queyrat
b) Carcinoma epidermoide bien diferenciado invasivo
c) Enfermedad de Bowen
d) Papulosis bowenoide
- Kumar, A. and Aster. (2015). 9na.ed. Robbins y Cotran "Patología estructural y funcional". Vías urinarias bajas y
aparato genital masculino. Ed.Elsevier. pp. (971-972). Fig. diapositiva 121-Teórico”Aparato genital masculino”
7 - ¿Cuál de los siguientes es un linfoma no Hodgkin de bajo grado?
a) Linfoma folicular.
b) Linfoma de Burkitt.
c) Linfoma difuso de células B grandes.
d) Mieloma múltiple.
- Kumar, A. and Aster. (2015). 9na.ed. Robbins y Cotran "Patología estructural y funcional". Enfermedades de los
leucocitos, ganglios linfáticos, bazo y timo. Ed.Elsevier. pp. (594-602)

8 - Con respecto a la Leucemia linfocítica crónica, señale la opción CORRECTA:


a) No se observa compromiso de ganglios linfáticos
b) La sintomatología es característica
c) Se altera la función inmunitaria por mecanismos inciertos
d) Los frotis de sangre periférica muestran numerosos linfocitos de gran tamaño
- Kumar, A. and Aster. (2015). 9na.ed. Robbins y Cotran "Patología estructural y funcional". Enfermedades de los
leucocitos, ganglios linfáticos, bazo y timo. Ed.Elsevier. pp. (594)

9 - ¿Cuál astrocitoma es de grado I/IV de la Organización Mundial de la Salud?


a) Astrocitoma anaplásico.
b) Astrocitoma difuso.
c) Glioblastoma.
d) Astrocitoma pilocítico.
- Kumar, A. and Aster. (2015). 9na.ed. Robbins y Cotran "Patología estructural y funcional". Sistema Nervioso Central.
Ed.Elsevier. pp. (1306-1309).

10 - Un hombre de 25 años ha padecido cefaleas durante los últimos 5 meses. En ese tiempo, sus familiares han notado
que el afectado no exhibía la misma agudeza mental que en el pasado y que presentaba mayor labilidad emocional.
En un plazo de 2 semanas sufre cuatro convulsiones generalizadas. En el examen físico no se observan edema de papila
ni alteraciones motoras. La TAC craneal revela una masa de 2 cm en el lóbulo frontal derecho. Se le realiza una biopsia
estereotáxica de esta lesión observándose sólo gliosis e indicios de hemorragias recientes y antiguas. La masa es
extirpada e histológicamente se observa un conglomerado de vasos tortuosos de distinto tamaño rodeados de gliosis.
¿Cuál es el diagnóstico más probable?
a) Angiosarcoma
b) Malformación arteriovenosa
c) Aneurisma sacular roto
d) Placa de esclerosis múltiple
- Kumar, A. and Aster. (2015). 9na.ed. Robbins y Cotran "Patología estructural y funcional". Sistema Nervioso Central.
Ed.Elsevier. pp. (1271).
DEPARTAMENTO DE PATOLOGÍA - PATOLOGIA II MODALIDAD VIRTUAL
SEGUNDO EXAMEN PARCIAL - TURNO JUEVES NOCHE (17-21hs.) - 3 de julio del 2020.

1 - Concurre a la consulta un paciente masculino de 62 años que refiere quemazón en la boca del estómago y
antecedente de reflujo gastroesofágico desde hace 10 años. Se realiza endoscopia digestiva alta y se evidencia
características compatibles con Esófago de Barret. Una patología importante que puede desarrollarse a posteriori es:
a) Linfoma de esófago
b) Carcinoma epidermoide
c) Esofagitis por reflujo
d) Adenocarcinoma
- Kumar, A. and Aster. (2015). 9na.ed. Robbins y Cotran "Patología estructural y funcional". Tubo digestivo. Ed.Elsevier.
pp. (758)

2 - En relación con el Cáncer Gástrico, indique la opción CORRECTA:


a) Los tumores GIST presentan positividad para CD117 o C-KIT en inmunohistoquímica.
b) Los GIST se originan en el epitelio de revestimiento gástrico.
c) Los adenocarcinomas gástricos de tipo intestinal suelen infiltrar la pared produciendo una linitis plástica.
d) Los adenocarcinomas suelen presentarse como pequeñas úlceras de bordes deprimidos.
- Kumar, A. and Aster. (2015). 9na.ed. Robbins y Cotran "Patología estructural y funcional". Tubo digestivo. Ed.Elsevier.
pp. (771,772,776)

3 - Una chica de 17 años que reside en el Congo lleva 2 años con una enfermedad febril crónica. Durante los últimos 2
días observa que su orina es de color pardo claro. La exploración física revela una presión arterial de 145/95 mmHg.
Las pruebas de laboratorio muestran creatinina de 3,7 mg/dl y urea de 35 mg/dl. En el análisis de orina se identifica
sangre 4+ y proteínas 1+, sin presencia de glucosa, cetonas o leucocitos. La cifra de haptoglobina está baja y la prueba
de Coombs es negativa. La concentración de hemoglobina es de 8,5 g/dl. El frotis de sangre periférica identifica escasos
trofozoítos en anillo. La inmunofluorescencia de la biopsia renal revela depósitos granulares de IgG y C3 en las
membranas basales de los capilares glomerulares. El estudio con microscopia electrónica revela depósitos
subepiteliales electrodensos. ¿Cuál de las siguientes enfermedades renales es más probable que tenga?
a) Glomeruloesclerosis focal y segmentaria
B) Nefropatía membranosa
c) Nefropatía IgA
d) Glomerulonefritis aguda
- Kumar, A. and Aster. (2015). 9na.ed. Robbins y Cotran "Patología estructural y funcional". Riñón. Ed.Elsevier. pp.
(909-910)

4 - A una paciente femenina de 63 años, tabaquista, le diagnosticaron por una TAC de abdomen, 3 nódulos en el riñón
derecho de características heterogéneas, con zonas hipodensas compatibles con líquido hemático y necrosis. Se le
realizó una punción biopsia percutánea que informó abundantes hematíes y en el estroma, células atípicas cúbicas y
cilíndricas bajas tapizando ejes fibrovasculares y cuerpos de psamoma, así como también células espumantes.
¿Cuál es su diagnóstico?
a) Carcinoma de células claras
b) Carcinoma papilar renal
c) Carcinoma cromófobo
d) Angiomiolipoma
- Kumar, A. and Aster. (2015). 9na.ed. Robbins y Cotran "Patología estructural y funcional". Riñón. Ed.Elsevier. pp.
(953-954)
5 - Un paciente de 45 años presenta en el glande una lesión solitaria, rojiza, deprimida y ligeramente pruriginosa de
hace 4 meses. El examen del frotis de la lesión y el cultivo microbiológico revelan que la flora cutánea es normal. En el
estudio anatomopatológico de la biopsia se observan “células pavimentosas, de núcleos irregulares e hipercromáticos
con pérdida de la polaridad nuclear y de la maduración superficial e imágenes de mitosis, en todo el espesor. La lesión
estaba limitada por la membrana basal”. ¿Cuál es para Ud. el diagnóstico más probable?

a) Enfermedad de Bowen
b) Condiloma acuminado
c) Carcinoma verrugoso
d) Herpes genital simple
- Kumar, A. and Aster. (2015). 9na.ed. Robbins y Cotran "Patología estructural y funcional". Vías urinarias bajas y
aparato genital masculino. Ed.Elsevier. pp. (971)

6 - Un hombre de 85 años ha presentado dificultad para iniciar la micción y nicturia durante el último año. Además,
ha sufrido una lumbalgia progresiva durante los últimos 6 meses. Al tacto rectal se encuentra una próstata dura e
irregular. El centellograma óseo revela áreas de captación en las vértebras torácicas y lumbares. Las pruebas de
laboratorio muestran una concentración de fosfatasa alcalina sérica de 300 U/l y de antígeno prostático específico
(PSA) de 72 ng/ml. La de urea es de 44 mg/dl y la de creatinina de 3,8 mg/dl. Se obtienen muestras transrectales de
biopsia de todos los lóbulos prostáticos. El estudio histológico de las mismas revela que más del 90% del tejido está
constituido por un patrón de cordones y láminas de células con núcleos pleomorfos hipercromáticos, con llamativos
nucléolos y escaso citoplasma. ¿A qué categoría de la gradación de Gleason se corresponden estas categorías
histológicas?
a) 1
b) 3
c) 5
d) 7
- Kumar, A. and Aster. (2015). 9na.ed. Robbins y Cotran "Patología estructural y funcional". Aparato genital masculino.
Ed.Elsevier. pp. (987)
- TEÓRICO: Genital Masculino (diapositivas 21 y 24)
- CIERRE DE CASO PROBLEMA “Carcinoma de próstata”
7 - Un hombre de 65 años comenzó con debilidad, fatiga, linfadenopatía generalizada, hepatoesplenomegalia, cuadro
de anemia moderada con trombocitopenia y marcada leucocitosis (con un 80% de linfocitos). Se efectúo biopsia
ganglionar constatándose “reemplazo difuso de la arquitectura ganglionar por una población de células linfoides
pequeñas con núcleo redondeado con cromatina condensada y escaso citoplasma.”. El estudio inmunofenotípico de
las células tumorales reveló que son ciclina D1 (-), PanB, CD5 y CD23 (+). ¿Cuál es para Ud. el diagnóstico más probable?

a) Linfoma de células del manto


b) Leucemia linfocítica crónica
c) Linfoma folicular
d) Linfoma inmunoblástico
- Kumar, A. and Aster. (2015). 9na.ed. Robbins y Cotran "Patología estructural y funcional". Enfermedades de los
leucocitos, ganglios linfáticos, bazo y timo. Ed.Elsevier. pp. (594)

8 - Un niño de 14 años refiere molestias en el tórax que han empeorado en los últimos 5 días. En la exploración física
presenta adenopatías generalizadas. La radiografía de tórax muestra campos pulmonares limpios, pero el mediastino
está ensanchado. Una TC torácica revela una masa de 10 cm en el mediastino anterior. Se realiza una biopsia de la
masa y al microscopio aparece cubierta por células linfoides con núcleos lobulados que contienen cromatina nuclear
de aspecto frágil con un fino puntillado. Hay escaso citoplasma y se observan muchas mitosis. Las células expresan
antígenos CD2 y CD7 y son negativas para la desoxinucleotidiltransferasa (TdT−). El análisis molecular revela una
mutación puntual del gen NOTCH1. El oncólogo informa a la familia que la quimioterapia resulta curativa en la inmensa
mayoría de los casos. ¿Cuál es el diagnóstico más probable?
a) Linfoma de Burkitt
b) Linfoma folicular
c) Linfoma de células del manto
d) Linfoma linfoblástico
- Kumar, A. and Aster. (2015). 9na.ed. Robbins y Cotran "Patología estructural y funcional". Enfermedades de los
leucocitos, ganglios linfáticos, bazo y timo. Ed.Elsevier. pp. (590-592)

9 - En referencia a los infartos cerebrales, indique cual de las siguientes afirmaciones es INCORRECTA:
a) Los infartos lacunares suelen ser pequeños y múltiples y se relacionan con la HTA
b) Los infartos cerebrales por oclusión se relacionan con ateroèmbolos provenientes de la carótida interna.
c) Los infartos cerebrales se deben siempre a fenómenos oclusivos arteriales.
d) La extensión del infarto dependerá del grado de obstrucción, calibre del vaso, estado y grado de anastomosis de los
vasos arteriales circundantes.
- Kumar, A. and Aster. (2015). 9na.ed. Robbins y Cotran "Patología estructural y funcional". Sistema Nervioso Central.
Ed.Elsevier. pp. (1264-1269)
10 -

La microfotografía presentada corresponde a tejido nervioso de un paciente con una meningoencefalitis vírica. Las
flechas indican una célula de Purkinje con inclusiones citoplasmáticas redondeadas a ovales y eosinófilas. ¿La presencia
de estas inclusiones se considera patognomónica de cuál de las siguientes entidades?
a) Arbovirus.
b) Virus del herpes simple tipo 1.
c) Rabia.
d) Citomegalovirus.
- Kumar, A. and Aster. (2015). 9na.ed. Robbins y Cotran "Patología estructural y funcional". Sistema Nervioso Central.
Ed.Elsevier. pp. (1277)
- TEÓRICO: SNC Patología infecciosa, desmielinizante y degenerativa - diapositiva 28
DEPARTAMENTO DE PATOLOGÍA - PATOLOGIA II MODALIDAD VIRTUAL
SEGUNDO EXAMEN PARCIAL - TURNO VIERNES NOCHE (17 - 21hs.) - 3 de julio del 2020.

1 - Paciente femenina de 23 años, concurre a la consulta luego de presentar cuadro de dolor abdominal de 3 meses
de evolución que se acompañó de diarrea y distensión abdominal. Refiere asociar los episodios de dolor y diarrea a
los alimentos que consume en especial fideos o pan.Ud., sospechando celiaquía, le solicita una VEDA (videoendoscopía
digestiva alta) para toma de biopsia. ¿Qué espera encontrar en ésta?
a) Pérdida de la superficie mucosa y del borde en cepillo con aumento del número de los linfocitos T CD8+
intraepiteliales e hiperplasia de las criptas.
b) Aumento de tamaño de las placas de Peyer en íleon terminal, formando elevaciones bien delimitadas de hasta 8cm
de diámetro, con infiltrado polimorfonuclear dentro de la lámina propia superficial con macrófagos que contienen
bacterias.
c) Pared intestinal engrosada y correosa como consecuencia del edema transmural, con abundantes neutrófilos que
infiltran y dañan las criptas del epitelio.
d) Infiltración leucocitaria intraepitelial tanto linfocítica como polimorfonucleares con atrofia de enterocitos.
- Kumar, A. and Aster. (2015). 9na.ed. Robbins y Cotran "Patología estructural y funcional". Tubo digestivo. Ed.Elsevier.
pp. (782-783)

2 - La Gastritis por H. pylori tiene las siguientes características, EXCEPTO:


a) H. pylori se ve en la capa epitelial de la superficie luminal
b) H. pylori se ve en áreas con metaplasia intestinal
c) H. pylori no invade la mucosa
d) La gastritis por H. pylori puede conducir a una transformación maligna
- Kumar, A. and Aster. (2015). 9na.ed. Robbins y Cotran "Patología estructural y funcional". Tubo digestivo. Ed.Elsevier.
pp. (763-764)

3 - Paciente masculino de 17 años, concurre a la consulta refiriendo problemas en la audición. Se realiza la prueba de
Weber que es positiva. En el interrogatorio, el paciente comenta que su madre tiene una enfermedad hereditaria que
afecta el riñón manifestándose con hematuria. El paciente no refiere el mismo signo, pero al hacerse un sedimento
urinario, informa abundantes hematíes y cilindros hemáticos. Siguiendo su sospecha diagnóstica, ud solicita un
examen de orina de 24hs que informa proteinuria de 2,6g.
¿Cuál sería su sospecha diagnóstica?
a) Síndrome de Alport
b) Lesión de la membrana basal delgada (Hematuria familiar benigna)
c) Nefritis lúpica
d) Enfermedad de Berger
- Kumar, A. and Aster. (2015). 9na.ed. Robbins y Cotran "Patología estructural y funcional". Riñón. Ed.Elsevier. pp.
(924)

4 - Con respecto al Carcinoma de células renales, señale la opción CORRECTA:


a) La mayor parte de los casos son familiares
b) La invasión del seno renal es un indicador de mal pronóstico
c) El gen VHL se ve mutado únicamente en familias que padecen el Síndrome de Von Hippel Lindau.
d) Es la neoplasia renal menos frecuente.
- Kumar, A. and Aster. (2015). 9na.ed. Robbins y Cotran "Patología estructural y funcional". Riñón. Ed.Elsevier. pp.
(953)
5 - Paciente masculino de 61 años, concurre a la consulta por presentar dolor suprapúbico, tenesmo vesical y
polaquiuria con ardor al orinar, de 2 meses de evolución. Refiere dolor lumbar que él asocia con sus problemas de
ciático y le entrega un estudio de laboratorio que informa PSA 9mcg/L. La biopsia prostática evidencia abundantes
glándulas con dilataciones quísticas revestidas por dos capas de epitelio.

¿A qué patología prostática correspondería este cuadro clínico?


a) Adenocarcinoma de próstata bien diferenciado
b) Neoplasia intraepitelial prostática (PIN) de alto grado
c) Prostatitis aguda
d) Hiperplasia prostática benigna
- Kumar, A. and Aster. (2015). 9na.ed. Robbins y Cotran "Patología estructural y funcional". Vías urinarias bajas y
aparato genital masculino. Ed.Elsevier. pp. (983-984)

6 - Paciente de 34 años que presenta dolor testicular y al examen clínico presenta agrandamiento testicular. Se opera,
se examina un testículo agrandado de tamaño, al corte de aspecto y coloración homogénea sin áreas de necrosis o
hemorragias, microscópicamente se caracteriza por presentar las células tumorales con un infiltrado linfocitico, usted
pensaría en:
a) Tumor de células de Leydig
b) Corioncarcinoma
c) Tumor del saco vitelino
d) Seminoma
- Kumar, A. and Aster. (2015). 9na.ed. Robbins y Cotran "Patología estructural y funcional". Vías urinarias bajas y
aparato genital masculino. Ed.Elsevier. pp. (976)

7 - Un hombre de 61 años se presenta con fatiga, esplenomegalia marcada, fiebre y malestar general. En su sangre
periférica se observa una leucocitosis principalmente a expensas de un incremento de los neutrófilos, y 7% de formas
blásticas. Una biopsia de su médula ósea revela una hipercelularidad (98%) constituida en su mayor parte por
precursores mieloides, y megacariocitos hipolobulados. Se le hizo un análisis citogenético que reveló una translocación
que involucra a los cromosomas 22 y 9. Dados los siguientes diagnósticos diferenciales, elija la opción CORRECTA para
establecer un diagnóstico de certeza:
a) Reacción leucemoide.
b) Leucemia mieloide aguda.
c) Leucemia neutrofílica crónica.
d) Leucemia mieloide crónica.
Kumar V, Abbas AK, Aster JC. Robbins y Cotran Patología estructural y funcional. 9na ed. Barcelona: Elsevier-Saunders;
2015. p. 616-618.
8 - Un paciente de 28 años consultó por astenia, febrícula vespertina y pérdida de peso. En el examen físico presentó
adenomegalias cervicales y en el examen radiológico se constataron adenomegalias en mediastino y en ganglios
retroperitoneales. Las biopsias demostraron un compromiso difuso ganglionar por infiltrado celular heterogéneo
constituido por linfocitos, eosinófilos, células plasmáticas y macrófagos, entremezclados con células grandes
binucleadas con nucleolo evidente y abundante citoplasma eosinófilo CD15 Y CD30 (+). ¿Cuál es para usted el
diagnóstico y la estadificación clínica más probable?

a) Linfoma no Hodgkin de células grandes B, estadío IV


b) Enfermedad de Hodgkin, esclerosis nodular, estadío III
c) Enfermedad de Hodgkin, celularidad mixta, estadío III
d) Enfermedad de Hodgkin, celularidad mixta, estadío II
- Kumar, A. and Aster. (2015). 9na.ed. Robbins y Cotran "Patología estructural y funcional". Enfermedades de los
leucocitos, ganglios linfáticos, bazo y timo. Ed.Elsevier. pp. (607-611)

9 - Paciente masculino de 65 años que presenta al examen del fondo de ojo una imagen tumoral vascularizada y
pigmentada, se decide la enucleación, se evidencia una tumoración de 10x12mm que se origina de la coroides y
protruye hacia cámara anterior. Está formado por células fusiformes y tiene marcadores (+) para HMB45 y proteína
S100. La patología más probable seria:
a) Metástasis de cáncer de próstata
b) Liposarcoma
c) Retinoblastoma
d) Melanoma ocular
- Kumar, A. and Aster. (2015). 9na.ed. Robbins y Cotran "Patología estructural y funcional". Ojo. Ed.Elsevier. pp. (1331)

10 - Paciente de 46 años de edad consulta por convulsiones repetidas y cefaleas de 15 días de evolución. Se solicita
RMN con contraste: “masa hipercaptante con focos de microcalcificación ubicada a nivel del lóbulo frontal”. Se realiza
cirugía. Histológicamente se observa una “proliferación de células uniformes de núcleo redondo e hipercromàtico
rodeado de escaso citoplasma con halos perinucleares acompañadas por una fina red de capilares anastomosados
entre sí. Pueden visualizarse focos microscópicos calcificados”. ¿Cuál es para Ud. el diagnóstico más probable?
a) Xantoastrocitoma pleomòrfico (GII/IV)
b) Glioblastoma multiforme (GIV/IV)
c) Oligodendroglioma clásico (GII/IV)
d) Astrocitoma anaplàsico (GIII/IV)
- Kumar, A. and Aster. (2015). 9na.ed. Robbins y Cotran "Patología estructural y funcional". Sistema Nervioso Central.
Ed.Elsevier. pp. (1310).
DEPARTAMENTO DE PATOLOGÍA - PATOLOGIA II MODALIDAD VIRTUAL
SEGUNDO EXAMEN PARCIAL - TURNO VIERNES TARDE (14 - 18hs.) - 3 de julio del 2020.

1 - Paciente femenina de 48 años, consulta por dolor abdominal asociado a acidez postprandial, vómitos según refiere,
hematemesis, de varios años de evolución. Trae a la consulta varios estudios: prueba de la ureasa positiva; biopsia
gástrica que informa atrofia de la mucosa gástrica con pérdida de células parietales y principales con infiltración
linfocítica de la lámina propia y folículos linfoides reactivos. Los marcadores de IHQ CD19, CD20 y CD43 son positivos.
¿Cuál es la patología en cuestión?
a) Gastritis linfocítica
b) Gastritis granulomatosa
c) Linfoma del tejido linfoide asociado a mucosas
d) Gastritis eosinofílica
- Kumar, A. and Aster. (2015). 9na.ed. Robbins y Cotran "Patología estructural y funcional". Tubo digestivo. Ed.Elsevier.
pp. (773)

2 - ¿Cuál de las siguientes características se encuentra asociada a la Enfermedad de Crohn?


a) La distribución de las lesiones es difusa conformando un cuadro de rectocolitis-úlcero-hemorrágica.
b) No se observa la presencia de granulomas y/o fístulas.
c) La complicación más frecuente es el megacolon tóxico.
d) Se constituye como una enteritis regional con frecuente afectación ileal.
- Kumar, A. and Aster. (2015). 9na.ed. Robbins y Cotran "Patología estructural y funcional". Tubo digestivo. Ed.Elsevier.
pp. (798-800)

3 - ¿Cuáles de los siguientes son factores de riesgo para el carcinoma de vejiga?


a) Tabaquismo, sexo masculino, edades entre 50 y 80 años, dieta alta en fibras vegetales, aumento de renina.
b) Tabaquismo, sexo femenino, edades entre 20 y 50 años, aumento de ADH, exposición a arilaminas.
c) Tabaquismo, sexo femenino, radiación, edades entre 20 y 50 años, aumento de ADH, consumo importante de
ciclofosfamida.
d) Tabaquismo, sexo masculino, radiación, edades entre 50 y 80 años, consumo importante de ciclofosfamida.
- Kumar, A. and Aster. (2015). 9na.ed. Robbins y Cotran "Patología estructural y funcional". Vías urinarias bajas y
aparato genital masculino. Ed.Elsevier. pp. (964-965)

4 - A una mujer de 19 años con episodios febriles, náuseas y eventuales vómitos se le realizan exámenes de laboratorio
de rutina destacándose las alteraciones en su orina que constan de marcada hematuria y moderada proteinuria. Para
estudiarla más profundamente se le efectúa una biopsia renal que evidencia numerosos neutrófilos a nivel del
glomérulo. Con el microscopio de epifluorescencia se identifican depósitos de IgG y C3 dispuestos granulares grandes
aislados sobre la membrana basal del capilar; y en la microscopia electrónica, depósitos en el sector subepitelial. ¿Cuál
es el diagnóstico?
a) Síndrome de Alport.
b) Glomerulonefritis postinfecciosa.
c) Síndrome de Goodpasture.
d) Glomerulonefritis membranoproliferativa tipo II.
- Kumar, A. and Aster. (2015). 9na.ed. Robbins y Cotran "Patología estructural y funcional". Riñón. Ed.Elsevier. pp.
(909-911)

5 - ¿Cuál de las siguientes características se asocia con hiperplasia prostática benigna?


a) Muy baja frecuencia, especialmente en edades avanzadas.
b) Cuando presenta síntomas, suelen ser asociados a la obstrucción del flujo urinario.
c) Debido a su mayor ubicación en la zona periférica, suelen ser asintomáticos pero rápidamente palpables por tacto
rectal.
d) En general, el tejido próstático aumenta su tamaño independientemente de la acción de la dihidrotestosterona.
- Kumar, A. and Aster. (2015). 9na.ed. Robbins y Cotran "Patología estructural y funcional". Riñón. Ed.Elsevier. pp.
(982-983)
6 - Con respecto a Patología Testicular, indique la opción INCORRECTA:
a) Los seminomas son los tumores de células germinales más frecuentes.
b) La histología del seminoma consta de células claras dispuestas en lóbulos delimitados por finos tabiques de tejido
conectivo e infiltrado linfocitario.
c) La malignidad de los seminomas radica en la infiltración local y no metastatiza.
d) Los seminomas se originan a partir de una lesión precursora denominada neoplasia intratubular de células
germinales.
- Kumar, A. and Aster. (2015). 9na.ed. Robbins y Cotran "Patología estructural y funcional". Aparato genital masculino.
Ed.Elsevier. pp. (976)

7 - Una paciente de 44 años, presenta adenopatías indoloras, leve hepato y esplenomegalia. Se le extirpa una
adenopatía cervical derecha, que mide 3cm de diámetro. En la superficie de corte se observan nódulos blanquecinos.
Microscópicamente se observaron “conglomerados nodulares de linfocitos atípicos pequeños (centrocitos) y otros más
grandes (centroblastos)”. IHQ: CD10 y la proteína BCL2 (+). ¿Cuál es para Ud. el diagnóstico más probable?

a) Linfoma folicular
b) Linfoma Hodgkin
c) Linfoma difuso de células grandes B
d) Linfoma de células T del adulto
- Kumar, A. and Aster. (2015). 9na.ed. Robbins y Cotran "Patología estructural y funcional". Enfermedades de los
leucocitos, ganglios linfáticos, bazo y timo. Ed.Elsevier. pp. (594-596)

8 - Llega a la consulta una persona con su hijo de 3 años. Se lo nota preocupado al padre porque refiere que su hijo
hace un mes que no quiere jugar, que lo nota cansado, lo ve algo más pálido. En la anamnesis se resalta que está con
sangrados leves-moderados, y en el último año tuvo 5 veces otitis, de causa infecciosa. Se sospecha de una patología
hemolinfoide, ¿de cuál probablemente se trate?
a) Leucemia mieloide aguda.
b) Leucemia mieloide crónica.
c) Síndrome mielodisplásicos.
d) Leucemia/linfoma linfoblásticos agudos.
- Kumar, A. and Aster. (2015). 9na.ed. Robbins y Cotran "Patología estructural y funcional". Enfermedades de los
leucocitos, ganglios linfáticos, bazo y timo. Ed.Elsevier. pp. (590-593)
9 - Con respecto a los Tumores del SNC, indique la opción INCORRECTA:
a) Los glioblastomas multiformes se presentan como masas infiltrantes hemorrágicos y necróticos.
b) Los glioblastomas multiformes presentan histología similar a los astrocitomas anaplásicos y se les agrega necrosis y
proliferación vascular o de células endoteliales.
c) En el glioblastoma las células tumorales se agrupan alrededor de áreas necróticas dando imágenes en empalizada.
d) El astrocitoma pilocítico suele presentarse en ancianos.
- Kumar, A. and Aster. (2015). 9na.ed. Robbins y Cotran "Patología estructural y funcional". Sistema Nervioso Central.
Ed.Elsevier. pp. (1308)

10 - Paciente de 54 años consulta por cefalea. Se le realiza una TAC de cerebro y se observa una “lesión nodular,
parasagital cerca de la hoz del cerebro”. ¿Cuál de las siguientes descripciones histológicas Ud. esperaría encontrar en
la microscopia?

a) Células pequeñas indiferenciadas, hipercromasia y abundantes mitosis.


b) Células ahusadas con núcleos ovoides claros, regulares, dispuestas en verticilos o remolinos, con formación de
cuerpos de psammoma.
c) Células atípicas que se disponen formando estructuras glandulares, algunas con vacuolas citoplasmáticas.
d) Células escamosas atípicas con ocasionales perlas corneas.
- Kumar, A. and Aster. (2015). 9na.ed. Robbins y Cotran "Patología estructural y funcional". Sistema Nervioso Central.
Ed.Elsevier. pp. (1314-1315). Fig. 28-53
DEPARTAMENTO DE PATOLOGÍA - PATOLOGIA II MODALIDAD VIRTUAL
PRIMER EXAMEN PARCIAL - TURNO JUEVES NOCHE (17-21hs.) - 21 de mayo del 2020.

1 - Un paciente masculino de 62 años evidencia en una zona de piel expuesta al sol, una lesión (menor a 1cm de
diámetro) que al tacto es áspera como papel de lija y presenta a la microscopia “estrato corneo con paraqueratosis,
células basales atípicas y colección de fibras con degeneración basofílica (elastosis) en la dermis superior”. En base a
estos datos, usted…
a. Considera que se trata de una dermatitis herpetiforme.
b. Le indica extirpación por pensar que puede ser un carcinoma basocelular.
c. Le prescribe una pomada con corticoide considerándolo dermatítis de contacto y lo cita en 3 meses.
d. Lo interpreta como un caso de queratosis actínica.
- Kumar, A. and Aster. (2015). 9na.ed. Robbins y Cotran "Patología estructural y funcional". Piel. Ed.Elsevier. pp.(1154)
y figura 25-12 B.

2 - Paciente de 12 años, sin antecedentes traumáticos, comienza con malestar general y fiebre. Además, refiere dolor
y tumefacción a nivel del tercio medio del fémur derecho. Una Rx revela en la diáfisis femoral, la presencia de un área
focal lítica de bordes difusos con reacción perióstica tipo “capas de cebolla”. La biopsia de la lesión reveló estar
constituída por células redondas, azules y pequeñas, que en sectores se disponen formando Rosetas de Homer-Wright.
Usted considera que se trata de……
a. Osteomielitis hematógena.
b. Osteosarcoma.
c. Sarcoma de Ewing/PNET.
d. Quiste óseo aneurismático.
- Kumar, A. and Aster. (2015). 9na.ed. Robbins y Cotran "Patología estructural y funcional". Huesos, articulaciones y
tumores de partes blandas. TFSE. Ed.Elsevier. pp. (1203)

3 - La evolución progresiva de la necrosis por coagulación de los miocitos, la aparición brusca de un infiltrado
polimorfonuclear constituído por neutrófilos y la pérdida de núcleos y estriaciones de los miocitos son cambios
morfológicos que pueden observarse durante la evolución de un infarto de miocardio. ¿A qué período de tiempo
corresponden los cambios mencionados?
a. 12 a 72 horas.
b. 3 a 7 días.
c. 1 hora.
d. 1 a 2 semanas.
- Kumar, A. and Aster. (2015). 9na.ed. Robbins y Cotran "Patología estructural y funcional". Corazón. Cardiopatía
Isquémica. Ed.Elsevier. pp.(544)
4 - Paciente femenina de 68 años de edad con disnea, antecedentes de “Cáncer de mama”. Examen físico: ruidos
cardíacos alejados, TA 90/60, FC 122/min. Se realiza ECG, Rx (IC-Índice cardiotorácico: aumentado con silueta
cardíaca en forma de "botellón") y angiotac que confirman el diagnóstico. Se realiza pericardiocentesis. ¿Cuál es el tipo
de derrame pericárdico más frecuente en esta entidad nosológica?
a. Seroso
b. Fibrinoso
c. Purulento
d. Hemorrágico
- Kumar, A. and Aster. (2015). 9na.ed. Robbins y Cotran "Patología estructural y funcional". Corazón. Enfermedad
Pericárdica. Ed.Elsevier. pp. (574)

5 - ¿Cuál es la causa más frecuente del enfisema?


a. Tabaquismo.
b. Déficit de α1-antitripsina.
c. Patologías pleurales.
d. Cicatrización.
Kumar, A. and Aster. (2015). 9na.ed. Robbins y Cotran "Patología estructural y funcional". Pulmón. Enfermedades
Pulmonares obstructivas. Ed.Elsevier. pp.(675-676)

6 - En relación con los distintos tipos de carcinoma de pulmón cuál es su inmunofenotipo expresado generalmente.
Marque la opción CORRECTA.
a. El adenocarcinoma expresa CK7, p63, sinaptofisina y TTF1.
b. El carcinoma escamoso expresa p40, p63, cromogranina y TTF1.
c. El carcinoma de células de pequeñas expresa sinaptofisina, cromogranina, CD57 y TTF1.
d. El adenocarcinoma expresa CK20, Napsin A, TTF1 y p40.
- Kumar, A. and Aster. (2015). 9na.ed. Robbins y Cotran "Patología estructural y funcional". Pulmón. Tumores.
Ed.Elsevier. pp.(717)
- Teórico “Cáncer de Pulmón”. Diapositiva 71

7 - El feocromocitoma se asocia con todos los siguientes síndromes, EXCEPTO:


a. Síndrome paraganglioma-feocromocitoma hereditario.
b. Neoplasia endocrina múltiple 1 (MEN 1).
c. Neoplasia endocrina múltiple 2a (MEN 2a).
d. Neoplasia endocrina múltiple 2b (MEN 2b).
- Kumar, A. and Aster. (2015). 9na.ed. Robbins y Cotran "Patología estructural y funcional". Sistema Endócrino.
Neoplasias endócrinas múltiples. Ed.Elsevier. pp.(1136-1137)

8 - Con respecto al tumor mucinoso de ovario, señale la opción CORRECTA:


a. La mayor parte de estos tumores son benignos o borderline.
b. La presencia de un pseudomixoma indica benignidad.
c. La ausencia de áreas sólidas indica malignidad.
d. Constituye el 90% de los tumores ováricos.
- Kumar, A. and Aster. (2015). 9na.ed. Robbins y Cotran "Patología estructural y funcional". Aparato Genital Femenino.
Tumores de Ovario. Ed.Elsevier. pp.(1026)

9 - Paciente de sexo femenino de 65 años de edad, lo consulta por el hallazgo mediante autoexamen, de una masa
palpable con protrusiones bulbosas en el CSE (cuadrante superoexterno) de la mama derecha. Refiere que no tiene
realizados estudios mamográficos recientes. Luego de la evaluación clínica, Ud. le indica estudio mamográfico y una
PAAF (punción aspiración con aguja fina) de dicha lesión. La paciente vuelve a la consulta con el siguiente informe
anatomo-patológico: los cortes histológicos muestran “nódulos de sobrecrecimiento estromal intralobulillar y altamente
proliferativo, con gran número de mitosis y polimorfismo nuclear. Asimismo, dicha proliferación estromal, rodea, empuja
y distorsiona el epitelio glandular asociado”. En base a los datos clínico-epidemiológicos y a dicho informe, Ud. pensaría
en:
a. Fibroadenoma.
b. Tumor Phyllodes.
c. Carcinoma lobulillar in situ.
d. Carcinoma invasor NST.
Kumar, A. and Aster. (2015). 9na.ed. Robbins y Cotran "Patología estructural y funcional". Mama. Tumores del estroma.
Ed.Elsevier. pp.(1069)
10 - Se encuentra ante una paciente de 42 años con antecedente de haber realizado el ultimo control ginecológico, y
le trae el resultado de una biopsia de cuello uterino que informa: imagen compatible con SIL de alto grado (HSIL, CIN
III), en la microscopia esperaría encontrar:
a. Epitelio cervical intacto, sin alteraciones histoarquitecturales.
b. Displasia leve epitelial que abarca el 1/3 basal del epitelio.
c. Presencia de coilocitos, hemorragia por debajo de la membrana basal y microinvasion.
d. Alteración de todo el espesor epitelial, perdida de la maduración celular, atipia celular, anisocitosis, mitosis.
- Kumar, A. and Aster. (2015). 9na.ed. Robbins y Cotran "Patología estructural y funcional". Aparato genital femenino.
Carcinoma Cervical. Ed.Elsevier. pp.(1004) y figura 22-14
DEPARTAMENTO DE PATOLOGÍA - PATOLOGIA II MODALIDAD VIRTUAL
PRIMER EXAMEN PARCIAL - TURNO JUEVES TARDE (14-18hs.) -21 de Mayo del 2020.

1 - Una mujer de 32 años, nadadora de aguas abiertas desde su niñez, que presenta en piel de dorso nasal una
pápula perlada con telangiectasias, ligeramente pigmentada. Se decide su extirpación y en el estudio histológico le
informan “lóbulos localizados en dermis constituidos por células basófilas, de núcleos hipercromáticos con disposición
en empalizada de las células de la periferia”. El diagnóstico histológico descripto corresponde a:

a. Nevo melanocítico intradérmico.


b. Carcinoma basocelular.
c. Carcinoma epidermoide.
d. Melanoma nodular
- Kumar, A. and Aster. (2015). 9na.ed. Robbins y Cotran "Patología estructural y funcional". Piel. Ed.Elsevier.
pp.(1157).
- Figura.Teórico “Piel”. Diapositiva 65

2 - El tumor primario mesenquimático maligno más frecuente del hueso, se caracteriza por (Marque la opción
INCORRECTA).
a. Presenta una distribución bimodal con relación a la edad de los pacientes.
b. No producir matriz osteoide ya que sus células producen matriz condroide exclusivamente.
c. Localizarse con más frecuencia en la metáfisis de fémur distal y la tibia proximal.
d. Presentar como signo radiológico característico, aunque no patognomónico el triángulo de Codman.
- Kumar, A. and Aster. (2015). 9na.ed. Robbins y Cotran "Patología estructural y funcional". Huesos, articulaciones y
partes blandas. Osteosarcoma. Ed.Elsevier. pp.(1198-1200)
- Figura 26-24 (pp.1199)
3 - Con relación a los tumores cardiacos marce la opción CORRECTA.
a. El mixoma es el tumor primario maligno más frecuente.
b. El rabdomioma es el tumor secundario benigno más frecuente en pacientes pediátricos.
c. El angiosarcoma es el tumor secundario maligno más frecuente.
d. Ninguno de los enunciados anteriores es correcto.
- Kumar, A. and Aster. (2015). 9na.ed. Robbins y Cotran "Patología estructural y funcional". Corazón. Tumores
cardíacos. Ed.Elsevier. pp.(575-576)

4 - Ingresa por guardia un paciente masculino de 78 años que presentaba hace 6hs. dolor precordial, tiene laboratorio
con enzimas cardiacas aumentadas. ECG con afectación del segmento ST, el paciente fallece a las 2hs. de haber
ingresado. En el examen microscópico se encuentra:
a. Signos de miocitolisis y fibrosis
b. No se observan alteraciones microscópicas
c. Edema celular, comienzo de la necrosis por coagulación
d. Tejido de granulación, nódulos de Aschoff.
- Kumar, A. and Aster. (2015). 9na.ed. Robbins y Cotran "Patología estructural y funcional". Corazón. Cardiopatía
Isquémica. Ed.Elsevier. pp.(544)

5 - En referencia a la entidad nosológica “Derrame pleural”, indique la opción CORRECTA


a. En el derrame pleural, el exudado purulento se inicia desde la vía hematógena, a esta forma única se la denomina
Empiema.
b. En el derrame pleural, las colecciones serosas no inflamatorias están asociadas a derrame neoplásico.
c. En el derrame pleural la presencia de sangre franca se asocia a contacto con caverna tuberculosa.
d. En el derrame pleural, el Lupus y la Fiebre Reumática son causas de pleuritis serosas y serofibrinosas.
- Kumar, A. and Aster. (2015). 9na.ed. Robbins y Cotran "Patología estructural y funcional". Pulmón. Derrame pleural.
Ed.Elsevier. pp.(722)

6 - Con respecto al enfisema pulmonar marque la opción CORRECTA:


a. El enfisema panacinar es el tipo de enfisema más frecuente y se observa en los lóbulos superiores del pulmón.
b. El enfisema centroacinar es el de mayor frecuencia.
c. La pérdida del tejido elástico lleva al colapso de las vías respiratorias pequeñas durante la inspiración.
d. No produce espasmo de los vasos pulmonares.
- Kumar, A. and Aster. (2015). 9na.ed. Robbins y Cotran "Patología estructural y funcional". Pulmón. Enfisema.
Ed.Elsevier. pp.(675)

7 - Paciente de sexo femenino de 36 años de edad que presenta un nódulo dominante en lóbulo tiroideo derecho. Se
realiza una ecografía y se informa nódulo hipoecoico, sólido-quístico, de bordes difusos, con microcalcificaciones y
aumento del flujo sanguíneo intranodular, que mide 45mm de diámetro. Los extendidos citológicos obtenidos
mediante punción-aspiración con aguja fina del nódulo muestran células con núcleos de aspecto ópticamente vacío,
con hendiduras nucleares, surcos intranucleares (grooves) y seudoinclusiones nucleares.¿Cuál de los siguientes
considera Usted es el diagnóstico más probable?
a. Adenoma folicular de tiroides.
b. Carcinoma folicular de tiroides.
c. Carcinoma medular de tiroides.
d. Carcinoma papilar de tiroides.
- Kumar, A. and Aster. (2015). 9na.ed. Robbins y Cotran "Patología estructural y funcional". Sistema Endócrino.
Carcinoma Papilar. Ed.Elsevier. pp.(1096)

8 - Una persona de sexo femenino de 24 años consulta por “sentir algo duro” en la mama derecha. Durante la
anamnesis la paciente refiere que no le duele, y no se rescatan antecedentes relevantes. Al examen físico se detecta
un nódulo único, desplazable, elástico, de aproximadamente 2cm. ¿Cuál es la etiología más probable?
a. Carcinoma invasor.
b. Carcinoma in situ.
c. Fibroadenoma.
d. Tumor Phyllodes.
Kumar, A. and Aster. (2015). 9na.ed. Robbins y Cotran "Patología estructural y funcional". Mama. Fibroadenoma.
Ed.Elsevier. pp.(1069)
9 - En relación con el carcinoma de endometrio, indique la opción INCORRECTA:
a. El tipo 1 constituye la variante histológica más frecuentemente observada.
b. El tipo endometrioide es característico que aparezca sobre una hiperplasia endometrial y se asocie a obesidad,
DBT, HTA, infertilidad y estimulación estrogénica prolongada.
c. El tipo 2 suele afectar a pacientes en edad fértil, y en relación a úteros con sectores de atrofia endometrial.
d. El carcinoma intraepitelial endometrial seroso es una potencial lesión precursora del carcinoma tipo 2.
- Kumar, A. and Aster. (2015). 9na.ed. Robbins y Cotran "Patología estructural y funcional". Aparato Genital
Femenino. Carcinoma de Endometrio. Ed.Elsevier. pp.(1014)

10 - Usted es patòlogo y recibe una pieza quirúrgica producto de una ooforectomía derecha de una paciente de
60años de edad. Si su observación fuera la siguiente, “Macroscópicamente el ovario se halla reemplazado por una
formación tumoral en la superficie externa, múltiples formaciones papilares pequeñas, friables, que se desprenden
con facilidad. En su interior, el tumor tiene aspecto multilocular, con áreas sólidas, necróticas y hemorrágicas.
Histológicamente se observan estructuras papilares, con delicados ejes conectivo vasculares tapizadas por 4 o más
capas de células epiteliales atípicas con sectores de invasión estromal”. ¿Cuál es para Ud. el diagnóstico más
probable?
a. Cistoadenoma seroso papilar
b. Tumor seroso papilar boderline
c. Cistoadenocarcinoma seroso papilar
d. Cistoadenofibroma
- Kumar, A. and Aster. (2015). 9na.ed. Robbins y Cotran "Patología estructural y funcional". Aparato Genital
Femenino. Tumores de Ovario. Ed.Elsevier. pp.(1025)
- Teórico “Ovario”. Diapositiva 34-36
DEPARTAMENTO DE PATOLOGÍA - PATOLOGIA II MODALIDAD VIRTUAL
PRIMER EXAMEN PARCIAL - TURNO VIERNES NOCHE (17-21hs.) - 21 de mayo del 2020.

1 - Paciente de sexo femenino de 76 años de edad que presenta en región malar izquierda, en continuidad con la
comisura labial, una lesión cutánea nodular, de bordes poco definidos e irregulares, rojiza, rugosa, descamativa, con
ulceración central, de 8 meses de evolución. Se realiza una biopsia con sacabocado del borde de la lesión. Al
microscopio, se observan células neoplásicas que se extienden desde la epidermis hacia la dermis, poligonales, con
abundante citoplasma eosinófilo. Las mismas se organizan formando nidos con extensas zonas de queratinización
(perlas córneas). A mayor aumento se aprecia la presencia de puentes intercelulares entre células tumorales vecinas.
Esta lesión puede originarse a partir de una lesión preneoplásica. ¿Cómo se denomina a dicha lesión preneoplásica
cuando asienta sobre mucosa labial?
a. Queratosis actínica.
b. Queratoacantoma.
c. Queilitis actínica.
d. Queilitis angular.
- Kumar, A. and Aster. (2015). 9na.ed. Robbins y Cotran "Patología estructural y funcional". Piel. Tumores epidérmicos
premalignos y malignos. Ed.Elsevier. pp.(1154)

2 - En referencia a los Tumores del tejido adiposo. Señale la opción INCORRECTA:


a. El liposarcoma pleomórfico no suele metastatizar.
b. El tumor más frecuente de partes blandas se halla constituído por tejido adiposo madura sin atipia citológica.
c. El tamaño y localización tienen importancia en el pronóstico.
d. El margen quirúrgico adecuado debe ser amplio (de 2 a 3cm)
- Kumar, A. and Aster. (2015). 9na.ed. Robbins y Cotran "Patología estructural y funcional". Huesos, articulaciones y
tumores de partes blandas. Ed.Elsevier. pp.(1220)

3 - Una paciente de 76 años con polimialgia reumática es derivada de Oftalmología por pérdida súbita de la visión hace
5 días, que resultó en ceguera de ambos ojos. Al interrogatorio, la paciente refiere episodios de cefaleas, dificultad para
peinarse por dolor al roce y claudicación mandibular. Se palpan nódulos en la arteria temporal derecha izquierda. ¿Qué
patología sufre esta paciente y qué se encontraría en la biopsia?
a. Arteritis de Takayasu / Inflamación granulomatosa y necrosis de la media
b. Arteritis de células gigantes / Inflamación granulomatosa en la lámina elástica interna
c. Arteritis de Kawasaki / Infiltrado transmural denso
d. Panarteritis nodosa / Infiltrado transmural con necrosis fibrinoide
- Kumar, A. and Aster. (2015). 9na.ed. Robbins y Cotran "Patología estructural y funcional". Vasos Sanguíneos.
Vasculitis. Ed.Elsevier. pp. (508). Figura 11-24
4 - Ingresa por guardia un paciente masculino de 78 años que presentaba hace 6hs. dolor precordial, tiene laboratorio
con enzimas cardiacas aumentadas. ECG con afectación del segmento ST, el paciente fallece a las 2hs. de haber
ingresado. En el examen microscópico se encuentra:
a. Signos de miocitolisis y fibrosis
b. No se observan alteraciones microscópicas
c. Edema celular, comienzo de la necrosis por coagulación
d. Tejido de granulación, nódulos de Aschoff.
- Kumar, A. and Aster. (2015). 9na.ed. Robbins y Cotran "Patología estructural y funcional". Corazón. Cardiopatía
Isquémica. Ed.Elsevier. pp.(544)

5 - Un paciente obeso de 63 años de edad, ex tabaquista severo, con tos y producción de esputo la mayor parte de los
días en los últimos cuatro años, hipertenso y con historia de episodios cardiovasculares, es ingresado a la guardia de
un sanatorio por una descompensación súbita y fallece luego de 2hs. de recibir una atención médica adecuada. Sus
familiares solicitan una autopsia médica que confirma un infarto masivo del miocardio. Entre los hallazgos más
relevantes consignados en el informe de la autopsia los patólogos observaron que los macrófagos alveolares estaban
cargados con pigmento antracótico, y que en la tráquea y los bronquios el epitelio respiratorio tenía un incremento de
células caliciformes y disminución de ciliadas, con áreas de metaplasia escamosa, hipertrofia de glándulas
submucosas, infiltrado inflamatorio predominantemente linfocitario y fibrosis; además muchos bronquíolos estaban
tapados por material mucoso. ¿Cuál de las siguientes entidades corresponde a la descripción realizada?
a. Bronquitis crónica.
b. Enfisema.
c. Carcinoma escamoso.
d. Fibrosis pulmonar idiopática.
- Kumar, A. and Aster. (2015). 9na.ed. Robbins y Cotran "Patología estructural y funcional". Pulmón. Bronquitis Crónica.
Ed.Elsevier. pp.(678-679)

6 - Un hombre de 40 años, fumador de 2 atados de cigarrillos diario, consulta por tos, hemoptisis y adelgazamiento. En
la fibrobroncoscopía se observa una lesión peribronquial extensa que se biopsia. En el informe histopatológico informa
“células redondas, azules, con escaso citoplasma, límites celulares mal definidos, cromatina nuclear finamente
granulada y nucléolos ausentes. Alto índice mitótico y focos de necrosis. Las paredes vasculares presentan un llamativo
refuerzo basófilo”. IHQ: Cromogranina, sinaptofisina, CD57 y BCL2 +. ¿Cuál es para Ud. el diagnóstico más probable?
a. Carcinoma de células pequeñas
b. Tumorlet
c. Carcinoma de células grandes
d. Adenocarcinoma
- Kumar, A. and Aster. (2015). 9na.ed. Robbins y Cotran "Patología estructural y funcional". Pulmón. Tumores de
Pulmón. Ed.Elsevier. pp.(717)
- Figura 15-43 Variantes histológicas del Cáncer de pulmón. pp (715)
- Teórico “Cáncer de Pulmón”. Diapositivas 71 a 76.
7 - ¿Qué marcador con técnica de inmunohistoquímica caracteriza a las células neoplásicas del carcinoma medular de
tiroides?
a. Melan A.
b. Calcitonina.
c. Tiroglobulina.
d. Mieloperoxidasa.
- Kumar, A. and Aster. (2015). 9na.ed. Robbins y Cotran "Patología estructural y funcional". Sistema Endócrino.
Glándula Tioirdes. Ed.Elsevier. pp.(1098-1099)

8 - ¿Cuál de los siguientes factores es el predictor más importante de mal pronóstico en mujeres con carcinoma de la
mama?
a. Estado del receptor de estrógeno.
b. Grado histológico.
c. Metástasis en el ganglio linfático regional.
d. Tamaño del tumor.
- Kumar, A. and Aster. (2015). 9na.ed. Robbins y Cotran "Patología estructural y funcional". Mama. Factores pronósticos
y predictivos. Ed.Elsevier. pp.(1066)

9 - Con respecto a la Patología Placentaria, indique la opción que considera INCORRECTA:


a. El acretismo placentario ocurre en 1 de cada 10.000 embarazos.
b. El riesgo de placenta accreta aumenta en función del número de cesáreas previas.
c. El tumor del sitio placentario se origina en células del trofoblasto intermedio.
d. El embarazo ectópico es responsable del 4 al 10% de de los fallecimientos asociados a la gestación.
- Kumar, A. and Aster. (2015). 9na.ed. Robbins y Cotran "Patología estructural y funcional". Aparato genital femenino.
Ed.Elsevier. pp.(1034-1041)

10 - Paciente mujer de 35 años que consulta por masa pélvica, hidrotórax derecho y ascitis. Al examen clínico se
constata masa anexial en ovario derecho. Se realiza ecografía transvaginal que describe masa ovárica de 6,5cm de
diámetro mayor. Se realiza excéresis cuya Macroscopía informa masa sólida, consistencia dura, coloración grisácea,
recubierta por serosa lisa y brillante. A la microscopía se observa proliferación de fibroblastos sin actividad mitótica ni
alteración de la relación núcleo-citoplasmática. La prueba histoquímica para demostración de grasa fue negativa ¿en
qué patología piensa?
a. Fibroma
b. Hiperplasia folicular
c. Cistoadenoma seroso
d. Teratoma maduro
- Kumar, A. and Aster. (2015). 9na.ed. Robbins y Cotran "Patología estructural y funcional". Aparato Genital Femenino.
Ovarios. Ed.Elsevier. pp.(1033)
DEPARTAMENTO DE PATOLOGÍA - PATOLOGIA II MODALIDAD VIRTUAL
PRIMER EXAMEN PARCIAL - TURNO VIERNES TARDE (14 - 18hs.) - 22 de mayo del 2020.

1 - En relación al melanoma marque la opción CORRECTA


a. Es un tumor benigno y por lo tanto tiene capacidad de hacer metástasis.
b. Son tumores malignos con diferenciación melanocítica y que carecen de la mutación activadora del BRAF.
c. Son tumores de potencial maligno indeterminado que pueden recidivar pero no hacen metástasis.
d. Son tumores malignos con diferenciación melanocitica y que presenta la mutación activadora del BRAF.
- Kumar, A. and Aster. (2015). 9na.ed. Robbins y Cotran "Patología estructural y funcional". Piel. Melanoma. Ed.Elsevier.
pp. (1148)

2 - ¿Cuál de los siguientes enunciados es CORRECTO para el Osteosarcoma?


a. El tipo más frecuente asienta en diáfisis de huesos largos, es primario, intracortical, osteoblástico, y de bajo grado.
b. El tipo más frecuente asienta en metáfisis de huesos largos, es primario, intramedular, osteoblástico y de alto
grado.
c. El tipo más frecuente asienta en diáfisis de huesos largos, es secundario, intramedular, osteoblástico y de alto grado.
d. El tipo más frecuente asienta en metáfisis de huesos largos, es secundario, intracortical, osteoblástico y de bajo grado.
- Kumar, A. and Aster. (2015). 9na.ed. Robbins y Cotran "Patología estructural y funcional". Huesos, articulaciones y
tumores de partes blandas. Ed.Elsevier. pp. (1199)
- Cierre de caso problema “Osteosarcoma”. Diapositiva 10

3 - ¿Qué inmunohistoquímica solicitaría ante la sospecha de un Carcinoma Epidermoide de pulmón?

a. TTF-1, napsina A
b. Calretinina, WT1
c. p63, p40
d. Sinaptofisina, cromogranina
- Kumar, A. and Aster. (2015). 9na.ed. Robbins y Cotran "Patología estructural y funcional". Pulmón. Ed.Elsevier.
pp.(717)
- Figura en diapositiva 30 del teórico “Cáncer de Pulmón”.

4 - Un varón de 20 años, consulta por presentar desde hace 48hs fiebre (39°C). Como antecedentes es adicto a cocaína
por vía intravenosa y refiere haber cursado un síndrome gripal hace una semana. Se decide internación. En la Rx. tórax
se observa una imagen de consolidación pulmonar con broncograma aéreo. Se efectúa test para HIV siendo el resultado
negativo. Presenta elevación de reactantes de fase aguda. ¿Cuál es su diagnóstico?
a. Neumonía intersticial usual
b. Neumonía aguda por staphylococcus aureus
c. Daño alveolar difuso
d. Neumonía por pneumocystiscarinii
- Kumar, A. and Aster. (2015). 9na.ed. Robbins y Cotran "Patología estructural y funcional". Pulmón. Ed.Elsevier.
pp.(703)
5 - Las siguientes son características de “Cardiopatía Reumática Aguda”, EXCEPTO:
a. Pericarditis fibrinosa inespecífica
b. Vegetaciones pequeñas en valvas de la válvula mitral
c. Pancarditis con presencia de nódulos de Aschoff
d. Valvulopatía fibrótica deformante “en boca de pescado”
- Kumar, A. and Aster. (2015). 9na.ed. Robbins y Cotran "Patología estructural y funcional". Corazón. Ed.Elsevier.
pp.(558)

6 - ¿Cuál de las siguientes asociaciones entre tipo de Pericarditis y etiología es CORRECTA?


a. Pericarditis serosa / enfermedades autoinmunitarias e infecciones virales.
b. Pericarditis purulenta / radiación, neoplasias y fiebre reumática.
c. Pericarditis hemorrágica / infarto agudo de miocardio, síndrome de Dressler, radiación.
d. Pericarditis fibrinosa y serofibrinosa / tuberculosis.
- Kumar, A. and Aster. (2015). 9na.ed. Robbins y Cotran "Patología estructural y funcional". Corazón. Pericarditis.
Ed.Elsevier. pp. (573-574)

7 - El examen histológico de una pieza de mastectomía remitida por motivos oncológicos informó la presencia de un
tumor constituido por láminas sólidas de grandes células neoplásicas con pleomorfismo nuclear y nucléolos prominentes,
con numerosas figuras mitóticas, e infiltrado linfocitario intratumoral y peritumoral. El patólogo concluye que se trata de
un carcinoma con rasgos medulares. Respecto al mismo, señale la respuesta INCORRECTA:
a. Su consistencia es más bien blanda, con escasa desmoplasia.
b. Es común que se presente como una masa relativamente bien delimitada.
c. Es un tipo de carcinoma invasor con buen pronóstico.
d. Suele expresar Her2 y ser negativo para receptores hormonales.
- Kumar, A. and Aster. (2015). 9na.ed. Robbins y Cotran "Patología estructural y funcional". Mama. Tipos de carcinoma
de mama. Ed.Elsevier. pp. (1065-1066)

8 - Mujer de 30 años con antecedentes de radioterapia en cabeza y cuello, concurre a realizar control ecográfico tiroideo
periódico, durante el cual se detecta un nódulo de 1,5cm de diámetro. Se realiza PAAF; observándose en el extendido
citológico planchas de “células con núcleos ovoides, solapados, algunos de los cuales presentan surcos o hendiduras, y
otros pseudoinclusiones o vacuolas intranucleares”. ¿Cuál es para Ud. el diagnóstico más probable?

a. Carcinoma anaplásico
b. Carcinoma medular
c. Hiperplasia nodular
d. Carcinoma papilar
- Kumar, A. and Aster. (2015). 9na.ed. Robbins y Cotran "Patología estructural y funcional". Sistema Endócrino. Glándula
Tiroides. Ed.Elsevier. (pp.1095-1097)
- Diapositivas 39 a 43 - Teórico "Tiroides"
- Cierre de caso problema "Carcinoma Papilar de Tiroides"
9 - En Referencia al Carcinoma de Cuello uterino y sus lesiones precursoras, señale la opción CORRECTA.
a. El 80% de los casos de L-SIL aparecen de novo, sólo un 20% se desarrolla a partir de L-SIL previas.
b. El 80% de L-SIL y el 100% de H-SIL se asocian a los VPH de alto riesgo (VPH-16 y VPH-18).
c. La progresión desde H-SIL a carcinoma de cuello uterino invasor, de producirse, tarda pocos meses en la mayoría de
los casos.
d. El 80% de los casos de carcinoma de cuello uterino invasor corresponde al subtipo histológico de adenocarcinoma.
- Kumar, A. and Aster. (2015). 9na.ed. Robbins y Cotran "Patología estructural y funcional". Aparato Genital Femenino.
Cuello Uterino. Ed.Elsevier. pp. (1004)
- CIERRE Carcinoma in situ de cuello uterino: diapositivas 10 y 12

10 - ¿De dónde derivan los tumores serosos de ovario?


a. Epitelio celómico.
b. Cordones sexuales.
c. Células germinales.
d. Metástasis de cánceres primarios extraováricos.
- Kumar, A. and Aster. (2015). 9na.ed. Robbins y Cotran "Patología estructural y funcional". Aparato Genital Femenino.
Ovario. Ed.Elsevier. pp. (1023)
DEPARTAMENTO DE PATOLOGÍA - PATOLOGIA II MODALIDAD VIRTUAL
1° RECUPERATORIO DEL PRIMER EXAMEN PARCIAL - 28 de Mayo del 2020.

1 - Respecto a las cardiopatías congénitas, señale la opción INCORRECTA:


a) La comunicación interventricular es la cardiopatía congénita más frecuentemente observada.
b) La tetralogía de Fallot consiste en una comunicación interventricular, con la aorta situada sobre el tabique
interventricular, hipertrofia del ventrículo izquierdo y estenosis de la arteria pulmonar.
c) Las malformaciones que causan una derivación del flujo sanguíneo desde el lado derecho al lado izquierdo del
corazón se caracterizan por hipoxia y consecuente cianosis que se expresa clínicamente en forma temprana.
d) La infección congénita por rubéola, la diabetes gestacional y la deficiencia de ácido fólico son factores ambientales
implicados en la aparición de cardiopatías congénitas.
- Kumar, A. and Aster. (2015). 9na.ed. Robbins y Cotran "Patología estructural y funcional". Corazón. Cardiopatía
Congénita. Ed.Elsevier. pp.(536).

2 - Paciente de 47 años de edad con disnea, antecedentes de “Lupus eritematoso sistémico”. Examen físico: ruidos
cardíacos alejados, TA 90/60, FC 122/min. Se realiza ECG, Rx (IC-Índice cardiotorácico: aumentado con silueta
cardíaca en forma de "botellón") y angiotac que confirman el diagnóstico. Se realiza pericardiocentesis. ¿Cuál es el tipo
de derrame pericárdico más frecuente en esta entidad nosológica?
a) Fibrinoso
b) Purulento
c) Seroso
d) Hemorrágico
- Kumar, A. and Aster. (2015). 9na.ed. Robbins y Cotran "Patología estructural y funcional". Corazón. Enfermedad
Pericárdica. Ed.Elsevier. pp.(573).

3 - Un hombre de 68 años, fumador de 1 atado de cigarrillos diario, consulta por tos persistente y adelgazamiento. En
la fibrobroncoscopía se observa una lesión que se biopsia, y el diagnóstico anatomopatológico resulta ser “Carcinoma
de Células Pequeñas”. Cuál de los siguientes enunciados es INCORRECTO:
a) Rara vez se comporta agresivamente
b) Es uno de los tipos de Cáncer de Pulmón que se asocia con mayor frecuencia a la producción de hormonas
ectópicas.
c) Todos los carcinomas de células pequeñas son de alto grado
d) La inmunohistoquímica demuestra niveles altos de BCL2 en el 90% de los tumores de este tipo.
- Kumar, A. and Aster. (2015). 9na.ed. Robbins y Cotran "Patología estructural y funcional". Pulmón. Tumores.
Ed.Elsevier. pp.(717).

4 - ¿Cuál de las siguientes descripciones corresponde a “Neumonía en período de hepatización roja”?


a) Hiperemia vascular con abundante líquido intraalveolar y escasos neutrófilos
b) Importante digestión enzimática e infiltrado macrofágico
c) Alveolos ocupados por neutrófilos, fibrina y detritus
d) Alveolos ocupados por neutrófilos y glóbulos rojos
- Kumar, A. and Aster. (2015). 9na.ed. Robbins y Cotran "Patología estructural y funcional". Pulmón. Infecciones
Pulmonares. Ed.Elsevier. pp.(704).

5 - Una paciente de 40 años presentó deformidad del cuello, disfagia de 1 mes de evolución, episodios de taquicardia
y diarrea. En la ecografía del cuello, en la tiroides, tenía una masa homogénea hiperecoica de 6cm de diámetro, que
en el estudio intraoperatorio correspondía a una masa tumoral bien delimitada, de color pardo rojizo, sólida con algunos
focos de hemorragia. La microscopía reveló “células poligonales dispuestas en nidos y cordones separados por
abundante matriz amorfa extracelular, eosinófila (Rojo Congo +). Inmunohistoquímica: CEA y calcitonina (+)”. ¿Cuál es
para Ud. el diagnóstico más probable?
a) Carcinoma papilar de tiroides, variante sólida
b) Carcinoma folicular de tiroides
c) Carcinoma medular de tiroides
d) Adenoma folicular
- Kumar, A. and Aster. (2015). 9na.ed. Robbins y Cotran "Patología estructural y funcional". Sistema Endócrino.
Neoplasias Tiroideas. Ed.Elsevier. pp.(1099).
- Figura 24-22. Pág.1099
6 - Un paciente presenta tumoración en la metáfisis del fémur derecho que en la Rx se ve como una excrecencia
fungiforme con áreas cortical y medular continuas con las del hueso donde se origina. Se reseca e histológicamente
se observa una “proliferación de tejido óseo maduro cubierta por fuera por una capa de cartílago hialino”. ¿Cuál es
para Ud. el diagnóstico más probable?
a) Osteocondroma
b) Osteosarcoma
c) Condrosarcoma
d) Encondroma
- Kumar, A. and Aster. (2015). 9na.ed. Robbins y Cotran "Patología estructural y funcional". Huesos, articulaciones y
tumores de partes blandas. Tumores formadores de cartílago. Ed.Elsevier. pp.(1200).
- Figura 26-26. Pág. 1200
7 - ¿Cuál de las siguientes características corresponde con el Carcinoma Basocelular de piel?
a) Crecimiento rápido con invasión local.
b) Crecimiento lento con invasión local.
c) Frecuente invasión ganglionar.
d) Frecuente presencia de metástasis.
- Kumar, A. and Aster. (2015). 9na.ed. Robbins y Cotran "Patología estructural y funcional". Piel. Tumores epidérmicos
premalignos y malignos. Ed.Elsevier. pp.(1155).

8 - El hallazgo de células escamosas con núcleos grandes hipercromáticos, con irregularidad de la membrana nuclear y halo
perinuclear en la citología exfoliativa cérvico-vaginal (PAP) indica:
a) Mucosa cervical típica
b) SIL de alto grado
c) Carcinoma cervical invasor
d) Presencia de coilocitos por efecto citopàtico del HPV
- Kumar, A. and Aster. (2015). 9na.ed. Robbins y Cotran "Patología estructural y funcional". Aparato Genital Femenino. Cuello
uterino. Ed.Elsevier. pp.(1003).

9 - Referente a los tumores mucinosos de ovario, señale la opción CORRECTA.


a) Representan el 80% de todas las neoplasias del ovario.
b) Se producen principalmente antes de la pubertad.
c) El 75% son tumores malignos.
d) La mutación del prooncogen KRAS es una alteración genética presente en la mayoría de estos tumores.
- Kumar, A. and Aster. (2015). 9na.ed. Robbins y Cotran "Patología estructural y funcional". Aparato Genital Femenino. .
Ed.Elsevier. pp.(1026).

10 - Recibe la biopsia de un nódulo de la mama derecha de una paciente de 45 años y decide comenzar terapia neoadyuvante
con el anticuerpo monoclonal trastuzumab. ¿Cuál de las siguientes opciones fue positiva para tomar dicha decisión?
a) BRCA1
b) BRCA2
c) Her2/neu
d) p53
- Kumar, A. and Aster. (2015). 9na.ed. Robbins y Cotran "Patología estructural y funcional". Mama. Ed.Elsevier. pp.(1062).
14/12/2020 EXAMEN FINAL 14 DE DICIEMBRE 2020

EXAMEN FINAL 14 DE DICIEMBRE 2020


*Obrigatório

Instrucciones

RECUERDE:
- En las preguntas tipo múltiple choice, 1 sola opción es la correcta.
- En las preguntas tipo casilla de verificación, deberá identificar todos los enunciados CORRECTOS
teniendo en cuenta que el marcar enunciados correctos de menos o enunciados incorrectos, la pregunta
se da automáticamente por anulada.

En referencia a la entidad “Melanoma", señale cual/es de los siguientes


enunciados es CORRECTO. Recuerde que deberá marcar todas las opciones
correctas, considerando que al marcar opciones correctas de menos o 1
respuesta incorrecta, el sistema dará por anulado el ejercicio. *

El fenómeno de satelitosis no se relaciona con el aumento de la recurrencia.

Son tumores malignos con diferenciación melanocítica y que presenta la mutación


activadora del BRAF pasible de ser blanco terapéutico.

Con respecto a su relación con la exposición solar, algunos estudios indican que las
quemaduras intensas y periódicas al principio de la vida son el factor de riesgo más
importante.

La fase de crecimiento vertical se correlaciona con la aparición de un subclon


tumoral con potencial metastásico.

El parámetro de medición de Breslow determina la dimensión de crecimiento radial


del melanoma. INCORRECTO: Determina la dimensión del crecimiento vertical.

no todos los melanomas en los sujetos con Síndrome del nevo displásico se originan
sobre nevos displásicos.
https://docs.google.com/forms/d/e/1FAIpQLSdKi_dZV2bOZn2rwTEhPhlWRrzWZ94_YBeF3iMBprHZ4Z3XrQ/formResponse 1/28
14/12/2020 EXAMEN FINAL 14 DE DICIEMBRE 2020

Un niño de 5 años ha desarrollado durante los últimos 6 meses rasgos indicativos


de pubertad. La exploración física revela presencia de características sexuales
secundarias, como vello púbico y aumento del tamaño peneano. ¿Cuáles de las
siguientes características morfológicas es más probable que se encuentre en sus
glándulas suprarrenales? *

Hiperplasia cortical

Nódulo medular

Hiperplasia medular

Atrofia cortical

Una lactante de 8 meses es llevada a servicio de oftalmología pediátrica. La


madre refiere que aproximadamente a los 3 meses de edad notó que la lactante
presentaba desviación del ojo derecho hacía afuera y un “brillo blanco en el ojo
de la niña”. Como antecedentes, la niña nació en un parto doméstico dado que la
familia vive en área rural con difícil acceso a centros de atención médica. Al ser
evaluada se constata leucocoria y baja agudeza visual. En base a su diagnóstico
presuntivo, señale la afirmación CORRECTA: *

Lo más probable es que la niña tenga un retinoblastoma, que histológicamente se


caracteriza por ser pleomórfico, con células poliédricas de citoplasma claro y grandes
núcleos con nucléolo prominente.

El antecedente de parto doméstico pudo haber influido en la génesis de este tumor


debido al aumento de posibilidades de infecciones intraoculares por acontecer en un
ambiente con asepsia inadecuada..

Por la edad de presentación, lo más probable es que haya heredado un alelo mutado
del gen Rb y tiene mayor posibilidad de presentar un tumor de iguales características
en el ojo contralateral que si la presentación se hubiera dado a mayor edad.

https://docs.google.com/forms/d/e/1FAIpQLSdKi_dZV2bOZn2rwTEhPhlWRrzWZ94_YBeF3iMBprHZ4Z3XrQ/formResponse 2/28
14/12/2020 EXAMEN FINAL 14 DE DICIEMBRE 2020
La alteración genética que caracteriza a estos tumores es la translocación EWS/ETS.

Un varón de 50 años presenta tos crónica, úlceras nasofaríngeas con aumento


de creatinina y urea en sangre. Una biopsia de fosas nasales muestra zona de
necrosis y vasculitis de vasos pequeños y mediano calibre. Se encuentran títulos
aumentados de c-ANCA. Pensamos en: *

Poliangitis microscópica.

Poliarteritis nodosa.

Vasculitis de Churg-Strauss.

Granulomatosis con polivasculitis (Enfermedad de Wegener) .

Una pieza de nefrectomía derecha muestra macroscópicamente grandes


cicatrices con forma de U en los polos con amputación de los cálices. El estudio
histológico revela infiltrados inflamatorios, que se extienden desde la médula a la
corteza con destrucción de los túbulos y extensa fibrosis intersticial. Se
encuentran abundantes linfocitos, células plasmáticas y neutrófilos. ¿Cuál de los
siguientes trastornos es más probable que ocasione estos hallazgos? *

Lupus eritematoso sistémico

Insuficiencia cardíaca congestiva

Reflujo vesicoureteral

Poliquistosis renal

https://docs.google.com/forms/d/e/1FAIpQLSdKi_dZV2bOZn2rwTEhPhlWRrzWZ94_YBeF3iMBprHZ4Z3XrQ/formResponse 3/28
14/12/2020 EXAMEN FINAL 14 DE DICIEMBRE 2020

Paciente de 26 años que consulta por presentar lesión ulcerada y sobreinfectada


en la piel del pene de 1 año de evolución. Se toma biopsia la cual informa:
“proliferación de células pavimentosas con núcleos irregulares e hipercromáticos
con pérdida de la polaridad nuclear. Se observan puentes intercelulares y focos
de queratinización en forma de perla córnea.”. ¿Cuál es para Ud. el diagnóstico
más probable? *

Enfermedad de Bowen

Condiloma acuminado

Carcinoma epidermoide invasivo

Papulosis bowenoide

En referencia a la "Patología vesical", señale cuál de los siguientes enunciados es


CORRECTO: *

La cistitis intersticial predomina en hombres y consta de fisuras, úlceras y una


fibrosis que puede comprometer todo el espesor de la pared.

La malacoplaquia es una infección generalmente provocada por Escherichia coli, e


histológicamente se caracteriza por la presencia de los cuerpos de Michaelis-
Gutmann que están en el interior de las células uroteliales.

En los papilomas vesicales se observa efecto citopático por la acción del HPV.

En la cistitis crónica los infiltrados de linfocitos pueden llegar a formar folículos


linfoides con presencia de centros germinales, constituyendo una cistitis folicular.

https://docs.google.com/forms/d/e/1FAIpQLSdKi_dZV2bOZn2rwTEhPhlWRrzWZ94_YBeF3iMBprHZ4Z3XrQ/formResponse 4/28
14/12/2020 EXAMEN FINAL 14 DE DICIEMBRE 2020

Una mujer de 65 años acude a control de salud y el único hallazgo es un


chasquido mesosistólico en la auscultación cardíaca. En los 5 años siguientes
desarrolla disnea progresiva. El ecocardiograma revela, insuficiencia mitral por
prolapso de una valva. ¿Cuál de las siguientes alteraciones es más probable que
presente esa valva? *

Necrosis fibrinoide

Fibrosis reumática

Degeneración mixomatosa

Calcificación distrófica

Paciente con enfermedad de Hodgkin que cursa con fiebre nocturna, diaforesis,
pérdida de peso, linfadenopatías en región cervical derecha y retroperitoneal.
¿Cuál es el estadio de Ann Arbor? *

II B

III A

II A

III B

Paciente femenina de 27 años, cursando su primer mes de puerperio, consulta


por síndrome febril asociado a eritema y dolor en la mama derecha. ¿Cuál es la
etiología más probable? *

Infección por Mycobacterium tuberculosis.


https://docs.google.com/forms/d/e/1FAIpQLSdKi_dZV2bOZn2rwTEhPhlWRrzWZ94_YBeF3iMBprHZ4Z3XrQ/formResponse 5/28
14/12/2020 EXAMEN FINAL 14 DE DICIEMBRE 2020
p y

Estasis de conductos galactóforos.

Patología maligna de la mama.

Infección por Staphylococcus aureus.

¿Que esperaría observar macroscópicamente en el Carcinoma Renal? *

nódulo pequeño, bien delimitado, homogéneo, de coloración de coloración


amarronada.

quiste de paredes gruesas, tabicado, revestido por papilas finas y friables que se
desprenden con facilidad.

masas esféricas de 3 a 15 cm de diámetro, habitualmente en un polo, bien


delimitadas, que deforman la silueta renal, de coloración amarillenta, con áreas
reblandecidas opacas blanco-grisáceas y focos de hemorragia.

masas de 5 a 10 cm de diámetro, blanquecinas, fibrosas, de límites poco netos, con


pequeños focos de hemorragia.

https://docs.google.com/forms/d/e/1FAIpQLSdKi_dZV2bOZn2rwTEhPhlWRrzWZ94_YBeF3iMBprHZ4Z3XrQ/formResponse 6/28
14/12/2020 EXAMEN FINAL 14 DE DICIEMBRE 2020

En referencia al “Carcinoma de próstata", señale cual/es de los siguientes


enunciados es CORRECTO. *

Un Gleason combinado 10 representa un adenocarcinoma bien diferenciado y de


buen pronóstico.

La gradación del carcinoma de próstata y la estadificación son los mejores factores


predictivos del pronóstico.

Las metástasis óseas son típicamente osteoclásticas y afectan en primera instancia


la columna lumbar.

Se localiza más frecuentemente en la zona periuretral.

Seleccionar la respuesta CORRECTA respecto a las lesiones de vagina: *

Casi todos los carcinomas primarios de la vagina son adenocarcinomas asociados a


infección por Herpes virus.

El tumor maligno que con más frecuencia afecta a la vagina es el carcinoma que se
extiende desde el cuello uterino.

El tumor epitelial maligno primario de vagina es una patología frecuente.

El sarcoma botrioides de vulva es un sarcoma de alta malignidad que se presenta en


mujeres añosas.

https://docs.google.com/forms/d/e/1FAIpQLSdKi_dZV2bOZn2rwTEhPhlWRrzWZ94_YBeF3iMBprHZ4Z3XrQ/formResponse 7/28
14/12/2020 EXAMEN FINAL 14 DE DICIEMBRE 2020

Una mujer de 46 años consulta al servicio de dermatología por unas lesiones


papulovesiculosas y pruriginosas en codos y rodillas, y en el interrogatorio refiere
episodios de diarrea y de debilidad por anemia desde hace años. Sospechando
que pudiere tratarse de celiaquía se le solicitan pruebas serológicas que
determinan altos títulos de IgA anti-transglutaminasa tisular y anti-endomisio.
Con todos los antecedentes clínicos y los datos de laboratorio, la paciente es
referida al gastroenterólogo, quien a los pocos días realiza una endoscopia para
tomar muestras de distintos sectores de su mucosa gástrica para el posterior
estudio anatomopatológico de las mismas. Respecto a la enfermedad celíaca,
señale la respuesta CORRECTA: *

El tratamiento comprende una dieta exenta de trigo, avena, cebada y centeno


(popularmente conocida como "sin TACC"), siendo la avena la gramínea con mayor
concentración de gluten.

Se vincula a distintas neoplasias malignas, siendo las más frecuentes los linfomas,
principalmente de estirpe B, y adenocarcinomas.

El hallazgo en conjunto de un incremento de linfocitos intraepiteliales y una atrofia


vellositaria de diversos grados es patognomónico para establecer el diagnóstico.

La malabsorción se debe a una disminución del tamaño de las vellosidades


intestinales y al reemplazo de enterocitos por células que aún son inmaduras.

Una paciente de 54 años consulta por cefalea. Se le realiza una TAC de cerebro
observándose una lesión nodular, parasagital cerca de la hoz del cerebro y en
contacto con las meninges. Qué hallazgos histopatológicos Ud. esperaría
encontrar: *

Masas sólidas compactas de células ahusadas, con presencia de cuerpos de


psammoma.

Células pequeñas indiferenciadas, hipercromasia, moldeamiento y abundantes


mitosis
https://docs.google.com/forms/d/e/1FAIpQLSdKi_dZV2bOZn2rwTEhPhlWRrzWZ94_YBeF3iMBprHZ4Z3XrQ/formResponse 8/28
14/12/2020 EXAMEN FINAL 14 DE DICIEMBRE 2020
mitosis.

Células escamosas atípicas con ocasionales perlas corneas.

Células atípicas que se disponen formando estructuras glandulares, algunas con


vacuolas citoplasmáticas.

En referencia a los “Tumores de partes blandas", señale cual de los siguientes


enunciados es CORRECTO. *

El sarcoma botrioides es una variante de leiomiosarcoma que se ubica en cavidades


que se presenta en adultos jóvenes.

El liposarcoma se presenta en personas adultas, de 50 a 70 años, y se localiza en las


extremidades, sobre todo en muslo y también en retroperitoneo.

El fibroma es el tumor benigno más frecuente de partes blandas en adultos.

El sarcoma sinovial deriva de los sinoviocitos.

En referencia a la “Hipertensión Pulmonar", señale cual/es de los siguientes


enunciados es CORRECTO. Recuerde que deberá marcar todas las opciones
correctas, considerando que al marcar opciones correctas de menos o 1
respuesta incorrecta, el sistema dará por anulado el ejercicio. *

La coexistencia de comorbilidades como fibrosis pulmonar, enfisema y bronquitis


crónica grave puede provocar hipoxia crónica y causar HTP.

La apnea obstructiva del sueño es un trastorno frecuente que se asocia a obesidad e


hipoxemia y puede ser un factor contribuyente al desarrollo de HTP y corazón
pulmonar.

El tratamiento de la HTP es sintomático, mediante el uso de vasodilatadores; el


trasplante de pulmón o cardiopulmonar está contraindicado en los pacientes con HTP
severa.

En la mayoría de los casos familiares de HTP se observan mutaciones inactivadoras


en la línea germinal del gen de la proteína morfogenética ósea de tipo 2 (BMPR2).

Las lesiones plexiformes, caracterizadas por la presencia de un “penacho” de


formaciones capilares que forman una red que llena la luz de las arterias pequeñas,
dilatadas, de paredes finas, es la lesión característica incipiente que se observa en
los casos más leves de HTP.
https://docs.google.com/forms/d/e/1FAIpQLSdKi_dZV2bOZn2rwTEhPhlWRrzWZ94_YBeF3iMBprHZ4Z3XrQ/formResponse 9/28
14/12/2020 EXAMEN FINAL 14 DE DICIEMBRE 2020

La hipertensión pulmonar idiopática se presenta con mayor frecuencia en hombres


mayores de 60 años, con disnea, cansancio y dolor torácico de tipo anginoso.

La hipertensión pulmonar (HTP) se define como una presión arterial pulmonar media
igual o mayor a 25 mmHg en reposo.

En referencia a los “Pólipos de colon", señale cuál de los siguientes enunciados es


INCORRECTO. *

El síndrome de Gardner corresponde a múltiples adenomas asociados a osteomas,


quistes cutáneos y desmoides.

Los adenomas sésiles serrados no presentan displasia citológica y comparten


características morfológicas con los pólipos hiperplásicos.

La poliposis adenomatosa familiar y el cáncer colorrectal hereditario no poliposo


ejemplifican vías distintivas de transformación y progresión neoplásica y son las vías
más frecuentes en el cáncer de colon familiar.

Los pólipos hiperplásicos tienen potencial de malignidad con transformación


adenocarcinomatosa.

Paciente femenina de 59 años, nulípara, consulta por dolor abdominal y cambios


en el ritmo evacuatorio alternando constipación y diarrea, de un mes de
evolución. Refiere palparse una masa en fosa ilíaca izquierda. Entre los estudios
realizados, la ecografía transvaginal evidenció una estructura quística mixta en
anexo izquierdo de 132 x 145mm, asociada a líquido libre en fondo de saco de
Douglas compatible con ascitis. Con estos datos, y teniendo en cuenta la
estadística y frecuencia de los tumores de ovario, ¿qué esperaría encontrar en el
estudio anatomopatológico de esta lesión? *

Paredes revestidas de epitelio cilíndrico ciliado con fibrosis y líquido seroso.

Proliferación abundante de células epitelioides cilíndricas con núcleos polimórficos


que infiltran la cápsula ovárica.

Células epiteliales cilíndricas altas con mucina apical.

Estructuras glandulares confluyentes con marcada atipia celular y polimorfismo


nuclear y secreción anómala de mucina.

https://docs.google.com/forms/d/e/1FAIpQLSdKi_dZV2bOZn2rwTEhPhlWRrzWZ94_YBeF3iMBprHZ4Z3XrQ/formResponse 10/28
14/12/2020 EXAMEN FINAL 14 DE DICIEMBRE 2020

Un hombre de 70 años consulta por dolores de espalda. Como antecedentes


refiere haber tenido múltiples cuadros infecciosos el último año que los resolvió
con tratamiento antibiótico. El médico lo evalúa y le solicita laboratorio y estudios
por imágenes. Al traer los resultados se constatan múltiples lesiones osteolíticas
en cuerpos vertebrales, que aparecen como imágenes en sacabocados y miden
alrededor de 2 a 3 cm. En el laboratorio muestra anemia normocítica y
normocrómica, aumento de urea, creatinina y calcio, con PSA dentro de los
valores considerados normales para la edad. En base a estos datos, señale la
opción correcta: *

Las lesiones óseas tienen los rasgos que característicamente se asocian a


metástasis de cáncer de próstata.

Por sospecha de mieloma múltiple, podría solicitarse dosaje de cadenas ligeras


(proteínas de Bence Jones) en orina e Ig en sangre periférica.

En esta patología, la presencia de deleciones 13q y 17p indican buen pronóstico.

El valor de PSA normal descarta el diagnóstico de cáncer de próstata.

https://docs.google.com/forms/d/e/1FAIpQLSdKi_dZV2bOZn2rwTEhPhlWRrzWZ94_YBeF3iMBprHZ4Z3XrQ/formResponse 11/28
14/12/2020 EXAMEN FINAL 14 DE DICIEMBRE 2020

En referencia a las “Proliferaciones leucocitarias reactivas”, señale cuales de los


siguientes enunciados es CORRECTO. Recuerde que deberá marcar todas las
opciones correctas, considerando que el marcar correctas de menos o 1
respuesta incorrecta, dará por anulado el ejercicio. *

La histiocitosis sinusal es el patrón morfológico característico observado en la


linfadenitis aguda inespecífica.

La hiperplasia folicular debe ser diferenciada morfológicamente del linfoma folicular


para lo cual una de las características es la conservación de la arquitectura
ganglionar acompañada de una importante variación del tamaño y la forma de los
folículos.

Las infecciones crónicas generan leucocitosis por aumento de producción de


factores de crecimiento hematopoyético en médula ósea.

En situaciones de stress, los glucocorticoides disminuyen la extravasación de


leucocitos hacia los tejidos.

Las reacciones leucemoides son patognomónicas de las leucemias mieloides.

En el contexto de una apendicitis aguda, la adenitis mesentérica observada consta de


ganglios macroscópicamente ingurgitados que histológicamente presentan grandes
centros germinales reactivos acompañados de figuras mitóticas e infiltrado
polimorfonuclear.

https://docs.google.com/forms/d/e/1FAIpQLSdKi_dZV2bOZn2rwTEhPhlWRrzWZ94_YBeF3iMBprHZ4Z3XrQ/formResponse 12/28
14/12/2020 EXAMEN FINAL 14 DE DICIEMBRE 2020

Un hombre de 53 años con una miocardiopatía dilatada idiopática se somete a un


trasplante cardíaco. Durante los 5 años siguientes presentó episodios de rechazo
celular que han sido tratados exitosamente aumentando la inmunodepresión. En
la exploración física se identifica la lesión que se muestra en la imagen. ¿Cuál de
las siguientes lesiones es mas probable que se encuentre en la biopsia? *

Dermatofibroma

Psoriasis

Liquen plano

Carcinoma epidermoide

https://docs.google.com/forms/d/e/1FAIpQLSdKi_dZV2bOZn2rwTEhPhlWRrzWZ94_YBeF3iMBprHZ4Z3XrQ/formResponse 13/28
14/12/2020 EXAMEN FINAL 14 DE DICIEMBRE 2020

Una mujer de 21 años padece fatiga progresiva con pérdida involuntaria de 7kg
de peso durante los últimos 4 meses acompañado de ansiedad y nerviosismo
con diarrea. En el examen físico se verifica que la glándula tiroides se encuentra
aumentada de tamaño de forma difusa. Temperatura de 37,5°C, FC: 103lat/min;
FR:28resp/min; TA 140/75mmHg. La gammagrafía tiroidea confirma un patrón de
captación difusa. Se muestran las imágenes macroscópicas y microscópicas.
Estos hallazgos son producidos por: *

Deficiencia de yodo en la dieta

Irradiación del cuello

Anticuerpos frente al receptor de TSH

Mutación del protooncogén RET

https://docs.google.com/forms/d/e/1FAIpQLSdKi_dZV2bOZn2rwTEhPhlWRrzWZ94_YBeF3iMBprHZ4Z3XrQ/formResponse 14/28
14/12/2020 EXAMEN FINAL 14 DE DICIEMBRE 2020

Con respecto a la Patología Tiroidea, señale la INCORRECTA. *

La tiroiditis de Hashimoto es de origen autoinmune y predomina en varones.

La enfermedad de Graves es la causa más frecuente de hipertiroidismo y corresponde


a un trastorno autoinmunitario.

El déficit de iodo en la dieta es la principal causa de hipotiroidismo en países


subdesarrollados.

La presencia de metástasis en ganglios linfáticos cervicales no tiene influencia


relevante en el pronóstico de los carcinomas papilares, que en general es bueno.

En referencia a la entidad nosológica “Tumores Seminomatosos”, indique la


afirmaciòn INCORRECTA: *

El seminoma clásico es el tumor más frecuente de testículo y predomina en adultos


de la 2da a 3ra década de la vida.

A pesar de ser quimiosensible tiene mal pronóstico por su diseminación precoz

15% de los seminomas son GCH + por la presencia de células trofoblásticas


intratumorales.

En los tumores mixtos puede combinarse con el coriocarcinoma.

https://docs.google.com/forms/d/e/1FAIpQLSdKi_dZV2bOZn2rwTEhPhlWRrzWZ94_YBeF3iMBprHZ4Z3XrQ/formResponse 15/28
14/12/2020 EXAMEN FINAL 14 DE DICIEMBRE 2020

Una mujer de 45 años ha presentado episodios de hemorragia vaginal fuera del


período menstrual y secreciones fétidas, malolientes, parduzcas durante los
últimos seis meses. En la exploración pélvica se reconoce una masa exofítica y
ulcerada de 4 cm, en el ectocérvix que ocupa los cuatro cuadrantes. En la biopsia
se describió una proliferación celular atípica que disrumpía la membrana basal en
forma de nidos y cordones. ¿Cuál de los siguientes diagnósticos es el más
probable? *

Pólipo endocervical

Carcinoma pavimentoso infiltrante

LSIL

Cervicitis crónica

Una mujer de 52 años, ingeniera electrónica y no tabaquista, refiere disnea


progresiva de 3 meses de evolución. Al examen físico está afebril y normotensa.
La TAC torácica muestra opacidades reticulares en los lóbulos inferiores. Se
realiza una biopsia transbronquial y el estudio histopatológico revela:
“Inflamación intersticial parcheada con infiltrado linfoplasmocitario. No se
reconocen microorganismos”. La situación clínica de la paciente se deteriora de
forma gradual y lenta en los siguientes 10 años. ¿Cuál de los siguientes es el
diagnóstico más probable? *

Neumonía intersticial inespecífica

Fibrosis intersticial idiopática

Neumonitis intersticial descamativa

Neumonitis por hipersensibilidad

https://docs.google.com/forms/d/e/1FAIpQLSdKi_dZV2bOZn2rwTEhPhlWRrzWZ94_YBeF3iMBprHZ4Z3XrQ/formResponse 16/28
14/12/2020 EXAMEN FINAL 14 DE DICIEMBRE 2020

Paciente de 12 años, sin antecedentes traumáticos, comienza con malestar


general y fiebre. Además, refiere dolor y tumefacción a nivel del tercio medio del
fémur derecho. Hace 10 días presentó una infección odontológica que
evolucionó favorablemente con la medicación indicada. Una Rx. revela en la
diáfisis femoral, la presencia de un área focal lítica de bordes difusos con
reacción perióstica en capas. La biopsia de la lesión reveló estar constituida por
células pequeñas, redondas y azules que en sectores se disponen formando
rosetas. ¿Cuál es el diagnóstico más probable? *

Osteomielitis hematógena.

Sarcoma de Ewing/PNET.

Quiste óseo aneurismático.

Osteosarcoma.

https://docs.google.com/forms/d/e/1FAIpQLSdKi_dZV2bOZn2rwTEhPhlWRrzWZ94_YBeF3iMBprHZ4Z3XrQ/formResponse 17/28
14/12/2020 EXAMEN FINAL 14 DE DICIEMBRE 2020

En referencia a la “Tumores del SNC", señale cual/es de los siguientes enunciados


es CORRECTO. Recuerde que deberá marcar todas las opciones correctas,
considerando que al marcar opciones correctas de menos o 1 respuesta
incorrecta, el sistema dará por anulado el ejercicio. *

El meduloblastoma aparece predominantemente en niños y en cerebelo.

Los gliomas (astrocitomas, oligodendrogliomas y ependimomas) conforman el grupo


más frecuente de tumores cerebrales primarios.

Los oligodendrogliomas anaplásicos con codeleción 1p/19q tienen menor respuesta


a la quimio y radioterapia y consecuentemente peor pronóstico.

El glioblastoma multiforme se caracteriza por presentar áreas de necrosis y


proliferación vascular.

Se denominan ependimomas mixopapilares a los tumores ubicados en el filum


terminale de la médula y que contienen elementos papilares sobre un fondo mixoide.

El glioblastoma se caracteriza por presentar células bipolares con prolongaciones


largas y finas que son GFAP (+) acompañadas de fibras de Rosenthal y cuerpos
granulares eosinófilos.

En el oligodendroglioma la presencia de mutaciones IDH1, IDH2 se asocia a mejor


pronóstico.

https://docs.google.com/forms/d/e/1FAIpQLSdKi_dZV2bOZn2rwTEhPhlWRrzWZ94_YBeF3iMBprHZ4Z3XrQ/formResponse 18/28
14/12/2020 EXAMEN FINAL 14 DE DICIEMBRE 2020

En referencia a la “Tumores de cavidad oral y orofaringe", señale cual/es de los


siguientes enunciados es CORRECTO. Recuerde que deberá marcar todas las
opciones correctas, considerando que al marcar opciones correctas de menos o
1 respuesta incorrecta, el sistema dará por anulado el ejercicio. *

El fibroma de irritación o fibroma traumático es una masa nodular submucosa de


estroma de tejido conjuntivo fibroso que aparece principalmente en la mucosa bucal
a lo largo de la línea de mordida o las encías.

Los carcinomas de células escamosas de la cavidad oral están relacionados


clásicamente con el consumo de alcohol y tabaco.

El carcinoma de células escamosas de la cavidad oral está asociado al HPV


frecuentemente.

Los tumores malignos «clásicos» de la cavidad oral no tienen lesiones precursoras


reconocibles.

La mayoría de los tumores malignos de cavidad oral son adenocarcinomas.

La mayoría de los carcinomas de orofaringe son carcinomas epidermoides.

En la orofaringe, hasta el 70% de los carcinomas de células escamosas,


especialmente de amígdalas, base de la lengua y faringe, contienen variantes
oncogénicas del HPV, sobre todo HPV-16.

https://docs.google.com/forms/d/e/1FAIpQLSdKi_dZV2bOZn2rwTEhPhlWRrzWZ94_YBeF3iMBprHZ4Z3XrQ/formResponse 19/28
14/12/2020 EXAMEN FINAL 14 DE DICIEMBRE 2020

En relación a las enfermedades ampollares de la piel, indique la opción


INCORRECTA: *

El pénfigo vulgar es la forma más frecuente de la enfermedad y consta de vesículas


superficiales y ampollas distribuidas en mucosas y piel de cuero cabelludo, cara,
axilas, ingle y superficies de presión, que se rompen fácilmente.

El penfigoide ampolloso suele afectar a pacientes jóvenes con lesiones en cara


interna de muslos y superficies flexoras de antebrazos, axilas e ingle. Suelen
observarse lesiones orales en un 80-85% de los casos.

La dermatitis herpetiforme (Enfermedad de Duhring-Brocq) se observa con mayor


frecuencia en hombres y constituye un trastorno infrecuente caracterizado por
urticaria y vesículas agrupadas.

El pénfigo vegetante es una forma infrecuente de la enfermedad que suele debutar


clínicamente sin ampollas y con presencia de placas verrugosas de apariencia
húmeda y pústulas, predominantemente en ingles, axilas y superficies flexoras.

https://docs.google.com/forms/d/e/1FAIpQLSdKi_dZV2bOZn2rwTEhPhlWRrzWZ94_YBeF3iMBprHZ4Z3XrQ/formResponse 20/28
14/12/2020 EXAMEN FINAL 14 DE DICIEMBRE 2020

En referencia a la “Cardiopatía isquémica", señale cual/es de los siguientes


enunciados es CORRECTO. Recuerde que deberá marcar todas las opciones
correctas, considerando que al marcar opciones correctas de menos o 1
respuesta incorrecta, el sistema dará por anulado el ejercicio. *

A las tres horas de producido el infarto de miocardio, microscópicamente se observa


un infiltrado inflamatorio intersticial de neutrófilos.

La arteria coronaria más frecuentemente afectada cuya lesión desencadena un


infarto de miocardio es la circunfleja izquierda.

En más del 90% de los casos, la isquemia miocárdica es consecuencia de reducción


del flujo sanguíneo debida a lesiones ateroescleróticas obstructivas en las arterias
coronarias epicárdicas.

Entre las complicaciones del infarto se encuentran la rotura ventricular, rotura del
músculo papilar, la formación de aneurismas, trombo mural, arritmias, la pericarditis y
la insuficiencia cardíaca tardía progresiva.

El infarto de miocardio se diagnostica en virtud de los síntomas, los cambios


electrocardiográficos y la medición de CK-MB y troponinas. Las lesiones macro y
microscópicas siguen una cronodinamia que se desarrolla tras horas o días.

El término angina inestable hace referencia al patrón de angina o molestia torácica


causada por espasmo arterial coronario.

La característica común de los síndromes coronarios agudos es la isquemia


miocárdica anterógrada.

https://docs.google.com/forms/d/e/1FAIpQLSdKi_dZV2bOZn2rwTEhPhlWRrzWZ94_YBeF3iMBprHZ4Z3XrQ/formResponse 21/28
14/12/2020 EXAMEN FINAL 14 DE DICIEMBRE 2020

Paciente masculino de 67 años concurre a la guardia presentando dolor


retroesternal que irradia hacia el dorso y que aumenta su intensidad. Refiere que
nunca lo había sentido y que es insoportable. Como antecedentes de relevancia
el paciente refiere haber tenido sífilis cuando tenía 20 años tratado con penicilina
e hipertensión diagnosticada a los 30 años, no tratada. La causa de su dolor se
debe más frecuentemente a: *

Oclusión de la arteria carótida interna derecha, provocando isquemia distal a su


obstrucción.

Desgarro de la capa íntima de la aorta y propagación del hematoma por debilidad de


la capa media.

Desgarro de la capa íntima de la raíz de la aorta, debilitada por un aneurisma


preexistente.

Reflujo gastroesofágico que produce irritación de la mucosa esofágica provocando


dolor retroesternal.

https://docs.google.com/forms/d/e/1FAIpQLSdKi_dZV2bOZn2rwTEhPhlWRrzWZ94_YBeF3iMBprHZ4Z3XrQ/formResponse 22/28
14/12/2020 EXAMEN FINAL 14 DE DICIEMBRE 2020

Una mujer de 85 años con antecedentes de HTA, hipotiroidismo y enfermedad de


Parkinson, ingresa por anemia aguda (Hto 16%, previamente presentaba 35%) y
disnea de 7 días de evolución. Este cuadro se acompaña de hemorragia digestiva
alta manifestada por melena. Se efectúo endoscopía digestiva alta que reveló
lesión tipo Borrmann IV decidiéndose intervención quirúrgica. Se presentan los
hallazgos correspondientes a la pieza quirúrgica de gastrectomía. El diagnóstico
anatomopatológico más probable es: *

carcinoma gástrico variante difusa con células en “anillo de sello”

tumor del estroma gastrointestinal (GIST)


https://docs.google.com/forms/d/e/1FAIpQLSdKi_dZV2bOZn2rwTEhPhlWRrzWZ94_YBeF3iMBprHZ4Z3XrQ/formResponse 23/28
14/12/2020 EXAMEN FINAL 14 DE DICIEMBRE 2020

carcinoma gástrico variante intestinal

carcinoide

Respecto de las "Neoplasias óseas", señale cual/es de los siguientes enunciados


es INCORRECTO: *

El osteosarcoma es la neoplasia ósea maligna no hematopoyética más frecuente, y


suele afectar con mayor frecuencia a varones, a nivel de la rodilla.

La exostosis es una alteración consistente en crecimiento de hueso laminar con un


recubrimiento de cartílago hialino, que prolifera hasta generar células con marcada
atipía.

El osteosarcoma se caracteriza por una proliferación de células osteoblásticas


malignas, pleomórficas, y presencia de matriz osteoide o de hueso maduro.

El osteoma osteoide se caracteriza por la presencia de un nido que presenta una


mineralización irregular y que se rodea de una zona de esclerosis reactiva.

En referencia a las “Nefropatías poliquísticas renales”, señale cuál de los


siguientes enunciados es INCORRECTO. *

La nefropatía poliquística renal del adulto es un trastorno hereditario que se


caracteriza por múltiples quistes expansivos renales bilaterales que finalmente
destruirán el parénquima renal llevando a la insuficiencia.

La nefropatía poliquística renal del adulto es responsable del 5-10% de los casos de
nefropatía terminal que requieren trasplante o diálisis.

En la nefropatía poliquística renal del adulto, los riñones afectados presentan al corte
múltiples quistes de pequeño tamaño que se distribuyen en la corteza y la médula
renal.

Los pacientes con nefropatía poliquística renal también tienen malformaciones


congénitas extrarrenales, presentando el 40% de ellos enfermedad poliquística
hepática, de tipo asintomática y aneurismas saculares del polígono de Willis.

https://docs.google.com/forms/d/e/1FAIpQLSdKi_dZV2bOZn2rwTEhPhlWRrzWZ94_YBeF3iMBprHZ4Z3XrQ/formResponse 24/28
14/12/2020 EXAMEN FINAL 14 DE DICIEMBRE 2020

Una mujer de 40 años presenta debilidad progresiva de los miembros,


hormigueo y entumecimiento; ataxia y pérdida unilateral de la visión durante
unos pocos días. En el examen del líquido cefalorraquídeo se constata leve
aumento de las proteínas, moderado aumento de leucocitos y aumento de la
gamma globulina. La paciente fallece. Se presentan los hallazgos de la necropsia.
El diagnóstico más probable es: *

esclerosis múltiple

enfermedad de Parkinson

enfermedad de Creutzfeld-Jakob

enfermedad de Alzheimer

Paciente de 32 años, concurre a la guardia por presentar un nódulo en la tiroides.


Ud. Decide punzar dicho nódulo con una aguja fina y realizar un estudio
citológico. De esta manera usted puede: *

Diagnosticar cáncer papilar de tiroides.

Diagnosticar un cáncer folicular de tiroides.


https://docs.google.com/forms/d/e/1FAIpQLSdKi_dZV2bOZn2rwTEhPhlWRrzWZ94_YBeF3iMBprHZ4Z3XrQ/formResponse 25/28
14/12/2020 EXAMEN FINAL 14 DE DICIEMBRE 2020
g

Diagnosticar bocio multinodular.

Descartar patología maligna de la glándula.

Un hombre de 24 años refiere disnea progresiva de 10 semanas de evolución. En


la exploración se encuentra afebril. Se identifica matidez a la percusión en la
parte posterior de los pulmones, con atenuación del murmullo vesicular. La
radiografía de tórax muestra derrame pleural bilateral extenso con
ensanchamiento mediastínico. Se realiza una toracocentesis izquierda, que
obtiene 500 cc de líquido blanquecino lechoso. Las pruebas de laboratorio sobre
dicho líquido revelan un alto contenido en proteína y el estudio histológico de la
muestra muchos linfocitos y glóbulos de grasa. ¿Cuál es la causa más probable
de estos hallazgos? *

neumonía bacteriana con empiema

cardiopatía congénita con insuficiencia cardiaca

linfoma no Hodgkin con obstrucción linfática

tuberculosis miliar con pleuritis granulomatosa

https://docs.google.com/forms/d/e/1FAIpQLSdKi_dZV2bOZn2rwTEhPhlWRrzWZ94_YBeF3iMBprHZ4Z3XrQ/formResponse 26/28
14/12/2020 EXAMEN FINAL 14 DE DICIEMBRE 2020

Lesión peribronquial que infiltra el tejido pulmonar, de coloración blancogrisácea,


con áreas de necrosis y hemorragia. El estudio histopatológico reveló playas
sólidas de células atípicas, sin puentes intercelulares ni queratinización.
Inmunohistoquímica: p63 (+), p40 (+) y Ck5 (+). ¿Qué tumor presenta estas
características? *

Carcinoma Epidermoide pobremente diferenciado

Carcinoma Epidermoide bien diferenciado

Adenocarcinoma

Carcinoma de Células Pequeñas

Voltar Enviar

Nunca envie senhas pelo Formulários Google.

Este formulário foi criado em Facultad de Medicina. Denunciar abuso


https://docs.google.com/forms/d/e/1FAIpQLSdKi_dZV2bOZn2rwTEhPhlWRrzWZ94_YBeF3iMBprHZ4Z3XrQ/formResponse 27/28
Micropráctica - Patología Digestiva
*Obligatorio

Instrucciones

RECUERDE:
- En las preguntas tipo múltiple choice, 1 sola opción es la correcta.
- En las preguntas tipo casilla de verificación, deberá identificar todos los enunciados correctos teniendo
en cuenta que el marcar enunciados correctos de menos o enunciados incorrectos, la pregunta se da
automáticamente por anulada.

Una paciente de 28 años consultó por presentar anorexia, pérdida de peso,


adinamia e ictericia de varias semanas de evolución. Se le realizó una TAC en la
que se observó una masa hepática única, de bordes irregulares de
aproximadamente 7cm de diámetro ubicada en el lóbulo hepático derecho con
una marcada cicatriz central. Se realiza resección quirúrgica. Histológicamente
se observaron “células poligonales atípicas bien diferenciadas, que crecen
formando nidos o cordones separados por haces densos de colágeno”. ¿Cuál es
para Ud. el diagnóstico más probable? *

Adenoma hepático

Hiperplasia nodular focal

Colangiocarcinoma

Hepatocarcinoma fibrolamelar

/
En un paciente con epigastralgia se realiza biopsia endoscópica de estómago,
diagnosticándose “Infección por Helicobacter Pylori”. ¿Con cuál de las siguientes
patologías se relaciona esta bacteria? *

Todas las anteriores son correctas

Adenocarcinoma gástrico

Gastritis crónica activa

Linfoma MALT

Paciente de 53 años de edad que se presenta a la consulta por padecer pirosis y


regurgitación ácida de larga data. Se realiza fibroendoscopia digestiva alta en la
cual se observan en los últimos 5cm esofágicos, múltiples áreas de mucosa roja
aterciopelada que alternan con áreas de mucosa blanquecina y homogénea. El
resultado de la biopsia expresa “mucosa de tipo glandular con células
caliciformes e infiltrado mononuclear en la lámina propia”. Con respecto a esta
patología señale lo CORRECTO: *

Para su diagnóstico se requiere la presencia en la endoscopía de mucosa anormal


por encima de la unión gastroesofágica y metaplasia intestinal demostrada
histológicamente.

Su presencia aumenta el riesgo de carcinoma epidermoide esofágico.

Se presenta con frecuencia en menores de 30 años.

La mayoría de los pacientes con este diagnóstico desarrollan tumores esofágicos.

/
En referencia a la "Patología neoplásica del estómago", señale cual/es de los
siguientes enunciados es/son CORRECTO/S. Recuerde que deberá marcar todas
las opciones correctas, considerando que el marcar respuestas correctas de
menos o 1 respuesta incorrecta, dará por anulado el ejercicio. *

El marcador diagnóstico más útil en el diagnóstico de los GIST gástricos es el c-Kit o


CD 117.

En el adenocarcinoma gástrico de tipo intestinal se observan playas de células con


grandes vacuolas citoplasmáticas de mucinas.

El MALToma gástrico es inducido, típicamente, como consecuencia de una gastritis


crónica por H.Pylori.

El adenocarcinoma gástrico representa más del 90% de todos los procesos gástricos
malignos.

Ante el diagnóstico de un adenoma gástrico, el tamaño del mismo no está


relacionado con el riesgo de padecer un adenocarcinoma gástrico.

20% de todos los GIST poseen mutaciones activadoras en las tirosina cinasas KIT o
PDGFRA.

Los tumores carcinoides se caracterizan por presentar una inmunohistoquímica


positiva para sinaptofisina y cromogranina A.

Los factores pronósticos más importantes a considerar en el momento del


diagnóstico son la profundidad de la invasión y las metástasis regionales y a
distancia.

Los tumores gástricos de tipo intestinal tienden a formar tumores voluminosos y


pueden estar ulcerados, mientras que aquellos compuestos por células en anillo de
sello muestran un patrón de crecimiento infiltrante difuso.

/
En referencia a la "Patología esofágica", señale cual/es de los siguientes
enunciados es/son CORRECTO/S. Recuerde que deberá marcar todas las
opciones correctas, considerando que el marcar respuestas correctas de menos
o 1 respuesta incorrecta, dará por anulado el ejercicio. *

El adenocarcinoma se presenta con mayor frecuencia en el 1/3 superior esofágico.

La Enfermedad de Chagas puede generar acalasia por destrucción del plexo


mientérico.

El esófago de Barrett es una complicación del reflujo gastroesofágico crónico que se


caracteriza por metaplasia intestinal con aumento del riesgo de carcinoma
epidermoide esofágico.

Las esofagitis infecciosas se observan con mayor frecuencia en pacientes


inmunodeprimidos siendo cándida, el virus del herpes simple y el CMV los agentes
etiológicos más frecuentemente observados.

El carcinoma epidermoide de esófago afecta a los hombres con una frecuencia 4


veces mayor que a las mujeres.

Los desgarros de Mallory Weiss provocan un desgarro transmural y rotura del


esófago distal.

Los tumores mesenquimatosos esofágicos más frecuentes son los leiomiomas.

Las obstrucciones mecánicas esofágicas se manifiestan por disfagia progresiva.

Paciente de 42 años presentó dolor en hemiabdomen superior irradiado a la


espalda, anorexia, pérdida de peso y marcada astenia. En el examen físico se
aprecia ictericia. En la ecografía abdominal se observó deformación de la silueta
pancreática, con una zona hipoecogénica en su interior. En la biopsia de la
cabeza de páncreas se observaron “células epiteliales con núcleos irregulares,
hipercromáticos y escaso citoplasma, que formaban pseudoglándulas con
invasión perineural, rodeadas de un estroma con marcados cambios
desmoplásicos”. ¿Cuál es para Ud. el diagnóstico más probable? *

Insulinoma

Glucagonoma

Adenocarcinoma pancreático

Adenoma pancreático

/
En referencia a la "Cáncer colorrectal", señale cual/es de los siguientes
enunciados es/son CORRECTO/S. Recuerde que deberá marcar todas las
opciones correctas, considerando que el marcar respuestas correctas de menos
o 1 respuesta incorrecta, dará por anulado el ejercicio. *

Los factores pronósticos más importantes son la profundidad de la invasión y la


presencia o ausencia de MTS a ganglios linfáticos regionales y a distancia.

La mayoría de los adenomas evolucionan a adenocarcinoma.

El tamaño de los adenomas colorrectales es la característica más importante que se


correlaciona con el riesgo de padecer un adenocarcinoma.

La secuencia clásica adenoma-carcinoma es responsable de hasta el 80% de los


tumores de colon esporádicos y se refiere a la mutación de APC al inicio del proceso
neoplásico.

La combinación de episodios moleculares que conducen al adenocarcinoma


esporádico de colon es heterogéneo e incluye anormalidades genéticas y
epigenéticas.

En pacientes con EII (colitis ulcerosa o enfermedad de Crohn) el riesgo de carcinoma


de colon aumenta según la duración y extensión de las lesiones como la intensidad
de la respuesta inflamatoria.

El Síndrome de Lynch es la forma sindrómica hereditaria más frecuente de cáncer de


colon cuya localización preferencial es el colon izquierdo.

Los carcinomas en el colon distal crecen como masas exofíticas polipoideas y como
consecuencia de la distensibilidad de la pared raramente causan obstrucción.

En referencia al “Carcinoma de vesícula biliar” señale cual de las siguientes


opciones es CORRECTA: *

Tumor maligno más frecuente de la vía biliar extrahepática.

El riesgo de padecerlo no se ve incrementado en pacientes con vesícula de porcelana.

El patrón de crecimiento más frecuente es exofítico.

El factor de riesgo más importante es el hábito enólico.

/
Paciente femenino de 16 años que consulta a la guardia del hospital por dolor
abdominal, náuseas, vómitos y fiebre. A la palpación revela dolor a la presión
profunda en fosa ilíaca derecha (signo de Mc. Burney +). Con diagnóstico
presuntivo de apendicitis aguda ingresa a cirugía. Se realiza apendicectomía.
Cuál de las siguientes complicaciones no es frecuente encontrar en esta
patología: *

Absceso hepático

Perforación apendicular

Carcinoma apendicular

Trombosis venosa portal

Un muchacho de 22 años con diarrea crónica presenta en el laboratorio título


elevado de anticuerpos antigliadina y anti transglutaminasa tisular. ¿Cuál de los
siguientes cuadros cree Ud. que puede ser más probable hallar en una biopsia
duodenal de este paciente? *

Atrofia vellositaria con linfocitosis intraepitelial e hiperplasia críptica.

Presencia de Microsporidium en la superficie epitelial.

Acúmulos de histiocitos con elementos PAS (+).

Acumulación densa de macrófagos espumosos en la lámina propia.

Atrás Enviar

Nunca envíes contraseñas a través de Formularios de Google.

Este formulario se creó en Facultad de Medicina. Noti car uso inadecuado

Formularios

/
Micropráctica - Patología Renal y de Vías
Urinarias
*Obligatorio

Instrucciones

RECUERDE:
- En las preguntas tipo múltiple choice, 1 sola opción es la correcta.
- En las preguntas tipo casilla de verificación, deberá identificar todos los enunciados correctos teniendo
en cuenta que el marcar enunciados correctos de menos o enunciados incorrectos, la pregunta se da
automáticamente por anulada.

Que elementos morfológicos son los que permiten justificar el diagnóstico de


Glomerulonefritis Post infecciosa o también llamada Postestreptocócica: *

glomérulos con proliferación mesangial. Inmunofluorescencia positiva para IgA, IgG,


IgM y C3, en mesangio.

glomérulos con áreas de esclerosis, hialinosis focal y segmentaria.


Inmunofluorescencia positiva focal para IgM y C3.

glomérulos aumentados de tamaño, con proliferación difusa de células endoteliales,


mesangiales y epiteliales, con infiltrados polimorfonucleares y algunos monocitos.
Inmunofluorescencia positiva para IgG y C3 granular en mesangio y depósitos
subepiteliales grandes en forma de joroba en la microscopía electrónica.

glomérulos con engrosamiento difuso de las paredes capilares. Imnunofluorescencia


positiva para IgG y C3 granular con patrón difuso intramembranoso.

/
En relación al carcinoma de células renales, marque la opción INCORRECTA: *

Los carcinomas papilares pueden ser multifocales y bilaterales, hemorrágicos y


quísticos, constituídos por células cúbicas bajas, células espumosas intersticiales y
cuerpos de psamomma.

Los carcinomas cromófobos estan formados por células eosinófilas pálidas con un
halo claro perinuclear, dispuestas en láminas sólidas, de forma que las células más
grandes se concentran alrededor de los vasos sanguíneos.

Los carcinomas de células claras surgen del epitelio tubular proximal y se presentan
como lesiones unilaterales únicas, predominantemente en relación a un patrón
hereditario y de tipo familiar.

Los carcinomas del conducto colector constituyen una variante con formación de
canales irregulares recubiertos por un epitelio intensamente atípico con un patrón
morfológico en tachuela.

Respecto al papiloma urotelial invertido, señale la opción CORRECTA. *

Generalmente se presenta en adultos jóvenes como lesiones múltiples.

Consiste en una evaginación del epitelio, sin papilas y mínimo estroma.

Está conformado por células ovales o fusiformes sin signos de atipía.

Su presencia excluye la presencia de un carcinoma urotelial.

/
En referencia a la "Patología Vesical", señale cual/es de los siguientes enunciados
es/son CORRECTO/S. Recuerde que deberá marcar todas las opciones correctas,
considerando que el marcar respuestas correctas de menos o 1 respuesta
incorrecta, dará por anulado el ejercicio. *

La cistitis aguda es una entidad no relacionada con la pielonefritis bacteriana.

El 95% de los tumores vesicales poseen origen mesenquimatoso.

Las cistitis agudas se caracterizan macro y microscópicamente por presentar una


mucosa hiperémica e infiltrado de neutrófilos con o sin exudado.

En el desarrollo del carcinoma vesical existe una fuerte asociación con el


tabaquismo.

En los carcinomas uroteliales la invasión de la muscular de la mucosa tiene


importancia pronóstica.

Las lesiones precursoras más frecuentes del carcinoma urotelial invasivo son los
tumores papilares no invasivos que se originan a partir de la hiperplasia papilar
urotelial.

La anomalía congénita más frecuente y grave es el reflujo vesicoureteral.

Los carcinomas uroteliales papilares de bajo grado se caracterizan por presentar un


patrón histoarquitectural conservado con escasa atipia nuclear e infrecuentes
mitosis. No obstante, presentan mayor incidencia de invasión de la capa muscular
por lo cual son de mal pronóstico.

Paciente femenina de 43 años de edad con antecedente de infección de vías


urinarias bajas, diabética. Presenta fiebre, dolor costal, disuria. La paciente se
descompensa por su estado diabético y fallece. En la autopsia al examen
macroscopico del riñón, este, evidencia superficie con abscesos delimitados y
amarillentos, microscópicamente se ve necrosis licuefactiva que afecta el
parénquima renal, abscesos y neutrofilos intratubulares. La descripción es
compatible con: *

Pielonefritis aguda

Carcinoma de células renales

Necrosis tubular aguda

Intoxicación por AINES

/
¿Que esperaría observar macroscópicamente en el Carcinoma Renal? *

masas esféricas de 3 a 15 cm de diámetro, habitualmente en un polo, bien


delimitadas, que deforman la silueta renal, de coloración amarillenta, con áreas
reblandecidas opacas blanco-grisáceas y focos de hemorragia.

masas de 5 a 10 cm de diámetro, blanquecinas, fibrosas, de límites poco netos, con


pequeños focos de hemorragia.

nódulo pequeño, bien delimitado, homogéneo, de coloración de coloración


amarronada.

quiste de paredes gruesas, tabicado, revestido por papilas finas y friables que se
desprenden con facilidad.

Un hombre de 58 años, originario de Beijing ha observado que su orina es más


oscura desde hace 3 semanas. La exploración física no revela alteraciones. El
análisis de orina muestra presencia de sangre. Se realiza una cistoscopia y se
identifica una zona rojiza de 0,5 x 1,7 cm en la cúpula vesical. Se obtienen
biopsias e histológicamente se observa una proliferación celular de tipo papilar,
constituida por células uroteliales atípicas de núcleos irregulares, con marcada
anisocariosis y citoplasma eosinófilo que no sobrepasa la membrana basal. ¿Cuál
es el factor de riesgo más probable para esta enfermedad? *

esquitosomiasis

mutación génica hereditaria

obesidad

tabaquismo

/
En referencia a las "Glomerulopatías", señale cual/es de los siguientes enunciados
es/son CORRECTO/S. Recuerde que deberá marcar todas las opciones correctas,
considerando que el marcar respuestas correctas de menos o 1 respuesta
incorrecta, dará por anulado el ejercicio. *

La GNRP de tipo II, se define por la ausencia de anticuerpos antimembrana basal


glomerular o inmunocomplejos en la inmunofluorescencia y al microscopio
electrónico.

En el Síndrome de Goodpasture los anticuerpos antimembrana basal reaccionan de


manera cruzada con otras membranas basales y pueden generar patología
simultáneamente en pulmón y riñón.

El síndrome nefrítico es causado por lesión glomerular y está caracterizado por


hematuria, proteinuria e HTA.

La mayoría de las glomerulopatías se producen por mecanismos inmunitarios


mientras que las patologías tubulares e intersticiales se deben con frecuencia a
agentes tóxicos o infecciosos.

El síndrome nefrótico se caracteriza por proteinuria >3,5gr/día, hipoalbuminemia,


hiperlipidemia y edema.

La enfermedad mediada por anticuerpos antimembrana basal glomerular se


caracteriza por depósitos lineales de IgG y, en muchos casos de C3 en la membrana
basal glomerular.

Los anticuerpos antimembrana basal provocan una nefropatía a cambios mínimos de


excelente pronóstico.

La glomerulonefritis posinfecciosa aguda suele desarrollarse tras una infección


estreptocóccica en niños y adultos jóvenes caracterizada por depósitos de
inmunocomplejos subepiteliales con abundantes neutrófilos y proliferación celular
glomerular.

Con respecto a la Patología de riñón indique la opción CORRECTA: *

La glomerulonefritis membranoproliferativa se caracteriza por depósitos


electrodensos a nivel del mesangio.

La pielonefritis aguda presenta abundante infiltrado inflamatorio linfocitario a nivel de


los túbulos.

La pielonefritis crónica es un trastorno en el que la inflamación tubulointersticial


crónica y la cicatrización afectan calices y pelvis renal.

En el Síndrome de Alport se observa intensa afectación de los glomérulos.

/
En referencia a las "Neoplasias Renales", señale cual/es de los siguientes
enunciados es/son CORRECTO/S. Recuerde que deberá marcar todas las
opciones correctas, considerando que el marcar respuestas correctas de menos
o 1 respuesta incorrecta, dará por anulado el ejercicio. *

El carcinoma cromófobo renal es uno de los menos frecuentes y se encuentra


formado por células eosinófilas pálidas cuyas membranas celulares son prominentes
y a menudo con un halo perinuclear.

En el 10% de los pacientes el carcinoma renal se presenta con dolor costovertebral,


masa palpable y hematuria.

A pesar de su constitución vascular, el angiomiolipoma no tiene tendencia a la


hemorragia espontánea.

El adenoma papilar se caracteriza por estar constituido por células atípicas con
diferentes grados de disqueratosis.

El carcinoma de células claras renal es el tumor maligno más frecuente del riñón.
Suele presentarse como lesiones solitarias unilaterales y se origina del túbulo
contorneado proximal.

Una característica particularmente problemática de los carcinomas renales es su


tendencia a metastatizar difusamente antes de dar lugar a síntomas y signos locales.

Los tumores de la pelvis renal suelen dar signos tardíos y generalmente han generado
MTS a distancia al momento del diagnóstico.

Todos los adenomas, con independencia de su tamaño, son potencialmente


malignos.

Atrás Enviar

Nunca envíes contraseñas a través de Formularios de Google.

Este formulario se creó en Facultad de Medicina. Noti car uso inadecuado

Formularios

/
Micropráctica - Patología del Aparato
Genital Masculino
-Deberán contar con 4 preguntas correctas para poder aprobar el ejercicio.
-Las inquietudes que pudieran surgir durante la realización o al finalizar la micropráctica,
podrán evacuarlas con sus respectivos docentes.
- Las microprácticas no enviadas se registran como ausentes.
- Deberán registrarlas con su nombre/s y apellido/s tal como figuran en los listados de
alumnos asignados por el Departamento de Informática de Fmed. Caso contrario la
micropráctica será anulada.
- Si se registraran 2 microprácticas bajo la misma identidad, se tomará en cuenta la 1° que
haya sido enviada y registrada por el sistema.

*Obligatorio

Dirección de correo electrónico *

En referencia a las "Adenocarcinoma de Próstata", señale cual/es de los


siguientes enunciados es/son CORRECTO/S. Recuerde que deberá marcar todas
las opciones correctas, considerando que el marcar respuestas correctas de
menos o 1 respuesta incorrecta, dará por anulado el ejercicio. *

El cáncer de próstata presenta pérdida de E-cadherina al igual que el carcinoma


invasivo de células en anillo de sello gástrico y el carcinoma lobulillar de mama.

La gradación según el sistema de Gleason es importante en el cáncer de próstata, ya


que el grado y el estadío son los mejores factores predictivos del pronóstico.

Los andrógenos tienen un papel importante en el desarrollo de esta patología.

La diseminación hematógena se da fundamentalmente a huesos del esqueleto axial


generando metástasis típicamente osteoblásticas

Histológicamente se caracteriza por proliferación de glándulas, algunas de las cuales


presentan dilataciones quísticas, recubiertas con dos capas celulares, con
proliferación fibrosa o muscular del estroma.

En la gran mayoría de los casos surge en la zona central a nivel periuretral.

La determinación sérica de PSA total por sí sola, es diagnóstico de cáncer de


próstata.

El hallazgo de PIN de alto grado puede estar asociado con adenocarcinoma (hasta en
un 80% de los casos)

DNI *

/
Marque la opción CORRECTA respecto del Adenocarcinoma de próstata: *

En el examen microscópico muestra glándulas revestidas por múltiples capas de


células atípicas.

Las metástasis óseas son de tipo osteolítico.

Se origina más frecuentemente en la zona periférica y posterior de la glándula.

Su diseminación se produce exclusivamente por vía hematógena.

Un paciente de 60 años de edad consultó por dificultad miccional, palpándose la


próstata difusamente aumentada de tamaño, de consistencia firme-elástica.
PSA:7ng/ml. Se le realizó punción biopsia transrectal observándose “glándulas
cuyo epitelio muestra cambios reactivos, con abundantes linfocitos, macrófagos
y plasmocitos, rodeadas de un estroma fibromuscular”. ¿Cuál es para Ud. el
diagnóstico más probable? *

Prostatitis aguda

Hiperplasia nodular

Prostatitis crónica

Neoplasia intraepitelial

Paciente de 65 años, con antecedentes de fimosis, presenta una lesión ulcerada


e indurada de 1,5cm en la superficie interna del prepucio, cerca del surco
coronal. Asimismo, se palpan ganglios inguinales. Se realizó una biopsia,
observándose “Proliferación de células epiteliales atípicas con marcada
anisocariosis, mitosis atípicas, hipercromasia, cromatina irregular y nucleólo
evidente, citoplasma eosinófilo con focos de disqueratosis y aisladas perlas
córneas que infiltran la membrana basal”. ¿Cuál es para Ud. el diagnóstico más
probable? *

Papulosis bowenoide

Enfermedad de Bowen

Eritroplasia de Queyrat

Carcinoma epidermoide

Nombre/s *

/
En referencia a la entidad nosológica “Tumores Seminomatosos”, indique la
afirmación INCORRECTA: *

El seminoma clásico es el tumor más frecuente de testículo y predomina en adultos


de la 2da a 3ra década de la vida.

En los tumores mixtos puede combinarse con el coriocarcinoma.

A pesar de ser quimiosensible tiene mal pronóstico por su diseminación precoz.

15% de los seminomas son GCH + por la presencia de células trofoblásticas


intratumorales.

En referencia a las "Patología Neoplásica Testicular", señale cual/es de los


siguientes enunciados es/son CORRECTO/S. Recuerde que deberá marcar todas
las opciones correctas, considerando que el marcar respuestas correctas de
menos o 1 respuesta incorrecta, dará por anulado el ejercicio. *

Todos los seminomas presentan HCG elevada.

A diferencia de otros tumores malignos de la economía, la mayoría de los tumores


malignos testiculares de células germinales no se originan de una lesión precursora.

La mayoría de los tumores de las células de Sértoli son hormonalmente silentes y se


presentan como una masa testicular.

El tumor testicular más frecuente en menores de 3 años de edad es el tumor del seno
endodérmico.

Los coriocarcinomas son tumores pequeños que no suelen provocar el aumento del
tamaño testicular y también pueden observarse en el aparato genital femenino.

El tumor maligno paratesticular más frecuente en adultos es el liposarcoma.

La mayoría de los tumores de células germinales del testículo son agresivos, capaces
de una diseminación rápida y extensa.

El linfoma difuso de linfocitos B grandes (LDCGB) debe ser considerarse un


diagnóstico diferencial ante las presencia de una masa testicular en pacientes
mayores de 60 años.

/
Un paciente de 68 años de edad, con trastornos miccionales y diagnóstico de
Hiperplasia fibroadenomatosa de la próstata, es intervenido quirúrgicamente
resecándose varios fragmentos irregulares de tejido blanquecino con sectores
de aspecto criboso que en conjunto pesan 140gr. ¿Cuáles serían para Ud. los
elementos histológicos que permiten justificar el diagnóstico? *

Proliferación de glándulas pequeñas con una sola capa de células y núcleos grandes
vacuolados con nucléolo prominente con escaso estroma interpuesto.

Proliferación de glándulas, algunas de las cuales presentan dilataciones quísticas,


recubiertas con dos capas celulares, con proliferación fibrosa o muscular del
estroma.

Proliferación de células epiteliales que forman, en sectores, glándulas y en otros


cordones comprimidos por el aumento del número de células del estroma.

Proliferación de glándulas de aspecto criboso, con una sola capa de células,


rodeadas de estroma fibromuscular con cambios desmoplásicos.

En referencia a la “Patología tumoral peneana”, señale el enunciado CORRECTO:


*

Tanto la enfermedad de Bowen y la papulosis bowenoide como representantes del


carcinoma in situ peneano comparten su importante asociación con la infección por
el HPV (16).

El carcinoma verrugoso es una variante endofítica indiferenciada del carcinoma


epidermoide.

<de 1,5cm de invasión en profundidad no tiene riesgo de metástasis.

El tabaquismo y la acumulación de esmegma por mal hábito higiénico, no se hallan


relacionados con el riesgo de desarrollar carcinoma peneano.

NÚMERO DE LA COMISIÓN EN LA QUE ESTÁ CURSANDO (1, 2, 3, 4, 5, 6, 7, 8) *

/
Por lo general, los tumores testiculares de células germinales se asocian a un
espectro de trastornos que se conocen en conjunto como Sindrome de
disgenesia testicular. ¿Cuál de los siguientes trastornos forman parte de este
sindrome? *

Esperma de baja calidad

Criptorquidia

Todas las anteriores

Hipospadia

Apellido/s *

Que elementos morfológicos son los que permiten justificar el diagnóstico de


Seminoma: *

Macroscópicamente: estructura nodular pequeña, con bordes mal definidos y


extensas áreas de hemorragia y necrosis. Microscópicamente: células epiteliales
formando láminas irregulares, con grandes núcleos hipercromáticos.

Macroscópicamente: estructura nodular de gran tamaño, de aspecto heterogéneo con


áreas sólidas y otras quísticas. Microscópicamente: células neurales, musculares,
cartilaginosas y de diferentes tejidos maduros inmersas en un estroma fibroso.

Macroscópicamente: estructura nodular de aspecto sólido, blanco-grisácea,


homogénea, delimitada, con tractos blanquecinos. Microscópicamente: células
uniformemente grandes de núcleo redondo, nucléolo prominente y citoplasma claro,
septos con infiltrado de linfocitos.

Macroscópicamente: estructura nodular pequeña de aspecto mucinoso, homogéneo y


amarillento. Microscópicamente: células neoplásicas cúbicas dispuestas en red.

Enviar /
Micropráctica - Patología Hemolinfoide
*Obligatorio

Instrucciones

RECUERDE:
- En las preguntas tipo múltiple choice, 1 sola opción es la correcta.
- En las preguntas tipo casilla de verificación, deberá identificar todos los enunciados correctos teniendo
en cuenta que el marcar enunciados correctos de menos o enunciados incorrectos, la pregunta se da
automáticamente por anulada.

En referencia a la entidad “Linfoma de Hodgkin”, señale la opción 1 punto


CORRECTA: *

Se describe a la célula de Reed Sternberg típica como una célula pequeña


mononuclear y citoplasma granular.

El tipo “Celularidad Mixta” es el menos frecuente de los tipos de Linfoma Hodgkin y


no se ha encontrado asociación con el VEB.

En el tipo “Predominio Linfocitario” se observa el 90% de positividad para VEB.

El tipo “Esclerosis Nodular” es el más frecuente de los Linfomas Hodgkin y se


caracteriza por presentar histológicamente nódulos circunscriptos por bandas
colágenas, células lacunares y expresa CD15, CD30 Y PAX5.

En referencia a la entidad “Linfadenitis”, señale la opción INCORRECTA: * 1 punto

La hiperplasia folicular se debe a estímulos que activan la respuesta inmune humoral


y morfológicamente deberá diferenciarse del linfoma folicular.

La linfadenitis crónica inespecífica puede generar diversos patrones morfológicos


como la hiperplasia folicular, hiperplasia paracortical y la histiocitosis sinusal.

Las adenomegalias inducidas por infecciones piógenas en ocasiones dejan cicatrices


residuales.

La histiocitosis sinusal es particularmente prominente en los ganglios linfáticos en el


curso de las infecciones virales.

La fase leucémica del linfoma cutáneo de células T se llama: * 1 punto

Micosis fungoide

Leucemia de células T adultas

Síndrome de Sezary

Leucemia de células pilosas (tricoleucemia)

/
Paciente masculino de 54 años, consulta por episodios de fatiga y anorexia 1 punto
de 4 meses de evolución que se agravó en el último tiempo. Refiere
también haber perdido (8kg en 2 meses) peso pero lo asocia más que
nada a la falta de apetito. El examen físico revela dos adenopatías a nivel
inguinal bilateral y otras dos en hueco axilar izquierdo. Trae consigo una
Tomografía de abdomen y pelvis que evidencia hepatomegalia franca. El
laboratorio informa GB: 22000; Hto: 43; Hb: 15. Formula leucocitaria
N:7260, L:13740, M:750, E:100, B:150. ¿Qué espera encontrar en la biopsia
de una de las adenopatías? *

Patrón nodular en donde se distinguen dos tipos de células bien marcadas, células
hendidas pequeñas y otras mayores con cromatina laxa y varios nucléolos.

Infiltrado linfocitario denso que infiltra el ganglio en su totalidad, de células pequeñas


con cromatina densa y citoplasma escaso y centros de proliferación con células
mitóticamente activas.

Hiperplasia paracortical con abundantes linfocitos de tamaño mayor al normal con


núcleos redondeados con cromatina abierta y varios nucléolos prominentes.

Grandes centros germinales reactivos prominentes que contienen numerosas figuras


mitóticas y abundantes macrófagos y polimorfonucleares.

El cromosoma Filadelfia se caracteriza por: * 1 punto

t (8; 14)

t (14; 8)

t (22; 9)

t (9; 22)

¿Cuál de los siguientes trastornos NO se asocia a esplenomegalia? * 1 punto

Insuficiencia cardíaca izquierda

Trombosis de la vena porta

Cirrosis hepática

Anemias Hemolíticas

/
Paciente de 58 años de edad que presenta anemia, neutropenia, 1 punto
leucocitosis, hemorragias, fiebre, espleno/hepatonmegalia, cuadro de
evolución aproximada desde hace 4 meses. Microscópicamente presencia
de blastos mieloides en médula osea +20%, citoplasma con cuerpos de
Auer, usted pensaría en: *

Mielodisplasia

Linfoma de células del manto

Leucemia mieloide aguda (LMA)

Mieloma múltiple

¿Cuál de los siguientes es un linfoma no Hodgkin indolente (de bajo 1 punto


grado)? *

Linfoma de Burkitt.

Linfoma folicular.

Mieloma múltiple.

Linfoma difuso de células B grandes.

En referencia al "Linfoma folicular", indique la opción INCORRECTA: * 1 punto

Suele presentarse con linfadenopatías generalizadas e indoloras

El linfoma folicular se origina en linfocitos B del centro germinal.

Presenta células linfoides de gran tamaño.

Puede afectar los testículos.

/
En referencia a los “Linfomas”, señale cuales de los siguientes enunciados 1 punto
es CORRECTO. Recuerde que deberá marcar todas las opciones correctas,
considerando que el marcar opciones correctas de menos o 1 respuesta
incorrecta, dará por anulado el ejercicio. *

El linfoma difuso de células grandes B y el linfoma folicular son los más frecuentes
dentro del grupo de los Linfomas no Hodgkin.

El linfoma folicular se caracteriza por no expresar BCL 2.

La leucemia linfocítica crónica y el linfoma linfocítico de células pequeñas difieren en


el grado de linfocitosis en sangre periférica.

El Linfoma de células grandes B (LDCGB) es un Linfoma no Hodgkin de alto grado que


histológicamente presenta un patrón de crecimiento difuso cuyas células tumorales
expresan habitualmente CD19 y CD20. Tumor agresivo.

Los MALTomas son linfomas de la zona marginal extranodal del tejido linfoide
asociado a mucosas. En el caso de MALToma gástrico se demostró su asociación a
infección por Helicobacter Pylori.

Sólo el Linfoma de Burkitt asociado al HIV presenta translocación del gen MYC.

La mayoría de las neoplasias linfoides se originan de Linfocitos B y la mayor parte de


las restantes de Linfocitos T. Sólo en escasas oportunidades se originan de
Linfocitos NK.

Atrás Enviar

Nunca envíes contraseñas a través de Formularios de Google.

Este formulario se creó en Facultad de Medicina. Noti car uso inadecuado

Formularios

/
Micropráctica - Neuropatología
*Obligatorio

Instrucciones

RECUERDE:
- En las preguntas tipo múltiple choice, 1 sola opción es la correcta.
- En las preguntas tipo casilla de verificación, deberá identificar todos los enunciados correctos teniendo
en cuenta que el marcar enunciados correctos de menos o enunciados incorrectos, la pregunta se da
automáticamente por anulada.

En referencia a las “Enfermedades Neurodegenerativas”, señale el 1 punto


enunciado INCORRECTO: *

La causa más frecuente de demencia es la Enfermedad de Parkinson.

La Enfermedad de Alzheimer se caracteriza a nivel patológico por presentar en su


fase final, placas neuríticas y ovillos neurofibrilares.

El hallazgo característico de la Enfermedad de Parkinson es la despigmentación de la


substancia nigra y del locus coeruleus y presencia de cuerpos de Lewy.

El proceso patológico común a la mayor parte de las enfermedades


neurodegenerativas es la acumulación de agregados de proteínas diferenciándose en
la localización anatómica y en otras alteraciones celulares específicas.

Un paciente de 2 años presentó desviación ocular (estrabismo), leucocoria 1 punto

y una formación tumoral en el examen del fondo de ojo. Se realizó la


enucleación, observándose una masa tumoral ocupante de la cavidad
vítrea con adhesión a la pared ocular, compuesta por células pequeñas
hipercromáticas y de escaso citoplasma, que formaban rosetas, con
amplias áreas de necrosis. ¿Cuál es para Ud. el diagnóstico más probable?
*

Retinoblastoma

Persistencia del vítreo primario hiperplásico

Fibroplasia retrolental

Rabdomiosarcoma
En referencia a la “Patología Vascular del Sistema Nervioso Central”, señale 1 punto
el enunciado INCORRECTO: *

La rotura de un aneurisma sacular (en fresa) en una arteria cerebral es la causa más
frecuente de hemorragia subaracnoidea.

La demencia vascular multiinfarto puede deberse a: ateroesclerosis cerebral,


trombosis o embolización o por lo efectos de la hipertensión arterial a nivel arteriolar
cerebral.

Las hemorragias intraparenquimatosas primarias se deben habitualmente a


hipertensión o a angiopatía amiloide cerebral.

Las telangiectasias capilares son las malformaciones vasculares que se asocian a


mayor riesgo de hemorragia y aparición de síntomas neurológicos.

Un muchacho de 18 años presenta hipoacusia, zumbidos y comienza con 1 punto


vértigo; en la TAC se observa una lesión ocupante de espacio en el ángulo
pontocerebeloso en relación con el VIII par. Histológicamente se observan
“células ahusadas dispuestas con dos patrones, unos más celulares con
cuerpos de Verocay y otros hipocelulares (áreas de “Antoni A” y “Antoni
B”). El estudio inmunofenotípico de las células neoplásicas demuestra
proteína S100 (+).” ¿Cuál es para Ud. el diagnóstico más probable? *

Astrocitoma

Meningioma

Schwannoma

Neurofibrosarcoma

Un paciente de 23 años de edad, inmunodeprimido y con otitis media 1 punto


aguda presenta alteraciones de la conducta, convulsiones y fiebre alta de
48hs de evolución. Se solicita una RMN con contraste en la que se observa
una zona redondeada hipodensa rodeada de un halo hipercaptante en
lóbulo frontal. Se le solicita una biopsia estereotáxica de la lesión.
Histológicamente se observa “abundante material necrótico, densos
infiltrados de tipo polimorfonuclear y gliosis reactiva”. ¿Cuál es para Ud. el
diagnóstico más probable? *

Absceso piógeno

Tuberculoma

Glioblastoma

Quiste coloide
Mujer de 34 años de edad, sin antecedentes médicos relevantes, 1 punto
entrenando en el gimnasio, sufre una brusca y fuerte cefalea y
posteriormente pierde el conocimiento. En la guardia le realizan una TAC
que revela extensa hemorragia subaracnoidea en la base del cerebro.
¿Cuál de las siguientes cree que ha sido la causa? *

ruptura de aneurisma congénito del polígono de Willis

hemorragia hipertensiva

ruptura de la arteria meníngea media

como consecuencia de una angiopatía amiloide

Paciente femenino de 76 años de edad con antecedentes de una 1 punto


enfermedad neurodegenerativa caracterizada por rigidez progresiva del
tronco, desequilibrio postural con frecuentes caídas y dificultad para los
movimientos oculares voluntarios, fallece en un geriátrico. La autopsia
revela una amplia pérdida neuronal en globos pálidos, sustancia nigra,
sustancia gris peri-acueductal, colículos y núcleos dentados del cerebelo,
donde se observan ovillos fibrilares globosos tanto en las neuronas como
en las células gliales. El análisis ultraestructural revela filamentos rectos de
15 nm constituídos por 4R tau. ¿Cuál es su sospecha diagnóstica? *

Creutzfeld-Jakob

Parálisis supranuclear progresiva

Parkinson

Alzheimer

En referencia a la Patología del SNC, señale la opción INCORRECTA * 1 punto

El astrocitoma pilocítico suele presentarse en ancianos.

Los glioblastomas multiformes se presentan como masas infiltrantes, hemorrágicas


y necróticas.

En el glioblastoma las células tumorales se agrupan alrededor de áreas necróticas


dando imágenes en "pseudoempalizada".

Los glioblastomas multiformes presentan histología similar a los astrocitomas


anaplásicos y se les agregar necrosis proliferación vascular o de células endoteliales.
En referencia a los “Tumores del SNC”, señale cuales de los siguientes 1 punto
enunciados es CORRECTO. Recuerde que deberá marcar todas las
opciones correctas, considerando que el marcar opciones correctas de
menos o 1 respuesta incorrecta, dará por anulado el ejercicio. *

Se denominan ependimomas mixopapilares a los tumores ubicados en el filum


terminale de la médula y que contienen elementos papilares sobre un fondo mixoide.

El glioblastoma multiforme se caracteriza por presentar áreas de necrosis y


proliferación vascular.

El meduloblastoma aparece predominantemente en niños y en cerebelo.

Las MTS cerebrales más frecuentes provienen de pulmón, mama, piel (melanoma),
riñón y aparato digestivo.

En el oligodendroglioma la presencia de mutaciones IDH1, IDH2 se asocia a mejor


pronóstico.

La neoplasia más frecuente en SNC de pacientes inmunodeprimidos es el Linfoma


difuso de grandes células B.

El glioblastoma se caracteriza por presentar células bipolares con prolongaciones


largas y finas que son GFAP (+) acompañadas de fibras de Rosenthal y cuerpos
granulares eosinófilos.

Los gliomas (astrocitomas, oligodendrogliomas y ependimomas) conforman el grupo


más frecuente de tumores cerebrales primarios.

Los oligodendrogliomas anaplásicos con codeleción 1p/19q tienen menor respuesta


a la quimio y radioterapia y consecuentemente peor pronóstico.

¿Cuál astrocitoma es de grado I/IV de la Organización Mundial de la Salud? 1 punto


*

Glioblastoma.

Astrocitoma pilocítico.

Astrocitoma difuso.

Astrocitoma anaplásico.

/
Micropráctica - Patología Dermatológica,
Osteoarticular y Partes Blandas
*Obligatorio

Instrucciones
Las preguntas se presentan en diferentes formatos:
* múltiple choice (sólo 1 opción es considerada correcta o incorrecta)
* casillas de verificación (deberá marcar TODAS las opciones correctas, considerando que el marcar 1 respuesta
incorrecta, dará por anulado el ejercicio)

Un paciente de 70 años de edad, de piel blanca, trabajador rural, consultó por presentar una
lesión de rápido crecimiento en la frente. En el examen físico se observó un nódulo de color rojo
violáceo, con bordes sobreelevados y con una umbilicación central, que mide 1cm de diámetro.
Se lo extirpó observándose un cráter central relleno de queratina, rodeado por una proliferación
de células epiteliales grandes con anisocariosis y nucleolo evidente ubicadas a los lados del
cráter y en profundidad, contactando con la dermis. ¿Cuál de las siguientes opciones es
CORRECTA? *
a) La macroscopía y la histología descripta son diagnósticas de carcinoma basocelular
b) La patología descripta debe ser considerada como diagnóstico diferencial de carcinoma epidermoide
c) La patología descripta es un carcinoma epidermoide pobremente diferenciado
d) La patología descripta debe ser considerada una lesión melánica de tipo preneoplásica

Una paciente de 60 años tuvo molestias abdominales difusas y pérdida de 12kg de peso en 2
meses. En la TAC abdominal se observa una gran masa retroperitoneal, que comprimía y
desplazaba las vísceras abdominales. “Macroscópicamente, era una masa lobulada,
pseudocapsulada de 25x15cm de coloración blancoamarillenta, e histológicamente mostraba
una proliferación de células redondas, con citoplasma claro vacuolado y núcleos ovoideos
periféricos anisocarióticos, hipercromáticos”. Tinción con Sudán(+). Inmunomarcación: Proteína
S100(+), Actina y Desmina(-)”.¿Cuál de las siguientes opciones es CORRECTA para esta
entidad nosológica? *
a) La variante histológica pleomórfica es la más frecuentemente observada.
b) Es uno de los sarcomas más frecuentes de la infancia
c) La velocidad de aparición de las metástasis no está en relación con el tamaño, la localización
(superficial – profundo) o el grado histológico tumoral.
d) Sus diferentes tipos no constituyen una enfermedad uniforme, por tanto, el conocimiento de sus
alteraciones moleculares permitirá el diagnóstico y el tratamiento más adecuado.

En relación a la patología presentada en la imagen, señale la


opción CORRECTA: *

a) La neoformación observada cuya coloración y consistencia es


heterogènea e invade el canal medular y partes blandas puede considerarse
una lesión de tipo degenerativa.
b) La observación macroscópica y ubicación predominantemente epifisaria
permite confirmar que se trata de un tumor de células gigantes.
c) La producción de sustancia osteoide es característica de esta
patología.
d) Se trata de una patología con excelente pronóstico (>90% sobrevive a
los 5 años)

Identifique cuál de las siguientes afirmaciones es CORRECTA: *


a) El nevo nevocelular está producido en un 25% por HPV, virus papiloma.
b) El melanoma in situ produce metástasis en el 25% al ganglio centinela.
c) La queratosis actínica está relacionada con el receptor 3, del factor de crecimiento de fibroblastos
(FGFr3)
d) Los nevos nevocelulares son causados en su mayoría por mutación en BRAF o RAS

En referencia a la entidad "Carcinoma epidermoide", señale cual/es de los siguientes


enunciados es correcto. Recuerde que deberá marcar todas las opciones correctas,
considerando que el marcar 1 respuesta incorrecta, dará por anulado el ejercicio *
a) La macroscopía correspondiente a un carcinoma in situ de piel se caracteriza por placas
descamativas, eritematosas y bien delimitadas
b) Se trata de un tipo de tumor mesenquimático.
c) La presencia de perlas córneas y puentes intercelulares permite graduar la diferenciación histológica
d) La piel adyacente a un carcinoma infiltrante puede mostrar áreas de carcinoma in situ o de
queratosis actínica
e) Cuando se observa a la microscopía la presencia de perlas córneas y puentes intercelulares puede
inferirse que se trata de un carcinoma epidermoide bien diferenciado
f) Fundamentalmente la diseminación de este tipo de tumores es por vía hematógena.
g) La queilitis actínica se considera una lesión preneoplásica
h) La queratosis seborreica está asociada a mutación de la P53 y a rayos UV.
i) El queratoacantoma es un diagnóstico diferencial a tener presente

Paciente de sexo masculino de 45 años de edad, que presenta una lesión pigmentada de tipo
nodular en la pierna. La misma es extirpada y examinada por un patólogo, quien le comenta al
dermatólogo que “se trata de un melanoma extensivo superficial en fase de crecimiento
vertical”. ¿Cuál es para Ud. el factor pronóstico más preciso en este caso? *
a) La edad de la paciente
b) Índice de Breslow
c) La topografía de la lesión
d) El infiltrado linfocitario

Mujer de 29 años a la cual se le realiza una amputación del fémur proximal. Usted recibe la
pieza quirúrgica y observa una neoformación hemorrágica rojizo-negruzca en la región
epifisaria. En el estudio histopatológico se observa proliferación de células mononucleares
ovales uniformes y numerosas células gigantes de tipo osteoclasto. ¿Cuál es para Ud. el
diagnóstico más probable? *

a) Condrosarcoma
b) Tumor de células gigantes
c) Osteosarcoma
d) Sarcoma de Ewing

Paciente de sexo masculino de 43 años de edad que presenta placas de color salmón-rosado
bien delimitadas cubiertas por escamas blanquecinas plateadas poco adherentes. Las mismas
se encuentran ubicadas preferentemente en codos, rodillas y cuero cabelludo. Se toma biopsia.
¿Cuál es para usted entre las siguientes la descripción histológica más apropiada para este tipo
de lesiones? *

a) acantosis regular de las crestas interpapilares, paraqueratosis, estrato granuloso adelgazado y


pequeños acúmulos de neutrófilos intracórneos. En la dermis papilar existe edema, vasodilatación e
infiltración linfocitaria perivascular.
b) epidermis con elongación y aguzamiento de red de crestas, en “diente de sierra”, con escamatización
de la capa basal, presencia de cuerpos de Civatte, y un infiltrado inflamatorio crónico.
c) hiperqueratosis, acantosis y proliferación intraepidérmica de pequeñas células basaloides típicas,
muchas de ellas pigmentadas, que en sectores encierran quistes córneos.
d) Incremento del estrato córneo engrosado y compactado, sin evidencias histológicas de inflamación.

En referencia a la entidad "Osteoartropatía degenerativa", señale cual/es de los siguientes


enunciados es correcto. Recuerde que deberá marcar todas las opciones correctas,
considerando que el marcar 1 respuesta incorrecta, dará por anulado el ejercicio *
a) La osteoartritis o enfermedad degenerativa articular, es el tipo más frecuente de enfermedad articular
b) Se caracteriza por la degeneración del cartílago articular que provoca un fracaso estructural y
funcional de las articulaciones sinoviales
c) La mayoría de los casos aparece en individuos jóvenes con factores predisponentes
d) La injuria de los condrocitos se halla relacionada con el envejecimiento, factores genéticos y
bioquímicos
e) Las lesiones histológicas se caracterizan por despulimiento y efracción del cartílago articular,
reabsorción y neoformación ósea reactiva subcondral
f) La fase temprana de la enfermedad se caracteriza por la pérdida de condrocitos y de cartílago con
cambios extensos en el hueso subcondral
g) Los osteofitos de las articulaciones interfalángicas proximales y distales se denominan
respectivamente "nódulos de Heberden" y "nódulos de Bouchard". Siendo los primeros más
frecuentes en hombres.

Teniendo en cuenta los factores pronósticos para los sarcomas de partes blandas ¿Cuáles de
los siguientes parámetros considera más importantes para establecer el grado histológico en
este tipo de tumores? *
a) Permeación vascular, necrosis y mitosis
b) Diferenciación, necrosis y mitosis
c) Diferenciación, mitosis y polimorfismo
d) Celularidad, polimorfismo, diferenciación
Micropráctica - Patología Ginecológica
*Obligatorio

Instrucciones
- Consta de 10 preguntas en formato múltiple choice (1 sola opción es la correcta) y casillas de
verificación (presentada una entidad nosológica deberán identificar todos los enunciados correctos
teniendo en cuenta que el marcar enunciados correctos de menos o enunciados incorrectos, la
pregunta se da automáticamente por anulada).
- Deberán con contar con 4 preguntas correctas para poder aprobar el ejercicio.
- En el caso de no enviar 1 micropráctica. Las microprácticas no enviadas se registran como
ausentes.
- Deberán registrarlas con su nombre/s y apellido/s tal como figuran en los listados de alumnos
asignados por el Departamento de Informática de Fmed. Caso contrario la micropráctica será
anulada.
- Si se registraran 2 microprácticas bajo la misma identidad, se tomará en cuenta la 1° que haya
sido enviada y registrada por el sistema.

En referencia a la "Patología Endometrial", señale cual/es de los siguientes


enunciados es CORRECTO (uno o varios). Recuerde que deberá marcar
todas las opciones correctas, considerando que el marcar 1 respuesta
incorrecta, dará por anulado el ejercicio. *
1 punto

La Hiperplasia Endometrial se asocia a estimulación estrogénica prolongada siendo


causa importante de metrorragias y precursora habitual del tipo más frecuente de
carcinoma endometrial.
La activación del gen supresor de tumores PTEN es una alteración genética que es
compartida por la hiperplasia endometrial y los carcinomas de endometrio.
Por tratarse de una lesión bien diferenciada, al carcinoma endometrial tipo I se lo
denomina Carcinoma Endometroide.
El Carcinoma endometroide se asocia a factores endógenos tales como: HTA, DBT y
obesidad.
El Carcinoma de endometrio tipo II engloba a tumores de alto grado (G2, G3) y suelen
desarrollarse sobre una hiperplasia endometrial atípica.
El pronóstico del Carcinoma Endometrial dependerá del estadío clínico al momento del
diagnóstico, el grado de diferenciación histológico y el subtipo.
El Carcinoma de Endometrio es frecuente en mujeres menores de 40años.

Una paciente de 59 años presenta metrorragia de la posmenopausia.


Ecografía transvaginal: Lesión exofítica de 2cm de longitud con base de
implantación estrecha en la cavidad uterina. Se le realizó una histeroscopía y
se extirpó la lesión, observándose “glándulas endometriales dilatadas, con
abundante estroma interpuesto y un pedículo con gruesos vasos”. ¿Cuál es
para Ud. el diagnóstico más probable? *
1 punto

Pólipo endometrial
Hiperplasia endometrial simple
Endometritis crónica
Adenocarcinoma endometrial
En referencia a la entidad nosológica "Enfermedad Trofoblástica Gestacional",
señale la opción INCORRECTA: *
1 punto

La persistencia de valores elevados de HCG posterior a una gesta molar debe hacer
sospechar mola invasora.
El fenómeno androgénico es la causa genética más frecuentemente observada en las
molas parciales.
La enfermedad molar se caracteriza histológicamente por presentar vellosidades
coriónicas hidrópicas, avasculares acompañadas con proliferación troboflástica variable.
En el caso de los coriocarcinomas la respuesta a la quimioterapia dependerá del
estadío tumoral.

En referencia a la entidad nosológica "Liquen escleroso vulvar", señale la


opción CORRECTA: *
1 punto

Su pico de incidencia se da en la pubertad dada su relación con el status hormonal


Con mayor frecuencia es un patología que cursa de manera asintomática
La histopatología es variable de acuerdo al momento evolutivo
No se observa relación con el carcinoma epidermoide queratinizante

Una mujer de 37 años ha notado una masa de tamaño creciente en la mama


izquierda durante los últimos 2 años. La paciente atribuye su crecimiento al
roce el corpiño en dicho sector. La ginecóloga le palpa una masa firme de
2cm de diámetro. Se le efectúa una nodulectomía. Macroscópicamente se
reconoce una masa firme-elástica, blanquecina nacarada, de límites netos
que la separan del tejido adiposo circundante. Histológicamente se halla
constituida por una proliferación epitelial canalicular y un estroma mixoide.
¿Cuál es el diagnóstico más probable? *
1 punto

Necrosis grasa por traumatismo


Carcinoma lobulillar
Carcinoma medular
Fibroadenoma

Una mujer de 35 años presenta dolor abdominal cíclico, coincidente con la


menstruación. La paciente se encuentra en estudio por infertilidad. El estudio
laparoscópico revela numerosas lesiones hemorrágicas de 0,2 a 0,5cm de
diámetro en las superficies peritoneales del útero y los ovarios. ¿Cuál de las
siguientes lesiones ováricas es más probable que se asocie a estos
hallazgos? *
1 punto

Fibroma
Tumor de Brenner
Quiste endometriósico
Tumor de Krukenberg
En referencia a los "Tumores de Ovario", señale cual/es de los siguientes
enunciados es CORRECTO (uno o varios). Recuerde que deberá marcar
todas las opciones correctas, considerando que el marcar 1 respuesta
incorrecta, dará por anulado el ejercicio. *
1 punto

El 80% de los tumores ováricos son benignos y afectan preferentemente a mujeres


entre los 20 y 45 años de edad.
La mayoría de los tumores ováricos son funcionales desde el punto de vista hormonal.
Las mutaciones heredadas de los genes BRCA1 y BRCA2 en la línea germinal
aumentan la susceptibilidad al cáncer de ovario.
Los tumores serosos borderline son generalmente quísticos con papilas más largas,
finas y friables e histológicamente muestran invasión estromal.
El coriocarcinoma es un tumor derivado de células epiteliales superficiales del ovario y
su pronóstico es favorable.
La mayoría de los tumores mucinosos primarios son bilaterales.
El disgerminoma es el más común de los tumores germinales malignos, presentándose
con mayor frecuencia en la infancia y juventud.
Los tumores serosos benignos del ovario se presentan como lesiones multiquísticas. La
pared quística es lisa y brillante sin engrosamiento epitelial.

En referencia al Carcinoma de Cuello uterino y sus lesiones precursoras,


señale la opción CORRECTA. *
1 punto

El 80% de los casos de L-SIL aparecen de novo, sólo un 20% se desarrolla a partir de
L-SIL previas.
El 80% de L-SIL y el 100% de H-SIL se asocian a los VPH de alto riesgo (VPH-16 y
VPH-18).
La progresión desde H-SIL a carcinoma de cuello uterino invasor, de producirse, tarda
pocos meses en la mayoría de los casos.
El 80% de los casos de carcinoma de cuello uterino invasor corresponde al subtipo
histológico de adenocarcinoma.

En un control de rutina a una paciente de 30 años se le identificó por


colposcopía una leucoplasia de la cual se tomó biopsia. Informe
histopatológico: “Alteración histoarquitectural del epitelio con alteraciones
celulares a nivel de la relación núcleo-citoplasma, anisocariosis, pérdida de la
polaridad acompañado de imágenes mitóticas en estrato inferior y medio”.
¿Cuál es para Ud. el diagnóstico más probable? *
1 punto
CIN I (SIL de bajo grado)
CIN II (SIL de alto grado)
CIN III (SIL de alto grado)
Carcinoma in situ

Usted se encuentra en el quirófano realizando una práctica de la cursada de


ginecología de la UDH. La intervención a la cual está asistiendo es una
resección de un nódulo de mama localizado en el cuadrante supero-externo
(CSE) de una paciente de sexo femenino de 63 años. Una vez realizada la
resección del nódulo, se le entrega la pieza de tumorectomía al patólogo para
que la evalué y usted lo acompaña. En la sala de anatomía patología del
quirófano observa que tras la realización de cortes seriados de la pieza se
identifica una lesión tumoral de 2 cm de diámetro mayor constituido por tejido
blanquecino de bordes irregulares y espiculados. Tras el examen
macroscópico el patólogo raspa la superficie del tumor con un portaobjeto y
con otro realiza un extendido citológico en donde se observa, grupos
irregulares de células poliédricas cohesivas de núcleo ovoide pleomórfico de
cromatina laxa con ocasionales nucléolo evidente y citoplasma amplio. Se
realizan cortes histológicos con el criostato y se identifican nidos solidos e
irregulares con ocasionales estructuras tubulares de diversa forma y tamaño
separadas por marcada reacción desmoplásica. Según los hallazgos y
recordando lo visto durante la cursada de patología II usted sospecha que la
lesión más probable sería compatible con: Marque la opción CORRECTA: *
1 punto

Una neoplasia mesenquimática maligna compatible con sarcoma.


Una inflamación crónica compatible con mastitis.
Una neoplasia epitelial maligna compatible con sarcoma epiteloide.
Una neoplasia epitelial maligna compatible con carcinoma.
MICROPRÁCTICA 3: PATOLOGÍA CARDIOLÓGICA
*Obligatorio

Instrucciones
- Consta de 10 preguntas en formato múltiple choice (1 sola opción es la correcta) y casillas de verificación (presentada una
entidad nosológica deberán identificar todos los enunciados correctos teniendo en cuenta que el marcar enunciados
correctos de menos o enunciados incorrectos, la pregunta se da automáticamente por anulada). - Deberán con contar con 4
preguntas correctas para poder aprobar el ejercicio. - En el caso de no enviar 1 micropráctica. Las microprácticas no enviadas
se registran como ausentes. - Deberán registrarlas con su nombre/s y apellido/s tal como figuran en los listados de alumnos
asignados por el Departamento de Informática de Fmed. Caso contrario la micropráctica será anulada. - Si se registraran 2
microprácticas bajo la misma identidad, se tomará en cuenta la 1° que haya sido enviada y registrada por el sistema.

1) Los aneurismas micóticos ocurren en pacientes que padecen, ¿Cuál de las


siguientes condiciones enumeradas a continuación? *

Síndrome de Marfan
Hipertensión arterial esencial
Carcinomatosis meníngea
Endocarditis bacteriana

2) Un hombre de 44 años, previamente sano, presenta progresivamente menor tolerancia al esfuerzo


y disnea marcada en los últimos 6 meses. Las constantes vitales son normales. Se aprecian estertores
difusos en todos los campos pulmonares y edema con fóvea hasta las rodillas. La Rx de tórax
demuestra cardiomegalia y edema de pulmón con derrame pleural. Un ecocardiograma revela
aumento de tamaño de las 4 cavidades cardíacas, así como insuficiencia mitral y tricúspidea, con una
fracción de eyección de 30%. La cinecoronariografía pone de manifiesto estenosis de un 10 a 20% en
las arterias coronarias principales. ¿Cuál es el diagnóstico más probable? *

Amiloidosis
Miocardiopatía dilatada idiopática
Cardiopatía reumática
Cardiopatía isquémica en fase dilatada

3) La evolución progresiva de la necrosis por coagulación de los miocitos, la aparición brusca de un


infiltrado polimorfonuclear constituído por neutrófilos y la pérdida de núcleos y estriaciones de los
miocitos son cambios morfológicos que pueden observarse durante la evolución de un infarto de
miocardio. ¿A qué período de tiempo corresponden los cambios mencionados? *

1 a 2 semanas
12 a 72hs
1hs
3 a 7 días

4) En referencia a las "Miocarditis", señale cual/es de los siguientes enunciados es CORRECTO.


Recuerde que deberá marcar todas las opciones correctas, considerando que el marcar respuestas
correctas de menos o 1 respuesta incorrecta, dará por anulado el ejercicio. *

La miocarditis de células gigantes es la variante de mejor pronóstico dentro del espectro de las miocarditis.
En la Enfermedad de Chagas, durante el período crónico, podrá observarse en las miofibrillas los nidos de
amastigotes acompañados de infiltrado inflamatorio mixto.
La miocardiopatía dilatada puede constituirse como complicación de una miocarditis viral.
La presentación clínica de la cardiopatía chagásica crónica es polimorfa pudiendo presentar: trastornos de la
conducción eléctrica intraventricular, arrtimias ventriculares, enfermedad del nodo sinusal, dilatación y
disfunción del ventrículo izquierdo con o sin insuficiencia cardíaca.
La biopsia endomiocárdica es el gold estándar para el diagnóstico de miocarditis ya que posee un 100% de
sensibilidad y especificidad.
El signo histopatológico característico de la miocarditis en actividad es el infiltrado inflamatorio intersticial y el
daño de los miocardiocitos.
En las miocarditis de etiología viral el daño está dado por efecto citopático directo o por la respuesta
inflamatoria exacerbada.

5) En referencia a la "Patología Pericárdica", señale cual/es de los siguientes enunciados es


CORRECTO. Recuerde que deberá marcar todas las opciones correctas, considerando que el marcar 1
respuesta incorrecta, dará por anulado el ejercicio. *

Las MTS en pericardio más frecuentes provienen de: Pulmón, mama, Leucemias, Linfomas y melanomas.
Las MTS de melanoma en pericardio pueden identificarse por inmunohistoquímica (HMB45, S100, MelanA),
confirmando el diagnóstico al obtener positividad en 2 de ellas.
Ante una Pericarditis Hemorrágica se debe sospechar que su origen sea por MTS o por proceso tuberculoso
como causas más frecuentes.
Los diferentes tipos de pericarditis evolucionan a la organización con fibrosis y su consecuente pericarditis
constrictiva.
La Pericarditis Serosa es característica en el contexto de una TBC.
La Pericarditis Purulenta se produce únicamente como consecuencia de la diseminación hematógena desde un
foco piógeno a distancia.
En la miocardiopatía chagásica crónica puede observarse una pericarditis crónica inespecífica con infiltrado
inflamatorio a predominio linfocitario con fibrosis acompañante.

6) Un hombre de 72 años lleva 2 meses con cefaleas. Temperatura: 37°C. Frecuencia cardíaca:
regular, 68 latidos x’. Frecuencia respiratoria 15 respiraciones x’. TA 130/85mmHg. La arteria
temporal derecha es prominente, palpable y dolorosa al tacto. No se auscultan soplos. La VSG es de
100mm/h. Se realiza una biopsia de la arteria temporal y el segmento resecado está engrosado e
histológicamente muestra inflamación granulomatosa con presencia de células gigantes y
fragmentación de la lámina elástica interna. El paciente responde bien al tratamiento con
corticoesteroides. ¿Cuál de las siguientes complicaciones de esta enfermedad es más probable que
ocurra en los pacientes no tratados? *

Hipertensión maligna
Insuficiencia renal
Hemoptisis
Ceguera

7) Con respecto a la entidad nosológica “Cardiopatía isquémica”, señale cuál de las siguientes
afirmaciones es INCORRECTA: *

En más del 90% de los casos se relaciona con ateroesclerosis de las coronarias epicárdicas.
Todas las entidades relacionadas a estos síndromes se caracterizan por presentar isquemia miocárdica.
Entre las complicaciones tempranas del Infarto Agudo de Miocardio se debe tomar en cuenta el aneurisma
ventricular.
La consecuencia histopatológica del Infarto Agudo de Miocardio es la sustitución de las fibras necróticas por
cicatriz colágena densa.

8) ¿Cuál de las siguientes afirmaciones describe morfológicamente a la entidad


nosológica “Endocarditis infecciosa”? *
Una hilera de pequeñas (1-2mm) vegetaciones verrugosas a lo largo de las líneas de cierre de las valvas.
Vegetaciones asépticas, múltiples, adheridas a la línea de cierre. Rara vez tienen más de 0.5 cm de tamaño. Sin
embargo, son friables y muy propensas a embolizar.
Vegetaciones pequeñas, asépticas, <5mm a ambos lados de las valvas.
Masas irregulares, friables y generalmente voluminosas sobre las cúspides valvulares que se extienden sobre
las cuerdas tendinosas.

9) Determine a qué tipo de miocardiopatía se corresponden las siguientes imágenes: *

Miocardio no compactado
Miocardiopatía dilatada
Miocardiopatía restrictiva
Miocardiopatía hipertrófica

10) El tumor cardíaco más frecuente del adulto es: *

Mixoma
Rabdomioma
Fibroelastoma papilar
Lipoma
Micropráctica - Patología Endócrina
*Obligatorio

Instrucciones

- RECUERDE: En las preguntas tipo múltiple choice, 1 sola opción es la correcta. - En las preguntas tipo
casilla de verificación, deberá identificar todos los enunciados correctos teniendo en cuenta que el marcar
enunciados correctos de menos o enunciados incorrectos, la pregunta se da automáticamente por
anulada.

En referencia a los "Tumores Tiroideos", señale cual/es de los siguiente/s


enunciado/s es CORRECTO. Recuerde que deberá marcar todas las opciones
correctas, considerando que el marcar 1 respuesta incorrecta, dará por anulado
el ejercicio. *

El 80% de los carcinomas medulares forman parte de Síndromes NEM siendo


multicéntricos y bilaterales.

En caso de niveles bajos de TSH, debe efectuarse una centellografía de tiroides, ya


que un nódulo funcionante casi invariablemente descarta cáncer.

La mayoría de los carcinomas anaplásicos se caracterizan por la invasión a


estructuras adyacentes, crecimiento rápido y MTS al momento del diagnóstico.

La presencia de mutaciones BRAF en los carcinomas papilares se relaciona con peor


pronóstico.

El resultado de una PAAF (punción aspiración con aguja fina) de nódulo tiroideo se
informa como una lesión folicular, pero no permite diferenciar un adenoma de un
carcinoma folicular.

El carcinoma papilar tiroideo es el tipo histológico menos frecuente y su diagnóstico


histológico se basa en la observación de estructuras papilares.

Los adenomas foliculares con mayor frecuencia son los precursores de los
carcinomas.

El carcinoma medular es una neoplasia neuroendócrina cuyo origen son las células
parafoliculares por lo cual el dosaje de calcitonina es útil para el diagnóstico y
seguimiento post quirúrgico.
En referencia a los "Tumores Hipofisarios", señale cual/es de los siguientes
enunciados es CORRECTO. Recuerde que deberá marcar todas las opciones
correctas, considerando que el marcar 1 respuesta incorrecta, dará por anulado
el ejercicio. *

Considerando todas las causas de hipertiroidismo, los adenomas hipofisarios


tirotropos constituyen la causa más frecuente.

Los adenomas hipofisarios no funcionantes son una causa probable de


hipopituitarismo.

Los adenomas hipofisarios no funcionales suelen manifestarse clínicamente en


etapas avanzadas cuando constituyen macroadenomas.

Los adenomas corticotropos funcionantes generan hipersecreción suprarrenal de


cortisol y desarrollo de hipercortisolismo.

El hallazgo de hiperprolactinemia, es diagnóstico de certeza de prolactinoma


hipofisario.

La visualización de erosión de la silla turca y las apófisis clinoides en las imágenes


radiológicas, es un signo indirecto de la presencia de un macroadenoma.

El Sindrome de Sheehan se caracteriza por la necrosis isquémica de la glándula


pituitaria en el período postparto.

La conducta biológica del adenoma hipofisario es previsible en función de su aspecto


histológico.
¿Cuál de las siguientes descripciones se corresponde con la imagen histológica?
Señale la opción CORRECTA: *

Células foliculares con leve anisocariosis, que formaban pequeños folículos, que en
sectores se propaga a la cápsula.

Parénquima reemplazado por infiltrado linfoplasmocítico difuso y nodular con


centros germinativos y células foliculares con citoplasma granular eosinófilo.

Parénquima reemplazado por células gigantes multinucleadas englobando lagunas


de coloide.

Folículos de contornos irregulares, tapizados por epitelio cilíndrico alto, con pequeñas
papilas que se proyectan en su luz.
Una ejecutiva de una multinacional es internada a causa de una severa
hipotensión, anorexia, náuseas y vómitos; presenta en el examen
hiperpigmentación cutáneomucosa; antecedentes de candidiasis a repetición, e
hipoparatiroidismo. En los estudios por imágenes, las suprarrenales se hallan
marcadamente disminuidas de tamaño, siendo incluso difíciles de identificar.
Fallece a causa de un grave trastorno hidroelectrolítico. ¿Cuál de los siguientes
cuadros cree Ud. será más probable hallar en la necropsia? *

Adrenalitis autoimmune en el contexto del Síndrome Poliendócrino Autoinmunitario


de tipo I

Adrenalitis tuberculosa

Lesión destructiva de la silla turca que compromete el hipotálamo e hipófisis

Metástasis adrenal de carcinoma primitivo pulmonar

Un paciente de 50 años presenta HTA, aumento de peso con deposición de


tejido adiposo a predominio en tronco, facies de “luna llena”, disminución de la
masa muscular, hiperglucemia, glucosuria y polidipsia. En el examen de
laboratorio presenta aumento de los niveles de cortisol libre en orina y pérdida
del patrón diurno de secreción de cortisol, bajo nivel de ACTH. Se le descubre y
extirpa un “tumor adrenal cortical amarillento rodeado por una fina cápsula y que
histológicamente muestra células semejantes a las de la zona fasciculada”. ¿Cuál
es para Ud. el diagnóstico más probable? *

Metástasis en adrenal

Feocromocitoma

Carcinoma adrenal

Adenoma adrenal
Una niña de 15 años se desmaya durante un partido de handball y es llevada a la
Guardia. Al ingreso está obnubilada. En el examen físico presenta taquicardia e
hipotensión, con una respiración rápida, profunda y dificultosa. Se le realizan
pruebas de laboratorio que revelan glucemia de 521 mg/dl; un análisis de orina
con glucosuria 4+ y cetonuria 4+, sin presencia de proteínas, sangre o nitritos.
¿Cuál sería la alteración patológica más probable a identificar en el páncreas de
esta niña? *

Inflamación crónica de los islotes de Langerhans

Hiperplasia de las células α de los islotes de Langerhans

Pérdida de células β de los islotes de Langerhans

Sustitución por amiloide de los islotes de Langerhans

Una paciente de 50 años acude a la consulta por un "bulto" a nivel del lóbulo
tiroideo derecho. Presenta dolor a la palpación y refiere haber tenido un episodio
gripal la semana anterior. Si realizara una PAAF cual sería la descripción más
apropiada para este tipo de presentación clínica: *

Células gigantes multinucleadas con infiltrado mononuclear alrededor de folículos


dañados.

Infiltrado mononuclear con formación de centros germinales y folículos atrofiados.

Células gigantes multinucleadas, linfocitos, células epiteloides y material amorfo


compatible con caseum.

Numerosos linfocitos, células de Hürthle y células foliculares con características


benignas.
Una paciente de 37 años consultó a su ginecólogo por amenorrea de 6 meses,
galactorrea y esterilidad. Tenía claros signos de acromegalia y hemianopsia
bitemporal. En la RMN se observó aumento de tamaño de la hipófisis con
compresión del quiasma óptico y del tallo hipofisario. Tras su extirpación, el
estudio histológico y la inmunohistoquímica confirmaron un Adenoma
Somatotropo. ¿Cuál es para Ud. la explicación de la signo-sintomatología
ginecológica? *

la hormona de crecimiento y la prolactina actúan sobre los mismos receptores


periféricos.

la compresión del tallo impide la llegada del factor inhibidor de prolactina.

la hormona de crecimiento tiene efecto “símil” factor liberador de prolactina.

la infiltración tumoral es la justificación de la signo-sintomatología que presenta la


paciente.

Esta imagen se corresponde con un carcinoma de tiroides. ¿A cuál de ellos


corresponde? *

Carcinoma medular

Carcinoma papilar

Carcinoma anaplásico

Carcinoma folicular
Un paciente de 45 años presenta cólicos renales frecuentes, diagnosticándose
Nefrolitiasis recidivante, bilateral, con cálculos de fosfato cálcico. Presenta
además dolores óseos generalizados con deformaciones óseas. Los análisis de
laboratorio muestran hipercalcemia, hipofosfatemia, hipercalciuria, e
hiperfosfaturia y aumento de la concentración de parathormona en sangre. En
glándulas paratiroides se encuentra una “masa redondeada en la glándula
superior izquierda, la cual es extirpada. En el examen histológico se observa un
nódulo compuesto por células poligonales uniformes con núcleos centrales,
pequeños, rodeado por una cápsula fibrosa, en el borde de la muestra se
observa un anillo de tejido paratiroideo comprimido”. ¿Cuál es para Ud. el
diagnóstico más probable? *

Hiperplasia paratiroidea

Adenoma paratiroideo

Hiperparatiroidismo secundario

Carcinoma paratiroideo

Atrás Enviar

Nunca envíes contraseñas a través de Formularios de Google.

Este formulario se creó en Facultad de Medicina. Noti car uso inadecuado

Formularios
¿Enviar la respuesta?
Solo puedes responder una vez este formulario. ¿Quieres continuar?

CAMBIAR DE CUENTA ENVIAR

Micropráctica - Patología Respiratoria


*Obligatorio

Instrucciones

RECUERDE:
- En las preguntas tipo múltiple choice, 1 sola opción es la correcta.
- En las preguntas tipo casilla de verificación, deberá identificar todos los enunciados correctos teniendo
en cuenta que el marcar enunciados correctos de menos o enunciados incorrectos, la pregunta se da
automáticamente por anulada.

Un paciente de 57 años, consultó por hemoptisis. En la placa de tórax se observó


una imagen radiopaca de límites difusos en relación con el bronquio fuente. Se le
realizó una fibrobroncoscopía con cepillado bronquial, observándose células
epiteliales atípicas con marcada anisocariosis, hipercromasia, cromatina irregular
y nucleólo evidente, citoplasma eosinófilo con focos de disqueratosis. ¿Cuál es
para usted el diagnóstico más probable? *

Adenocarcinoma

Carcinoma epidermoide

Carcinoma de células pequeñas

Tumor carcinoide
Lesión peribronquial que infiltra el tejido pulmonar, de coloración blancogrisácea,
con áreas de necrosis y hemorragia. El estudio histopatológico reveló playas
sólidas de células atípicas, sin puentes intercelulares ni queratinización. IHQ: p63,
p40 y CK5/6 (+). ¿Qué tumor presenta estas características? *

Adenocarcinoma

Carcinoma epidermoide bien diferenciado

Carcinoma de células grandes

Carcinoma epidermoide pobremente diferenciado


En referencia a las "Infecciones Pulmonares", señale cual/es de los siguientes
enunciados es CORRECTO. Recuerde que deberá marcar todas las opciones
correctas, considerando que el marcar respuestas correctas de menos o 1
respuesta incorrecta, dará por anulado el ejercicio. *

El streptococo neumoniae (neumococo) es la causa más frecuente de neumonía


intrahospitalaria.

La prevalencia de neumonía adquirida en la comunidad aumenta en pacientes con


factores predisponentes tales como EPOC, oncológicos, DBT, etc.

La TBC y la histoplasmosis pulmonar pueden producir granulomas con necrosis


caseificante

La etapa de congestión en la neumonía lobar se caracteriza a nivel histológico por


exudado fibrinoso a nivel alveolar.

El absceso pulmonar es un proceso supurativo local que produce necrosis del tejido
pulmonar siendo los streptococos aerobios y anaerobios, S.aureus y bacterias Gram -
los agentes etiológicos aislados con mayor frecuencia.

En el contexto de la respuesta inmune a la infección por SARS-COV2 se identifican a


las IL-1β, IL-6 y TNFα como moléculas beneficiosas.

Las infecciones virales con afectación pulmonar se caracterizan por una reacción
inflamatoria instersticial.

Una de las posibles complicaciones de la neumonía bacteriana aguda es la


diseminación bacteriémica hacia las válvulas cardíacas.

A un paciente de 60 años, fumador de larga data, que consulta por disfonía, se le


realiza una laringoscopia, observándose una lesión exofítica sobre la cuerda
vocal derecha. La biopsia informa "Carcinoma Laríngeo". Marque la opción
CORRECTA: *

Los carcinomas glóticos tienen mal pronóstico ya que su sintomatología es tardía.

El 95% corresponden histológicamente a adenocarcinomas.

La mucosa adyacente a la lesión infiltrante puede mostrar hiperplasia, displasia o


carcinoma in situ.

Frecuentemente el carcinoma se origina en los llamados nódulos laríngeos, nódulos


reactivos o pólipos, relacionados con grandes esfuerzos laríngeos.
En relación al Adenocarcinoma de Pulmón. Señale la opción INCORRECTA: *

La hiperplasia adenomatosa atípica se considera como una lesión precursora que


mide menos de 5mm y que se caracteriza histológicamente por poseer neumonocitos
displásicos que revisten las paredes alveolares acompañado de ligera fibrosis
intersticial.

Generalmente la topografía de la lesión es central.

El Adenocarcinoma in situ se describe como una lesión que mide ≤3cm, con un
patrón de crecimiento lepídico, que puede o no producir mucina pero no evidencia
invasión estromal, de la pleura visceral o linfovascular.

El TTF-1 es el marcador de mayor especificidad y sensibilidad para diferenciar al


Adenocarcinoma de otros carcinomas.

Con respecto al “Enfisema pulmonar” marque la opción CORRECTA: *

La pérdida del tejido elástico lleva al colapso de las vías respiratorias pequeñas
durante la inspiración.

No produce espasmo de los vasos pulmonares.

El enfisema centroacinar es el de mayor frecuencia.

El enfisema panacinar es el tipo de enfisema más frecuente y se observa en los


lóbulos superiores del pulmón.
En referencia a las "Neumopatías obstructivas", señale cual/es de los siguientes
enunciados es CORRECTO. Recuerde que deberá marcar todas las opciones
correctas, considerando que el marcar 1 respuesta incorrecta, dará por anulado
el ejercicio. *

La hipertrofia de las gándulas submucosas de la tráquea y los bronquios


conjuntamente con el aumento de las células caliciformes de la pequeña vía son las
responsables de la hipersecreción de moco en la Bronquitis Crónica.

El Asma atópica se debe fundamentalmente a una respuesta TH1 no relacionada con


los niveles de IgE.

Los espirales de Curschmann y los cristales de Charcot-Leyden observados en


muestras de esputo y Lavado bronquioalveolar conjuntamente con los hallazgos
histológicos tales como hiperplasia de células caliciformes, fibrosis de la membrana
subbasal, infiltrado inflamatorio con predominio de eosinófilos e hipertrofia muscular
bronquial se constituyen en hallazgos característicos de pacientes que padecen
Asma.

La Enfermedad Pulmonar Obstructiva Crónica (EPOC) se caracteriza por la limitación


al flujo aéreo durante la inspiración.

En la patogenia del enfisema debe considerarse estress oxidativo, liberación de


mediadores inflamatorios y el desequilibrio enzimático proteasa-antiproteasa.

El Enfisema Centroacinar se halla vinculado a estructuras cicatrizales.

El Enfisema Panacinar se observa con mayor frecuencia en las bases pulmonares


presentando en su patogenia una estrecha relación con la deficiencia de α1
antitripsina.

En los pacientes con Enfisema, el desarrollo de corazón pulmonar y, finalmente, de la


ICC, en relación con la Hipertensión Pulmonar 2°, se asocia a mal pronóstico.
Un paciente de 53 años de edad, trasplantado pulmonar, 9 meses después de la
cirugía comenzó con disnea, tos y febrícula. Se realizó Rx de tórax en la que se
observaron infiltrados reticulonodulilares en ambos pulmones. Se realizó una
biopsia transbronquial. Histológicamente se observaron bronquiolos con
destrucción parcial de la pared y un exudado organizado constituido por tejido
fibroso que obstruía la luz de los mismos. ¿Cuál es para usted el diagnóstico más
probable? *

Rechazo crónico

Bronconeumonía atípica

Rechazo agudo

Neumonitis por hipersensibilidad

En referencia a la entidad nosológica “Patología pleural”, indique la opción


CORRECTA *

Las pleuritis hemorrágicas se presentan generalmente en el contexto de las


colagenopatías.

Las colecciones serosas no inflamatorias están asociadas a patología neoplásica


maligna.

El Lupus eritematoso sistémico y la artritis reumatoidea son causa de pleuritis con


exudado seroso y serofibrinoso.

Hasta el momento no hay explicación fisiopatológica certera para el derrame pleural


de origen cardíaco.
Un paciente de 43 años con tuberculosis tratada hace 10 años presentó fiebre,
tos, expectoración y decaimiento general. En la Rx de tórax se observó una
imagen densa (nódulo pulmonar solitario). En la biopsia guiada por TAC, se
observó “fenómeno inflamatorio de tipo granulomatoso acompañado por un
conglomerado de filamentos delgados y tabicados, con ramificaciones en ángulo
agudo, técnicas de PAS y GROCOTT positivos”. ¿Cuál es para usted el
diagnóstico más probable? *

Candidiasis

Aspergilosis

Mucromicosis

Actinomicosis

Atrás Enviar

Nunca envíes contraseñas a través de Formularios de Google.

Este formulario se creó en Facultad de Medicina. Noti car uso inadecuado

Formularios
1/3/2021 EXAMEN FINAL 1° DE MARZO 2021

EXAMEN FINAL 1° DE MARZO 2021


*Obligatorio

Instrucciones

RECUERDE:
- En las preguntas tipo múltiple choice, 1 sola opción es la correcta.
- En las preguntas tipo casilla de verificación, deberá identificar todos los enunciados CORRECTOS
teniendo en cuenta que el marcar enunciados correctos de menos o enunciados incorrectos, la pregunta
se da automáticamente por anulada.

En referencia a la “Patología tumoral prostática”, señale cuales de los siguientes


enunciados es CORRECTO. Recuerde que deberá MARCAR TODAS LAS
OPCIONES CORRECTAS, considerando que el marcar correctas de menos o 1
respuesta incorrecta, dará por anulado el ejercicio. *

La gradación del cáncer de próstata mediante el sistema de Gleason se correlaciona


con el estadio patológico y el pronóstico.

Se considera que en la mayoría de los hombres a los 80 años se pueden encontrar


signos histológicos de hiperplasia prostática benigna.

El cáncer de próstata presenta pérdida de E-cadherina al igual que el carcinoma


invasivo de células en anillo de sello gástrico y el carcinoma lobulillar de mama.

La determinación sérica de PSA total por sí sola, es diagnóstico de cáncer de


próstata.

La forma más frecuente de presentación clínica en la hiperplasia prostática benigna


es la hematuria.

Considerando los componentes estructurales de la glándula, tanto las células


epiteliales como las del estroma están implicadas en el proceso de hiperplasia.
https://docs.google.com/forms/d/e/1FAIpQLSftEX0-83LIWJqKuSQUO8i0JvRYipp5ImopH62AT0G09vIGXw/formResponse 1/28
1/3/2021 EXAMEN FINAL 1° DE MARZO 2021
ep te a es co o as de est o a está p cadas e e p oceso de pe p as a.

En referencia a la entidad “Adenoma folicular de tiroides”, indique el enunciado


CORRECTO: *

El adenoma folicular puede carecer de cápsula fibroconectiva propia y ser una lesión
proliferativa con alta densidad celular y células de Hürthle.

El adenoma folicular puede tener crecimiento expansivo con infiltración y perforación


de la cápsula propia pero sin embolias.

El adenoma folicular puede tener células parafoliculares (células C), células


oncocíticas y estar asociada a tiroiditis linfocitaria.

Es una proliferación de células foliculares con cápsula propia que puede tener células
con citoplasma granular eosinófilo por la presencia de abundantes mitocondrias.

Respecto al "Mieloma Múltiple", señale cual de los siguientes enunciados es


INCORRECTO: *

La destrucción ósea no es evidente en el mieloma múltiple.

Puede haber infiltración por células plasmáticas en otros órganos además de la


ubicación ósea.

Puede producir insuficiencia renal.

Es una neoplasia de células plasmáticas asociada comúnmente a la presencia de un


componente M en sangre.

https://docs.google.com/forms/d/e/1FAIpQLSftEX0-83LIWJqKuSQUO8i0JvRYipp5ImopH62AT0G09vIGXw/formResponse 2/28
1/3/2021 EXAMEN FINAL 1° DE MARZO 2021

Informe histopatológico de lesión nodular irregular de antebrazo que medía


1,9cm, de coloración heterogénea: “Proliferación de células melanocíticas
atípicas, con 5mitosis/mm2 que se extiende 3mm en profundidad
comprometiendo la dermis reticular. Se observa infiltrado linfocitario por debajo
de la lesión. ¿Cuál de los siguientes es para Ud. el enunciado CORRECTO? *

La lesión se corresponde con un nivel de Clark III y un índice de Breslow T4

La lesión se corresponde con un nivel de Clark IV y un índice de Breslow T3

La lesión se corresponde con un nivel de Clark II y un índice de Breslow T2

La lesión se corresponde con un nivel de Clark V y un índice de Breslow T1

Con respecto al Esófago de Barrett, señale la opción CORRECTA: *

La mayoría de los pacientes con esófago de Barrett desarrolla tumores esofágicos.

Para su diagnóstico se requiere la presencia en la endoscopía de mucosa anormal


por encima de la unión gastroesofágica y metaplasia intestinal demostrada
histológicamente.

Se presenta con mayor frecuencia entre los 20 y 40 años de edad.

Su presencia aumenta el riesgo de carcinoma epidermoide de esófago.

En referencia a la entidad “Gota”, señale cuál de los enunciados es INCORRECTO:


*

d) Aproximadamente el 20% de los pacientes con gota crónica fallecen por


insuficiencia renal.

https://docs.google.com/forms/d/e/1FAIpQLSftEX0-83LIWJqKuSQUO8i0JvRYipp5ImopH62AT0G09vIGXw/formResponse 3/28
1/3/2021 EXAMEN FINAL 1° DE MARZO 2021

Se presenta como oligoartritis, poliartritis y afectación sistémica.

Manifestaciones de la Gota incluyen: artritis aguda, artritis tofácea crónica, tofos en


varias localizaciones y nefropatía gotosa.

Se debe a cantidades excesivas de ácido úrico producto final de la degradación de las


purinas dentro de los tejidos y líquidos corporales.

En referencia a la entidad “Tumores del SNC", señale cual de los siguientes


enunciados es CORRECTO. *

El meningioma fibroblástico presenta numerosos cuerpos de psamoma.

Los ependimomas mixopapilares suelen localizarse en el filum terminale de la


medula espinal.

Los tumores de células germinales cerebrales primarios se presentan con frecuencia


en lóbulos parietales y temporales del cerebro.

Los tumores neuronales son más frecuentes que los tumores gliales.

Paciente de 68 años de edad con disnea y Cáncer de mama. Examen físico:


ruidos cardíacos alejados, TA 90/60, FC 122/min. Se realiza ECG, Rx (IC-Índice
cardiotorácico: aumentado con silueta cardíaca en forma de "botellón") y
angiotac que confirman el diagnóstico. Se realiza pericardiocentesis. ¿Cuál es el
tipo de derrame pericárdico más frecuente en esta entidad nosológica? *

Fibrinoso

Seroso

Purulento

Hemorrágico

https://docs.google.com/forms/d/e/1FAIpQLSftEX0-83LIWJqKuSQUO8i0JvRYipp5ImopH62AT0G09vIGXw/formResponse 4/28
1/3/2021 EXAMEN FINAL 1° DE MARZO 2021

Respecto de los Tumores Mucinosos de Ovario, señale la opción que usted


considere CORRECTA: *

El cistoadenoma mucinoso se presenta habitualmente como un tumor sólido con


áreas de necrosis y hemorragia.

En el cistoadenocarcinoma, el porcentaje de moco y de necrosis intratumoral es un


factor pronóstico importante.

Se deberá establecer un diagnóstico diferencial entre cistoadenocarcinoma mucinoso


de ovario vs. adenocarcinomas mucinosos metastásicos de colon.

El criterio más importante para diferenciar el cistoadenoma del cistoadenocarcinoma


es el tamaño tumoral.

Varón de 45 años nota desde hace 4 meses un área rojiza solitaria de 1 cm. en el
cuerpo del pene. El estudio histológico de la biopsia de la lesión muestra células
epiteliales disqueratósicas y displásicas con mitosis dispersas en todo el espesor
del epitelio. ¿Cuál es el diagnóstico más probable? *

Enfermedad de Bowen

Eritroplasia de Queyrat

Balanopostitis

Condiloma acuminado

https://docs.google.com/forms/d/e/1FAIpQLSftEX0-83LIWJqKuSQUO8i0JvRYipp5ImopH62AT0G09vIGXw/formResponse 5/28
1/3/2021 EXAMEN FINAL 1° DE MARZO 2021

En referencia a la “Hipertensión Pulmonar” señale cual/es de los siguientes


enunciados es correcto. Recuerde que deberá MARCAR TODAS LAS OPCIONES
CORRECTAS, considerando que al marcar opciones correctas de menos o 1
respuesta incorrecta, dará por anulado el ejercicio: *

La apnea obstructiva del sueño es un trastorno frecuente que se asocia a obesidad e


hipoxemia y puede ser un factor contribuyente al desarrollo de HTP y corazón
pulmonar.

La hipertensión pulmonar (HTP) se define como una presión arterial pulmonar media
igual o mayor a 25 mmHg en reposo.

La hipertensión pulmonar idiopática se presenta con mayor frecuencia en hombres


mayores de 60 años, con disnea, cansancio y dolor torácico de tipo anginoso.

En la mayoría de los casos familiares de HTP se observan mutaciones inactivadoras


en la línea germinal del gen de la proteína morfogenética ósea de tipo 2 (BMPR2).

La coexistencia de comorbilidades como fibrosis pulmonar, enfisema y bronquitis


crónica grave puede provocar hipoxia crónica y causar HTP.

Las lesiones plexiformes, caracterizadas por la presencia de un “penacho” de


formaciones capilares que forman una red que llena la luz de las arterias pequeñas,
dilatadas, de paredes finas, es la lesión característica incipiente que se observa en
los casos más leves de HTP.

https://docs.google.com/forms/d/e/1FAIpQLSftEX0-83LIWJqKuSQUO8i0JvRYipp5ImopH62AT0G09vIGXw/formResponse 6/28
1/3/2021 EXAMEN FINAL 1° DE MARZO 2021

Un varón de 18 años se cae mientras practicaba skate golpeándose el lado


izquierdo de su cabeza contra una pared. Al examen físico solo se constata una
pequeña herida en el cuero cabelludo, con escasa hemorragia que se controla en
pocos minutos. Se encuentra activo luego del accidente, pero después de 30
minutos pierde la consciencia y fallece. Se presentan los hallazgos del examen
necrópsico (ver imagen). Estos hallazgos están más probablemente relacionados
con el daño de qué parte de la vasculatura cerebral? *

Vena de Galeno

Arteria cerebral media – ramas lentículo estriadas

Seno cavernoso

Arteria meníngea media

https://docs.google.com/forms/d/e/1FAIpQLSftEX0-83LIWJqKuSQUO8i0JvRYipp5ImopH62AT0G09vIGXw/formResponse 7/28
1/3/2021 EXAMEN FINAL 1° DE MARZO 2021

En referencia a la "Patología Intersticial pulmonar", señale cuál de los siguientes


enunciados es INCORRECTO. *

La neumonía intersticial descamativa es una lesión asociada al tabaquismo y se


caracteriza por espacios alveolares con macrófagos cargados de pigmento tabáquico
pardo-dorado.

En el patrón de neumonía intersticial no específica es frecuente observar focos


fibroblásticos, membranas hialinas y micropanalización.

En la asbestosis el patrón fibrosante es similar a la neumonía intersticial usual con


focos fibroblásticos y diversos grados de fibrosis acompañados por cuerpos de
asbesto.

El evento fisiopatogénico central es el fenómeno inflamatorio y de reparación luego


de la injuria de la pared alveolar.

Paciente femenina de 32 años de edad, con antecedentes familiares de cáncer


de mama en su madre y abuela. Presenta una lesión nodular de 2cm. de diámetro
de bordes regulares y limites netos. A la histología evidencia un componente
epitelial y otro estromal con importante proliferación de este último. El
diagnostico diferencial más acertado sería: *

Necrosis vs. Absceso mamario

Fibroadenoma vs. Tumor filoides

Mastitis quistica vs. Rabdomiosarcoma

Carcinoma ductal infiltrante vs. Carcinoma ductal In situ

https://docs.google.com/forms/d/e/1FAIpQLSftEX0-83LIWJqKuSQUO8i0JvRYipp5ImopH62AT0G09vIGXw/formResponse 8/28
1/3/2021 EXAMEN FINAL 1° DE MARZO 2021

En referencia a los “Tumores Testiculares”, cuál de las siguientes afirmaciones es


INCORRECTA: *

Los tumores derivados del estroma y de los cordones sexuales (células no


germinales) son los màs frecuentes y la gran mayoría de ellos tienen comportamiento
biológico agresivo.

El linfoma testicular es el tumor testicular más frecuente en ancianos.

El 60% de los tumores testiculares está compuesto por màs de un patròn “puro”.

Entre los factores de riesgo debe considerarse el Síndrome de disgenesia testicular.

En referencia a los “Linfomas Hodgkin”, señale cuales de los siguientes


enunciados es CORRECTO. Recuerde que deberá MARCAR TODAS LAS
OPCIONES CORRECTAS, considerando que el marcar correctas de menos o 1
respuesta incorrecta, dará por anulado el ejercicio. *

En el linfoma de Hodgkin clásico las células tumorales marcan con CD45 positivo,
CD15 y CD30 negativos.

Los linfocitos atípicos de esta neoplasia se acompañan de otras células grandes,


binucleadas, las llamadas células de Reed Sternberg de carácter reactivo.

En el tipo de predominio linfocítico «no clásico» los ganglios afectados están


difuminados por un infiltrado nodular de linfocitos pequeños mezclados con un
número variable de macrófagos y las Células de Reed-Sternberg variante L-H o
palomita de maíz.

En el tipo celularidad mixta coexiste un infiltrado celular heterogéneo que contiene


linfocitos T, eosinófilos, células plasmáticas y macrófagos, mezclados con células de
Reed- Sternberg variante clásica.

Las formas clásicas de Linfoma de Hodgkin se asocian a menudo a mutaciones


adquiridas que activan el factor de transcripción NF-κB y a infección por el VEB,
mientras que el tipo de predominio linfocítico expresa marcadores de linfocitos B y no
https://docs.google.com/forms/d/e/1FAIpQLSftEX0-83LIWJqKuSQUO8i0JvRYipp5ImopH62AT0G09vIGXw/formResponse 9/28
1/3/2021 EXAMEN FINAL 1° DE MARZO 2021
q p p p y
se asocia al VEB.

La esclerosis nodular tiende a afectar los ganglios infradiafragmáticos de hombres


mayores de 60 años.

En referencia a la "Trastornos ampollares inflamatorios de la piel”, señale cual/es


de los siguientes enunciados es CORRECTO. Recuerde que deberá marcar todas
las opciones correctas, considerando que al marcar opciones correctas de
menos o 1 respuesta incorrecta, el sistema dará por anulado el ejercicio. *

En el pénfigo vulgar se forman auto anticuerpos contra laminina IV y


hemidesmosomas que se demuestran con técnicas de inmunohistoquímica.

La dermatitis herpetiforme se presenta como vesículas pruriginosas y ampollas que


se agrupan. Tiene una fuerte asociación con la enfermedad celíaca; y frecuentemente
resuelve con una dieta libre de gluten.

El pénfigo vulgar afecta piel expuesta al sol, piel no expuesta, cuero cabelludo, tronco,
extremidades y zonas de piel sometidas a presión mecánica.

El penfigoide ampolloso se produce por destrucción autoinmune de


hemidesmosomas entre las células basales y la membrana basal subyacente.

El penfigoide ampolloso suele afectar a pacientes jóvenes, quienes presentan


ampollas tensas y difíciles de romper en cara interna de muslos y superficies flexoras
de antebrazos, axilas e ingle. Suelen observarse lesiones orales en un 80-85% de los
casos.

El eritema multiforme es una reacción de hipersensibilidad caracterizada por una


erupción con lesiones en diana y ampollas. El aspecto en diana de las lesiones se
debe a una necrosis epidérmica central rodeada de eritema.

https://docs.google.com/forms/d/e/1FAIpQLSftEX0-83LIWJqKuSQUO8i0JvRYipp5ImopH62AT0G09vIGXw/formResponse 10/28
1/3/2021 EXAMEN FINAL 1° DE MARZO 2021

En referencia a la "Patología tumoral de partes blandas”, señale cual/es de los


siguientes enunciados es CORRECTO. Recuerde que deberá MARCAR TODAS
LAS OPCIONES CORRECTAS, considerando que al marcar opciones correctas de
menos o 1 respuesta incorrecta, el sistema dará por anulado el ejercicio. *

Pacientes con poliposis adenomatosa familiar con mutaciones APC de la línea


germinal son propensos a padecer fibromatosis profunda.

El rabdomiosarcoma es el sarcoma de partes blandas más frecuente de la infancia y


la adolescencia.

Los tumores de partes blandas constituyen una patología exclusiva del adulto.

Los gangliones se producen por degeneración quística o mixoide del tejido conjuntivo
articular y presenta revestimiento epitelial.

El fibroma es el tumor benigno más frecuente de partes blandas en adultos.

Las variantes histológicas del rabdomiosarcoma son el embrionario, alveolar y


pleomorfo; los dos primeros más frecuentes en la infancia, el pleomorfo más
frecuente en adultos.

Un joven de 17 años presenta dolor de garganta seguido 1 día después de cefalea


intensa de inicio súbito. En el examen físico se observan hemorragias petequiales
en la piel, signos de faringitis leve y rigidez de nuca. Su temperatura es de 38,8
°C; la frecuencia cardíaca, 110 latidos/min y la frecuencia respiratoria de
26respiraciones/min. ¿Cuál es el diagnóstico más probable? *

Meningitis por Neisseria meningitidis

Encefalitis por Cryptococcus neoformans

Hematoma cerebral por malformaciones arteriovenosas

Meningitis por Mycobacterium tuberculosis


https://docs.google.com/forms/d/e/1FAIpQLSftEX0-83LIWJqKuSQUO8i0JvRYipp5ImopH62AT0G09vIGXw/formResponse 11/28
1/3/2021 EXAMEN FINAL 1° DE MARZO 2021

En referencia al “Hipopituitarismo”, señale cual de los siguientes enunciados NO


se corresponde como causa del mismo. *

Lesión cerebral traumática.

Hiperparatiroidismo secundario.

Adenomas hipofisarios no secretores.

Síndrome de Sheehan.

https://docs.google.com/forms/d/e/1FAIpQLSftEX0-83LIWJqKuSQUO8i0JvRYipp5ImopH62AT0G09vIGXw/formResponse 12/28
1/3/2021 EXAMEN FINAL 1° DE MARZO 2021

Una mujer de 55 años concurre a un examen médico de control, en el que se


detecta una TA de 165/110mm Hg. En el examen físico no se observan
alteraciones. El análisis de orina revela un pH de 7; una densidad de 1,020;
proteinuria de 1+ y ausencia de sangre, glucosa o cetonas. Los resultados de las
pruebas de anticuerpos antinucleares y ANCA son negativos. La concentración
de hemoglobina A1c se encuentra de los valores normales. La ecografía
abdominal demuestra riñones de tamaño pequeño, en forma simétrica y
bilateral, sin presencia de masas ocupantes. Se muestra una imagen
representativa de la corteza renal de esta paciente. ¿Cuál es su diagnóstico? *

Poliquistosis renal autosómica dominante

Glomeruloesclerosis nodular

Glomerulonefritis crónica

Amiloidosis

https://docs.google.com/forms/d/e/1FAIpQLSftEX0-83LIWJqKuSQUO8i0JvRYipp5ImopH62AT0G09vIGXw/formResponse 13/28
1/3/2021 EXAMEN FINAL 1° DE MARZO 2021

En la autopsia de un masculino de 46 años que falleció por traumatismo


encefalocraneano, se encontró en el hígado, una lesión de bordes irregulares,
pero bien delimitados, pálida, con una cicatriz estrellada deprimida blanco-
grisácea desde la que irradian tabiques fibrosos a la periferia. La histología
evidenció hepatocitos separados por gruesas trabéculas sinusoidales y vasos
arteriales que poseen hiperplasia fibromuscular con estrechamiento excéntrico
de la luz. ¿Cuál es para ud. el diagnóstico más probable de la lesión hepática? *

Un adenoma hepatocelular

Un colangiocarcinoma

Una hiperplasia nodular focal

Un hepatoblastoma

https://docs.google.com/forms/d/e/1FAIpQLSftEX0-83LIWJqKuSQUO8i0JvRYipp5ImopH62AT0G09vIGXw/formResponse 14/28
1/3/2021 EXAMEN FINAL 1° DE MARZO 2021

Una mujer de 77 años ha presentado episodios sincopales durante el último mes


en respuesta al esfuerzo. Al examen físico se encuentra afebril, FC:
66latidos/min., FR: 14respiraciones/min., TA: 125/85mmHg., se ausculta un soplo
eyectivo sistólico y algunos crepitantes en las bases pulmonares posteriores.
Observe la imagen macroscópica e indique cuales de los siguientes factores es
más probable que haya contribuido a su desarrollo. *

Lupus eritematoso sistémico

Sífilis terciaria

Hipercalcemia de las neoplasias malignas

Envejecimiento

https://docs.google.com/forms/d/e/1FAIpQLSftEX0-83LIWJqKuSQUO8i0JvRYipp5ImopH62AT0G09vIGXw/formResponse 15/28
1/3/2021 EXAMEN FINAL 1° DE MARZO 2021

En referencia al “Carcinoma de Vulva” señale cual/es de los siguientes


enunciados es CORRECTO. Recuerde que deberá MARCAR TODAS LAS
OPCIONES CORRECTAS, considerando que al marcar opciones correctas de
menos o 1 respuesta incorrecta, dará por anulado el ejercicio: *

El grupo de carcinoma basaloide y verrugoso es el más frecuente.

Los carcinomas epidermoides queratinizantes (70%) no estan vinculados a la


infección por HPV, son más frecuentes y afectan a mujeres de mayor edad.

La VIN clásica está constituída por una marcada atipía de la capa basal del epitelio
escamoso y diferenciación normal de las capas más superficiales.

El grupo de carcinoma basaloide y verrugoso se presenta a una edad más temprana y


se asocia con HPV.

El grupo de carcinoma epidermoide queratinizante no relacionado con HPV se


presenta a una edad más temprana.

El tipo epidermoide queratinizante es más frecuente en mujeres con liquen escleroso


o hiperplasia de células escamosas de larga evolución.

Un hombre de 50 años, fumador de 200 paquetes/año, ha desarrollado obesidad


central, lumbalgia y equimosis de fácil aparición en los últimos 5 meses. Se
encuentra afebril y la TA es de 160/95 mmHg. La Rx. tórax identifica una masa mal
delimitada en hilio izquierdo de 4 cm. El estudio citológico del lavado bronquial
obtenido mediante broncoscopia muestra células redondeadas de 25 µm de
diámetro, con escaso citoplasma. Se incluye el material y se realiza
inmunohistoquímica expresando TTF-1 (+), sinaptofisina (+) y CD56 (+). Se
informa al paciente que, aunque su enfermedad parece localizada de un solo
lado de la cavidad torácica, el tratamiento quirúrgico no conseguiría curarlo.
¿Cuál de las siguientes neoplasias es más probable que tenga este paciente? *

Carcinoma de células grandes


https://docs.google.com/forms/d/e/1FAIpQLSftEX0-83LIWJqKuSQUO8i0JvRYipp5ImopH62AT0G09vIGXw/formResponse 16/28
1/3/2021 EXAMEN FINAL 1° DE MARZO 2021

Linfoma no Hodgkin T

Carcinoide atípico bronquial

Carcinoma de células pequeñas

Consulta un paciente de 60 años por dolor suprapúbico de larga data


relacionado a polaquiuria y tenesmo vesical. Refiere tener Hiperplasia prostática
benigna diagnosticada hace 5 años, pero nunca se adecuó al tratamiento. No
refiere antecedentes de relevancia, no toma medicamentos y niega consumir
drogas, entre ellas, tabaco. Si ud. le efectuara una cistoscopia, ¿qué esperaría
encontrar? *

Tumoración exofítica de coloración heterogénea, friable, fuertemente adherida a la


pared que ocluye la uretra casi en su totalidad.

Atrofia de la mucosa vesical con ulceraciones sangrantes.

Aumento del tamaño vesical, hipertrofia de la pared con trabeculaciones que forman
criptas las cuales pueden evolucionar a divertículos.

Lesión excéntrica que ocupa más del 50% de la vejiga, de superficie irregular y friable
que se une a la pared vesical por un tallo fibrovascular.

En referencia a la entidad nosológica “Carcinoma Urotelial papilar de bajo grado”,


indique la afirmación CORRECTA: *

Este tipo de tumores se caracterizan por la alta frecuencia de recidivas e invasión.

Sin tratamiento el 75% progresa a carcinoma in situ.

Las células neoplásicas que lo componen presentan alta cohesividad y se hallan


distribuidas homogéneamente manteniendo la polaridad.

Las células neoplásicas presentan marcada atipia con figuras mitóticas frecuentes a
predominio basal.

https://docs.google.com/forms/d/e/1FAIpQLSftEX0-83LIWJqKuSQUO8i0JvRYipp5ImopH62AT0G09vIGXw/formResponse 17/28
1/3/2021 EXAMEN FINAL 1° DE MARZO 2021

Paciente de 57 años, tabaquista de larga data, que consulta por disfonía. Se le


realiza una laringoscopia, observándose una lesión exofítica ulcerada sobre la
cuerda vocal derecha. La biopsia informa: carcinoma. Con respecto a este tipo
de lesión laríngea, marque la opción INCORRECTA: *

Frecuentemente se originan en los llamados nódulos laríngeos, nódulos reactivos o


pólipos, relacionados con grandes esfuerzos laríngeos.

El tabaquismo es el principal factor de riesgo, siendo el peligro proporcional al grado


de exposición.

La mucosa adyacente a la lesión infiltrante puede mostrar hiperplasia, displasia o


carcinoma in situ.

Muy probablemente se trate de un carcinoma epidermoide.

https://docs.google.com/forms/d/e/1FAIpQLSftEX0-83LIWJqKuSQUO8i0JvRYipp5ImopH62AT0G09vIGXw/formResponse 18/28
1/3/2021 EXAMEN FINAL 1° DE MARZO 2021

Un hombre de 76 años ha presentado un nódulo en el párpado derecho que ha


ido aumentando de tamaño lentamente durante los últimos 4 años. En el examen
físico se observa un nódulo de 0.3cm., perlado con telangiectasias superficiales,
en la región malar derecha. Se extirpa la lesión. La imagen muestra el aspecto
histológico de la lesión. ¿Cuál es el diagnóstico más probable? *

Melanoma amelanótico

Carcinoma basocelular

Nevo nevocelular

Queratosis actínica

https://docs.google.com/forms/d/e/1FAIpQLSftEX0-83LIWJqKuSQUO8i0JvRYipp5ImopH62AT0G09vIGXw/formResponse 19/28
1/3/2021 EXAMEN FINAL 1° DE MARZO 2021

En referencia a las "Gastropatías hipertróficas", señale cual/es de los siguientes


enunciados es CORRECTO. Recuerde que deberá MARCAR TODAS LAS
OPCIONES CORRECTAS, considerando que el marcar opciones correctas de
menos o 1 respuesta incorrecta, dará por anulado el ejercicio. *

La neoplasia endócrina múltiple tipo IIA es un factor de riesgo para la aparición de la


enfermedad de Ménétrier.

El 25% de los pacientes con gastrinomas tienen una neoplasia endocrina múltiple tipo
I (NEM-I).

Las gastritis hipertróficas agrupan varias entidades como la gastritis autoinmunitaria,


la gastritis eosinofílica, la gastritis linfocítica y la gastritis granulomatosa.

Las gastropatías hipertróficas están vinculadas a una liberación excesiva del factor
de crecimiento transformante α (TGF- α).

La enfermedad de Ménétrier se caracteriza por hiperplasia difusa del epitelio foveolar


del cuerpo y fondo gástrico e hipoproteinemia.

El síndrome de Zollinger-Ellison se debe a tumores secretores de gastrina


(gastrinomas).

https://docs.google.com/forms/d/e/1FAIpQLSftEX0-83LIWJqKuSQUO8i0JvRYipp5ImopH62AT0G09vIGXw/formResponse 20/28
1/3/2021 EXAMEN FINAL 1° DE MARZO 2021

Una niña de 14 años sufre dolor intenso en la pierna derecha después de realizar
un ejercicio gimnástico de suelo. Al examen físico presenta dolor a la palpación
en la parte inferior del muslo derecho por encima de la rodilla. En la Rx se
visualiza un área de fractura que atraviesa una lesión diafisaria de 3cm. en la
parte distal del fémur. Esta lesión se ve bien delimitada, totalmente intramedular
y presenta una zona clara central rodeada de hueso esclerótico. Se obtiene una
biopsia y el estudio histopatológico informa: “Trabéculas curvas de hueso
reticular sobre un fondo de proliferación fibroblástica”. Observe la imagen e
indique cual es el diagnóstico más probable. *

Callo de fractura

Osteosarcoma

Displasia fibrosa

Sarcoma de Ewing

https://docs.google.com/forms/d/e/1FAIpQLSftEX0-83LIWJqKuSQUO8i0JvRYipp5ImopH62AT0G09vIGXw/formResponse 21/28
1/3/2021 EXAMEN FINAL 1° DE MARZO 2021

En referencia al “Melanoma uveal", señale cual/es de los siguientes enunciados es


CORRECTO. Recuerde que deberá MARCAR TODAS LAS OPCIONES
CORRECTAS, considerando que al marcar opciones correctas de menos o 1
respuesta incorrecta, el sistema dará por anulado el ejercicio. *

El sitio principal de metástasis es el cerebro y, en segundo lugar, los pulmones.

El tamaño tumoral y el fenotipo de células tumorales fusiformes constituyen factores


de mal pronóstico.

Es el tumor primario ocular más frecuentemente observado en el adulto.

Su principal vía de diseminación es la linfática por lo cual luego del diagnóstico se


requiere vaciamiento ganglionar o técnica del ganglio centinela.

Presentan vías moleculares de patogénesis distintas que los melanomas cutáneos.

Aquellos ubicados en el iris presentan mejor pronóstico que los del cuerpo ciliar y
coroides.

https://docs.google.com/forms/d/e/1FAIpQLSftEX0-83LIWJqKuSQUO8i0JvRYipp5ImopH62AT0G09vIGXw/formResponse 22/28
1/3/2021 EXAMEN FINAL 1° DE MARZO 2021

Un paciente de 45 años presenta cólicos renales frecuentes, diagnosticándose


Nefrolitiasis recidivante, bilateral, con cálculos de fosfato cálcico. Presenta
además dolores óseos generalizados con deformaciones óseas. Los análisis de
laboratorio muestran hipercalcemia, hipofosfatemia, hipercalciuria, e
hiperfosfaturia y aumento de la concentración de parathormona en sangre. En
glándulas paratiroides se encuentra una “masa redondeada en la glándula
superior izquierda, la cual es extirpada. En el examen histológico se observa un
nódulo compuesto por células poligonales uniformes con núcleos centrales,
pequeños, rodeado por una cápsula fibrosa, en el borde de la muestra se
observa un anillo de tejido paratiroideo comprimido”. ¿Cuál es para Ud. el
diagnóstico más probable? *

Carcinoma paratiroideo

Hiperparatiroidismo secundario

Adenoma paratiroideo

Hiperplasia paratiroidea

https://docs.google.com/forms/d/e/1FAIpQLSftEX0-83LIWJqKuSQUO8i0JvRYipp5ImopH62AT0G09vIGXw/formResponse 23/28
1/3/2021 EXAMEN FINAL 1° DE MARZO 2021

Un paciente de 25 años es intervenido quirúrgicamente realizándose una


colectomía total. Se presenta el aspecto macro y microscópico de la pieza
quirúrgica. ¿Cuál de las siguientes condiciones genéticas es más probable que
esté asociada con su patología? *

Síndrome del carcinoma colónico no polipoide hereditario

Poliposis adenomatosa familiar

Síndrome de Peutz-Jeghers

Síndrome de Gardner
https://docs.google.com/forms/d/e/1FAIpQLSftEX0-83LIWJqKuSQUO8i0JvRYipp5ImopH62AT0G09vIGXw/formResponse 24/28
1/3/2021 EXAMEN FINAL 1° DE MARZO 2021
Síndrome de Gardner

En referencia a la "Patología Tiroidea", señale cual/es de los siguientes enunciados


es CORRECTO. Recuerde que deberá MARCAR TODAS LAS OPCIONES
CORRECTAS, considerando que el marcar opciones correctas de menos o 1
respuesta incorrecta, dará por anulado el ejercicio. *

El carcinoma folicular de tiroides tiende a metastatizar por vía hematógena a


pulmones hueso o hígado y raramente lo hace vía linfática a los ganglios
locorregionales del cuello.

En la tiroiditis de Hashimoto se observa abundante infiltrado inflamatorio


polimorfonuclear.

Los carcinomas primarios de tiroides más frecuentes son los carcinomas foliculares.

Los adenomas tiroideos suelen ser masas múltiples.

En la enfermedad de Graves-Basedow la presencia de anticuerpos inhibidores de la


actividad de las células epiteliales tiroideas se asocia a un riesgo aumentado de
padecer enfermedades autoinmunes.

La tiroiditis de Riedel muestra extensas áreas de fibróticas que afectan a la tiroides y


las estructuras cervicales contiguas, dando el aspecto de una masa tiroidea dura y
fija que puede simular clínicamente un carcinoma de tiroides.

Llega a la consulta una persona con su hijo de 3 años. Se lo nota preocupado al


padre porque refiere que su hijo hace un mes que no quiere jugar, que lo nota
cansado, lo ve algo más pálido. En la anamnesis se resalta que está con
sangrados leves-moderados, y en el último año tuvo 5 veces otitis, de causa
infecciosa. Se sospecha de una patología hemolinfoide. ¿Cuál es para usted el
diagnóstico más probable? *

Leucemia/linfoma linfoblástico agudo.

https://docs.google.com/forms/d/e/1FAIpQLSftEX0-83LIWJqKuSQUO8i0JvRYipp5ImopH62AT0G09vIGXw/formResponse 25/28
1/3/2021 EXAMEN FINAL 1° DE MARZO 2021

Leucemia mieloide aguda.

Leucemia mieloide crónica.

Síndrome mielodisplásicos.

En referencia a la entidad “Aneurismas”, indique la opción CORRECTA: *

La mayoría de los casos de disección aórtica se presenta en adultos jóvenes con


anomalías del tejido conectivo.

No se observa relación de los aneurismas de aorta abdominal con otras afecciones


como cardiopatía isquémica o accidentes cerebrovasculares.

La aorta doble cañón se relaciona con hemorragias extraaórticas mortales ya que


genera falsos dobles conductos.

Su patogenia se relaciona con síntesis inadecuada del tejido conjuntivo, degradación


excesiva del conjuntivo y pérdida de células musculares lisas.

Una mujer de 43 años refiere hemorragia poscoital de 6 meses de evolución.


Tuvo la menarca a los 11 años y a lo largo de su vida ha tenido 12 parejas sexuales.
Sus ciclos menstruales son regulares sin hemorragias intermenstruales
anómalas. La colposcopía demuestra una lesión exofítica, heterogénea y
ulcerada. El PAP revela, HSIL. Al analizar la citología cervical se halla la presencia
del serotipo HPV 16. ¿Cuál de los siguientes tumores malignos es más probable
que desarrolle esta paciente si no es tratada? *

Sarcomoa botriodeo

Adenocarcinoma

Carcinoma epidermoide

Leiomiosarcoma

https://docs.google.com/forms/d/e/1FAIpQLSftEX0-83LIWJqKuSQUO8i0JvRYipp5ImopH62AT0G09vIGXw/formResponse 26/28
1/3/2021 EXAMEN FINAL 1° DE MARZO 2021

En referencia a las "Neoplasias Renales", señale cual/es de los siguientes


enunciados es CORRECTO. Recuerde que deberá MARCAR TODAS LAS
OPCIONES CORRECTAS, considerando que el marcar opciones correctas de
menos o 1 respuesta incorrecta, dará por anulado el ejercicio. *

La importancia clínica del angiomiolipoma, radica, principalmente, en su tendencia a


la hemorragia espontánea.

Un 5% de los tumores de Wilms muestran anaplasia, cuya presencia se correlaciona


con la existencia de mutaciones de TP53 y la aparición de resistencia frente a la
quimioterapia.

El carcinoma cromófobo renal es uno de los menos frecuentes y se encuentra


formado por células eosinófilas pálidas cuyas membranas celulares son prominentes
y a menudo con un halo perinuclear.

Los tumores de la pelvis renal suelen dar signos tardíos y generalmente han generado
metástasis a distancia al momento del diagnóstico.

El tumor de Wilms, a la microcospía, se caracteriza por ser un tumor bifásico.

El carcinoma de células claras renal es el tumor maligno más frecuente del riñón.
Suele presentarse como lesiones solitarias unilaterales y se origina del túbulo
contorneado proximal.

https://docs.google.com/forms/d/e/1FAIpQLSftEX0-83LIWJqKuSQUO8i0JvRYipp5ImopH62AT0G09vIGXw/formResponse 27/28
1/3/2021 EXAMEN FINAL 1° DE MARZO 2021

En referencia a la "Cardiopatía isquémica", señale cual/es de los siguientes


enunciados es CORRECTO. Recuerde que deberá MARCAR TODAS LAS
OPCIONES CORRECTAS, considerando que el marcar opciones correctas de
menos o 1 respuesta incorrecta, dará por anulado el ejercicio. *

El infarto de miocardio se diagnostica en virtud de los síntomas, los cambios


electrocardiográficos y la medición de la CK-MB y las troponinas. Las lesiones macro
y microscópicas siguen una cronodinamia que se desarrolla tras horas o días.

A las tres horas de producido el infarto de miocardio, microscópicamente se observa


un infiltrado inflamatorio intersticial de neutrófilos.

La arteria coronaria más frecuentemente afectada cuya lesión desencadena un


infarto de miocardio es la circunfleja izquierda.

La característica común de los síndromes coronarios agudos es la isquemia


miocárdica anterógrada.

El término angina inestable hace referencia exclusiva al patrón de angina o molestia


torácica causada por espasmo arterial coronario.

Entre las complicaciones del infarto se encuentran la rotura ventricular, rotura del
músculo papilar, la formación de aneurismas, trombo mural, arritmias, la pericarditis y
la insuficiencia cardíaca tardía progresiva.

Atrás Enviar

Nunca envíes contraseñas a través de Formularios de Google.

Este formulario se creó en Facultad de Medicina. Noti car uso inadecuado

Formularios

https://docs.google.com/forms/d/e/1FAIpQLSftEX0-83LIWJqKuSQUO8i0JvRYipp5ImopH62AT0G09vIGXw/formResponse 28/28
EXAMEN FINAL 22 DE FEBRERO 2021
*Obligatorio

Instrucciones

RECUERDE:
- En las preguntas tipo múltiple choice, 1 sola opción es la correcta.
- En las preguntas tipo casilla de verificación, deberá identificar todos los enunciados CORRECTOS
teniendo en cuenta que el marcar enunciados correctos de menos o enunciados incorrectos, la pregunta
se da automáticamente por anulada.

Una mujer de 25 años consulta a su médico por presentar un ganglio linfático


cervical indoloro, incluso a la palpación, que no disminuye de tamaño desde hace
3 semanas. El profesional al revisarla encuentra otras adenopatías de menor
tamaño próximas a la referida y también en el sector supraclavicular del mismo
lado. Sin demora, solicita una biopsia escisional de uno de los ganglios
comprometidos que el patólogo informa como "Linfoma de Hodgkin, variante
esclerosis nodular". Respecto de este caso, responda cuál es la opción
INCORRECTA: *

Las células linfoides neoplásicas son positivas para CD45, CD15 y CD30, y negativas
para CD3 y CD20.

Es una de las formas clásicas de la enfermedad, y la más frecuente entre ellas.

No suele asociarse a infección por el virus Epstein-Barr.

Con el tratamiento adecuado y oportunamente instaurado, la entidad suele ser de


muy buen pronóstico.
En referencia a la entidad “Esclerosis múltiple", señale cual de los siguientes
enunciados es INCORRECTO. *

La afectación del tronco encefálico y la médula espinal, forman parte de la evolución


natural de la enfermedad.

En pacientes asintomáticos se han detectado placas silentes a nivel de la RMN y la


autopsia.

Es considerada una enfermedad desmielinizante de origen autoinmunitario en la cual


se encuentran implicados factores genéticos y ambientales.

Se caracteriza por episodios reiterativos de déficit neurológico atribuibles


fundamentalmente a lesiones de la sustancia gris.

En referencia a la entidad “cardiomiopatía hipertrófica”, indique la opción


INCORRECTA: *

Suele cursar con dilatación de la cavidad ventricular.

Alrededor del 25% de los pacientes presenta obstrucción dinámica del tracto de
salida del ventrículo izquierdo.

Es una patología de causa genética, principalmente relacionada con mutaciones en


genes que codifican proteínas sarcoméricas.

En la microscopía óptica, es característico observar “desorganización miofibrilar”


(disarray).
En referencia a los "Tumores Hipofisarios", señale cual/es de los siguientes
enunciados es CORRECTO. Recuerde que deberá marcar todas las opciones
correctas, considerando que el marcar opciones correctas de menos o 1
respuesta incorrecta, dará por anulado el ejercicio. *

La causa más frecuente de hiperpituitarismo son los carcinomas hipofisarios.

Un subgrupo de adenomas hipofisarios muestra actividad aumentada del Ki67


constituyendo adenomas atípicos con mayor tendencia a la invasión y recidiva.

Los adenomas hipofisarios tirotropos constituyen una causa infrecuente de


hipertiroidismo y representan el 1% de todos los adenomas hipofisarios.

El hallazgo de hiperprolactinemia se observa en diferentes situaciones, algunas de


ellas fisiológicas como el embarazo o lactancia, otras patológicas como el
prolactinoma, hipotiroidismo o traumatismo craneal y otras farmacológicas.

El Sindrome de Sheehan se caracteriza por la necrosis isquémica de la glándula


pituitaria en el período postparto.

Considerando la totalidad de los tumores hipofisarios se establece que el 80% son


adenomas hipofisarios no funcionantes.

Un paciente de 83 años se interna para realizarse una cistoprostatectomia


debido a un tumor urotelial de alto grado de malignidad. En el postoperatorio
desarrolló un cuadro febril prolongado falleciendo a los pocos días. Se realiza la
autopsia y se obtienen los siguientes resultados: “Embolia e infarto cerebral. A
nivel cardíaco: presencia de pequeñas vegetaciones blandas(<0.5cm) en el
margen libre de la válvula aortica. Las vegetaciones se encontraban compuestas
por fibrina y plaquetas”. ¿Cuál es para Ud. el diagnóstico más probable? *

Endocarditis reumática crónica

Endocarditis trombótica no bacteriana

Embolia neoplásica cerebral

Complicación de una placa ateromatosa cerebral


En referencia a la “Patología tumoral mamaria” señale cual/es de los siguientes
enunciados es CORRECTO. Recuerde que deberá marcar todas las opciones
correctas considerando que al marcar opciones correctas de menos o 1
respuesta incorrecta, dará por anulado el ejercicio: *

El carcinoma lobulillar invasor crece en pequeños nidos con fuerte expresión de E-


Cadherina.

La enfermedad de Paget mamaria consiste en la migración intraepitelial de células


neoplásicas desde un foco de carcinoma ductal in situ (CDIS), hasta el pezón.

El riesgo de recidiva del CDIS depende de: alto grado nuclear, extensión de la
enfermedad y compromiso de márgenes quirúrgicos.

Los carcinomas negativos para receptores de estrógeno, receptores de progesterona,


y Her2neu ("triple negativos"), son más frecuentes en mujeres premenopáusicas.

El carcinoma ductal in situ tiene mayor riesgo de bilateralidad que el carcinoma


lobulillar in situ (CLIS).

Los carcinomas asociados a expresión de receptores de estrógenos representan un


grupo infrecuente y agresivo de cáncer de mama, pero susceptible de tratamiento con
tamoxifeno.

Una mujer de 39 años presenta un nódulo en la espalda que ha crecido durante


los últimos 2 meses. En la exploración física se observa una lesión pigmentada de
2,1cm., de márgenes irregulares con áreas de pigmentación heterogénea pardas
a negruzcas. Se realiza una biopsia escisional con márgenes amplios y el estudio
histopatológico de la misma identifica un melanoma constituido por células
epitelioides, que se extienden 2mm. dentro de la dermis reticular. Se observa una
banda de linfocitos por debajo de la lesión. ¿Cuál de los siguientes factores es
más importante para determinar el pronóstico de esa paciente? *

Localización en la piel

Extensión del crecimiento radial

Edad en el momento del diagnóstico


Profundidad de la lesión

Paciente con una historia progresiva de 8 años de evolución, a partir de los 70


años, de deterioro intelectual, errores inexplicables en la actividad cotidiana,
descuido en la higiene personal, que lleva al enfermo a una dependencia
absoluta de sus familiares, con inmovilidad total, incontinencia de esfínteres,
pérdida de peso, convulsiones que lo llevaron a la muerte. En la autopsia se
observaron en el cerebro placas neuríticas y ovillos neurofibrilares. Podremos
establecer un diagnóstico de: *

Encefalopatía espongiforme de Creutzfeldt-Jacob por "proteína prión"

Enfermedad de Alzheimer

Degeneración hepatolenticular o enfermedad de Wilson

Enfermedad de Parkinson

En referencia a la “Patología hepática y de los conductos biliares", señale cual/es


de los siguientes enunciados es CORRECTO. Recuerde que deberá marcar todas
las opciones correctas, considerando que al marcar opciones correctas de
menos o 1 respuesta incorrecta, el sistema dará por anulado el ejercicio. *

La característica histológica de la colangitis ascendente es la infiltración de


neutrófilos periductulillares de forma directa hacia el epitelio y la luz del conducto
biliar.

Una vez que la cirrosis está establecida, es muy dificultoso el diagnóstico etiológico
basándose sólo en los hallazgos morfológicos.

La forma más grave de colangitis es la colangitis supurativa, en la que la bilis


purulenta rellena y distiende los conductos biliares.

La causa más frecuente de obstrucción de los conductos biliares en los adultos es la


presencia de tumores malignos biliares.

El virus de la hepatitis B (VHB) es un virus ARN de la familia Flaviviridae que produce


una hepatitis aguda que evoluciona en un 80% de los casos a una hepatopatía
crónica.
La cirrosis afecta en forma difusa a todo el hígado, que aparece constituído por
nódulos cicatrizales densos rodeados por parénquima regenerativo y grados
variables de comunicación vascular.

En referencia a los "Tumores de vejiga", señale cual/es de los siguientes


enunciados es CORRECTO. Recuerde que deberá marcar todas las opciones
correctas, considerando que el marcar opciones correctas de menos o 1
respuesta incorrecta, dará por anulado el ejercicio. *

Un tumor de vejiga estadificado T2 corresponde a la invasión de la muscular propia.

Los papilomas representan el 1% o menos de los tumores de vejiga y normalmente se


ven en pacientes más jóvenes.

Histológicamente el 95% de los tumores de vejiga son de estirpe mesenquimatosa.

El carcinoma de vejiga es más frecuente en hombres que en mujeres y se relaciona


con el hábito tabáquico.

En su gran mayoría, las infecciones crónicas por Schistosoma Haematobium originan


cánceres glandulares.

En conjunto, la mayoría de los tumores papilares son de bajo grado.

En referencia a la “psoriasis", señale cual de los siguientes enunciados es


INCORRECTO. *

Es una dermatosis inflamatoria crónica con una probable base autoinmunitaria.

Es una dermatosis que produce escamas blanquecinas sobre placas salmón-rosado


bien delimitadas, en los planos de extensión (codo, rodillas), la nuca, cuero cabelludo
y pliegues interglúteos.

Las lesiones establecidas de psoriasis presentan un cuadro histológico característico


en el cual la acantosis observada es a expensas de un marcado aumento del estrato
granuloso.

Los neutrófilos dentro del estrato córneo paraqueratósico constituyen el Signo de


Auspitz.
Una paciente de 40 años concurre a la consulta porque se ha palpado un nódulo
en la tiroides. Usted solicita una punción por aspiración con aguja fina. A la
citología, observa múltiples células foliculares atípicas, por lo que decide realizar
una biopsia. ¿Qué elementos de la biopsia deberá tener en cuenta para poder
realizar el diagnóstico de certeza? *

Presencia de células de Hürthle

Invasión capsular y vascular

Fibrosis

Atipía marcada
Un paciente de 43 años de edad, con hepatitis crónica por virus C, consultó por
presentar edema generalizado. En los exámenes de laboratorio se constató
proteinuria masiva, hipoproteinemia e hipoalbuminemia. Se le realizó biopsia
renal,¿Qué elementos morfológicos son los que permiten justificar el diagnóstico
de Glomerulopatía membranosa? *

glomérulos con áreas de esclerosis, hialinosis focal y segmentaria.


Inmunofluorescencia positiva focal para IgM y C3. Depósitos subepiteliales en la
microscopía electrónica.

glomérulos con proliferación mesangial. Inmunofluorescencia positiva para IgA, IgG,


IgM y C3, en mesangio. Depósitos mesangiales en la microscopía electrónica.

glomérulos con engrosamiento difuso de las paredes capilares. Imnunofluorescencia


positiva para IgG y C3 granular con patrón difuso intramembranoso. Depósitos
subepiteliales en la microscopía electrónica.

glomérulos aumentados de tamaño, con proliferación difusa de células endoteliales,


mesangiales y epiteliales, con infiltrados polimorfonucleares y algunos monocitos.
Inmunofluorescencia positiva para IgG y C3 granular en mesangio. Depósitos
subepiteliales grandes en forma de joroba en la microscopía electrónica.
Paciente de 70 años de edad que se halla en estudio por alteración en el hábito
intestinal se le diagnostica por colonoscopía un tumor exofítico de 5cm. de
diámetro en colon sigmoides. La biopsia informa “adenocarcinoma bien
diferenciado infiltrante; no se observan otras lesiones en la mucosa del resto del
colon”. El paciente es intervenido quirúrgicamente. De las lesiones enumeradas a
continuación, ¿Cuál cree Ud. que puede haber sido precursora del
adenocarcinoma actual con mayor probabilidad? *

Adenoma velloso

Pólipo carcinoide

Pólipo hiperplásico

Pólipo juvenil o hamartomatoso

En referencia a las “Anemias hemolíticas”, señale cuales de los siguientes


enunciados es CORRECTO. Recuerde que deberá marcar todas las opciones
correctas, considerando que el marcar correctas de menos o 1 respuesta
incorrecta, dará por anulado el ejercicio. *

En la hemólisis crónica, la excreción biliar elevada de bilirrubina, favorece la


formación de litos biliares pigmentados.

La fisiopatología de la anemia de las células falciformes está relacionada con la


oclusión vascular causada por la formación de las células falciformes dentro de los
lechos microvasculares.

La eritroblastosis fetal es compatible con la vida intrauterina sin requerir tratamiento.

En la esferocitosis hereditaria la gran mayoría de los pacientes son asintomáticos.

En las anemias hemolíticas se produce un incremento compensador de la


eritropoyesis dando lugar a la reticulocitosis en sangre periférica.

La trombosis es la principal causa de muerte relacionada con la hemoglobinuria


paroxística nocturna.
En referencia a las "Vasculitis", señale cual/es de los siguientes enunciados es
CORRECTO. Recuerde que deberá marcar todas las opciones correctas,
considerando que el marcar opciones correctas de menos o 1 respuesta
incorrecta, dará por anulado el ejercicio. *

La tromboangeítis obliterante compromete arterias de mediano y pequeño calibre, a


predominio de las tibiales y radiales, produciendo insuficiencia vascular pudiendo
progresar con ulceraciones crónicas y gangrena de las extremidades. No se conocen
los factores de riesgo asociados a esta entidad.

La enfermedad de Kawasaki en una vasculitis necrosante que afecta generalmente a


los capilares, arteriolas y vénulas, pudiendo asociarse con glomerulonefritis y otros
trastornos inmunitarios.

La arteritis de Takayasu es una vasculitis granulomatosa que afecta


característicamente el cayado aórtico y se conoce también como “enfermedad sin
pulso” debido al debilitamiento del pulso en las extremidades superiores.

Las vasculitis asociadas a ANCA son denominadas “pauciinmunitarias” dado que los
autoanticuerpos citoplásmicos anti-neutrófilos (ANCA) son dirigidos contra
componentes celulares y no forman inmunocomplejos circulantes.

El síndrome de Churg-Strauss es una vasculitis necrosante de vasos pequeños que se


suele presentar como una enfermedad multisistémica con afectación cutánea,
hemorragia digestiva, enfermedad renal y afectación cardíaca.

La afectación cardíaca en el Síndrome de Churg-Strauss se caracteriza por una


miocarditis con infiltrado a predominio de linfocitos y células plasmáticas.

¿Cuál de los siguientes enunciados es INCORRECTO en referencia a los


Mesoteliomas? *

El mesotelioma maligno localizado presenta un crecimiento nodular circunscripto y


mejor pronóstico con respecto al tipo difuso.

El 90% de los mesoteliomas malignos difusos presentan antecedentes de exposición


al asbesto con un período de latencia entre 20-40 años.

El mesotelioma maligno epitelioide debe diferenciarse (a nivel inmunohistoquímico)


del carcinoma escamoso dado que ambos comparten una localización y clínica
semejante.

El mesotelioma maligno difuso invade por contigüidad a diafragma o a pared torácica


y a través de las cisuras al pulmón.

En un control de rutina una paciente de 30 años de edad recibe un informe de


citología exfoliativa vinculable a SIL de alto grado. En la colposcopia se identificó
una leucoplasia y se tomó biopsia. La misma mostró “alteración
histoarquitectural del epitelio con presencia de atipias y mitosis en el estrato
intermedio”. ¿Cuál es para Ud. el diagnóstico más probable? *

CIN I (SIL de bajo grado)

Carcinoma in situ

CIN III (SIL de alto grado)

CIN II (SIL de alto grado)

En referencia a la “Patologia respiratoria” señale cual/es de los siguientes


enunciados es correcto. Recuerde que deberá marcar todas las opciones
correctas considerando que al marcar opciones correctas de menos o 1
respuesta incorrecta, dará por anulado el ejercicio: *

La Enfermedad Pulmonar Obstructiva Crónica (EPOC) se caracteriza por la limitación


al flujo aéreo durante la inspiración.

La apnea obstructiva del sueño es un trastorno frecuente que se asocia a obesidad e


hipoxemia y puede ser un factor contribuyente al desarrollo de HTP y corazón
pulmonar.

La hiperplasia adenomatosa atípica se considera como una lesión precursora que


mide menos de 5mm y que se caracteriza histológicamente por poseer neumocitos
displásicos que revisten las paredes alveolares acompañado de ligera fibrosis
intersticial.

El patrón histológico de la neumonía intersticial usual consiste en áreas parcheadas


en las cuales las zonas afectadas se caracterizan por lesiones en diferentes
momentos evolutivos con focos fibroblásticos, fibrosis densa y panalización.

Habitualmente el carcinoma de laringe surge a partir de un papiloma previo.

La carencia de alfa-1 antitripsina está asociada al desarrollo del enfisema panacinar.


Marque la INCORRECTA en referencia a la entidad infecciosa relacionada con el
SARS-COV2: *

En el contexto de la respuesta inmune a la infección por SARS-COV2 se identifican a


las IL-1β, IL-6 y TNFα como moléculas beneficiosas.

El SARS-COV2 pertenece a un grupo de coronavirus al igual que el MERS-COV y SARS-


COV1.

El receptor de ACE2(enzima convertidora de angiotensina) se encuentra distribuido a


nivel alveolar, renal, miocárdico, intestino delgado y células endoteliales.

Los microtrombos, la capilaritis, los infiltrados subepiteliales y la bronconeumonía


bacteriana, son hallazgos posibles en autopsias de pacientes COVID (+).

En referencia a la entidad “Lesión tubular aguda” indique la opción INCORRECTA:


*

La lesión tubular aguda nefrotóxica puede ser morfológicamente inespecífica o


evidenciar la causa que le dio origen.

La fisiopatología de la lesión tubular aguda demuestra que invariablemente su origen


es la disminución del flujo arterial renal.

El evento central en la lesión tubular aguda isquémica, está dado por el efecto
vasoconstrictor de la endotelina y la disminución de ON (óxido nítrico) y prostaciclina.

En la lesión renal aguda isquémica, la intensidad de los hallazgos morfológicos puede


no correlacionarse con la intensidad de las manifestaciones clínicas.
En referencia a la "Patología tumoral ósea y de partes blandas”, señale cual/es de
los siguientes enunciados es CORRECTO. Recuerde que deberá marcar todas las
opciones correctas, considerando que al marcar opciones correctas de menos o
1 respuesta incorrecta, el sistema dará por anulado el ejercicio. *

El sarcoma sinovial deriva de los sinoviocitos.

Los tumores de la familia del sarcoma de Ewing habitualmente se localizan en la


diáfisis de los huesos tubulares largos.

El osteosarcoma se caracteriza por una proliferación de células osteoblásticas


malignas, pleomórficas, y presencia de matriz osteoide o de hueso maduro.

Pacientes con poliposis adenomatosa familiar con mutaciones APC de la línea


germinal son propensos a padecer fibromatosis profunda.

El sarcoma botrioides es una variante de leiomiosarcoma que se ubica en cavidades


que se presenta en adultos jóvenes.

La exostosis es una alteración consistente en crecimiento de hueso laminar con un


recubrimiento de cartílago hialino, que prolifera hasta generar células con marcada
atipía.
La imagen corresponde al estudio histológico a gran aumento del aspecto de los
riñones de una paciente adulta de 63 años, hipertensa y diabética que ha sufrido
varios episodios isquémicos transitorios cerebrales en los últimos dos meses.
¿Cuál de las siguientes lesiones renales es más probable que sufra esta paciente?
*

Necrosis tubular aguda

Angiomiolipoma renal

Nefritis intersticial

Arterioloesclerosis hialina

En referencia a la” Patología pancreática” indique la opción CORRECTA: *

En la pancreatitis aguda el epitelio ductal puede presentar sectores con metaplasia


escamosa.

La pancreatitis crónica es una lesión reversible del parénquima exocrino y endocrino.

Un alto porcentaje de las pancreatitis crónicas llevan una base genética.

La forma más leve de pancreatitis aguda es la pancreatitis aguda intersticial que


La forma más leve de pancreatitis aguda es la pancreatitis aguda intersticial que
presenta áreas focales de necrosis grasa en páncreas y grasa peri pancreática.

En referencia a la "Patología tumoral cutánea”, señale cual/es de los siguientes


enunciados es CORRECTO. Recuerde que deberá marcar todas las opciones
correctas, considerando que al marcar opciones correctas de menos o 1
respuesta incorrecta, el sistema dará por anulado el ejercicio. *

La queratosis actínica se considera una lesión preneoplásica que puede evolucionar


en un carcinoma basocelular.

En melanomas metastásicos, podría estar alterado el BRAF, lo que permite indicar


terapia de blanco contra inhibidores de BRAF.

Se denomina Síndrome de Sézary a la manifestación cutánea de los Linfomas de


grandes células B.

A diferencia del melanoma y el carcinoma basocelular, el carcinoma epidermoide no


tiene relación con la exposición a los rayos UV.

La posibilidad de presentar metástasis de un melanoma se predice midiendo, el


espesor de Breslow, considerando el recuento mitótico alto y la no inducción de la
respuesta inmunitaria local.

La alteración molecular más frecuente del carcinoma basocelular es la


pérdida/mutación de un gen supresor, el PTCH1 que regula la vía de señalización
Hedgehog,

Ante una paciente que presenta sangrado posmenopáusico, ¿cuál de estas


patologías debe sospechar en un primer momento por ser la causa más
frecuente? *

Adenocarcinoma de útero

Atrofia endometrial

Pólipos endometriales

Hiperplasia atípica uterina


En referencia a la entidad “Feocromocitoma”, señale cual de los siguientes
enunciados es CORRECTO. *

Las lesiones cardíacas evidenciadas se deben a la constricción de los vasos


sanguíneos miocárdicos provocada por las catecolaminas.

El 60% de los feocromocitomas son malignos, presentando infiltración local.

La presencia de abundantes mitosis, necrosis tumoral confluente y morfología


fusiforme, hablan de mayor agresividad y hacen diagnóstico de feocromocitoma
maligno.

Cuando son bilaterales se debe descartar la presencia de una Neoplasia endócrina


múltiple de tipo 1.
Un hombre de 70 años refiere dolor en la cadera y el muslo derechos de 8 meses
de evolución. En la exploración física se encuentra una reducción de la movilidad
en las dos caderas. No presenta hipersensibilidad o tumefacción a la palpación.
En la Rx de pelvis y pierna derecha se visualiza hueso cortical engrosado y
esclerótico con disminución del espacio articular en la cercanía del acetábulo.
Observe la siguiente imagen e indique cual de los siguientes trastornos es más
probable que produzca estos hallazgos. *

Artrosis degenerativa

Enfermedad de Paget

Osteocondroma

Hiperparatiroidismo
Un niño de 7 años tiene antecedentes de infecciones urinarias a repetición. Los
urocultivos identifican Escherichia coli y Proteus. En el examen físico se identifica
la desembocadura uretral en la superficie ventral del pene a 1,5 cm de la punta
del glande. ¿Qué término describe mejor a este trastorno? *

Epispadias

Balanitis

Hipospadias

Eritroplasia de Queyrat

En referencia a la “Patología vascular del SNC", señale cual/es de los siguientes


enunciados es CORRECTO. Recuerde que deberá marcar todas las opciones
correctas, considerando que al marcar opciones correctas de menos o 1
respuesta incorrecta, el sistema dará por anulado el ejercicio. *

Tras un traumatismo craneal menor es frecuente que en los ancianos y lactantes se


produzca un hematoma subdural.

Los infartos cerebrales por oclusión se relacionan con ateroémbolos provenientes de


la carótida interna.

Los infartos lacunares suelen ser pequeños, múltiples y se relacionan con la HTA.

El hematoma subdural se caracteriza por su rápida evolución constituyendo una


urgencia neuroquirúrgica.

Los hematomas subdurales suelen ser bilaterales y asentar en las caras mediales de
los hemisferios cerebrales.

La extensión del infarto dependerá del grado de obstrucción, calibre del vaso, estado
y grado de anastomosis de los vasos arteriales circundantes.
Paciente de femenino de 54 años de edad que consulta por tos y dolor pleurítico
durante las 3 últimas semanas. En la Rx. de Tórax se observa un área de
consolidación periférica de 2,5cm. que no responde al tratamiento antibiótico. Se
decide tomar una biopsia guiada por TAC. El estudio histopatológico informa:
“células cilíndricas atípicas dispuestas a lo largo de los tabiques alveolares” (ver
imagen). En algunos sectores se identifica la producción de una sustancia
eosinófila intracitoplasmática”. ¿Cuál es para Ud. el diagnóstico más probable? *

Adenocarcinoma in situ tipo mucinoso

Hiperplasia adenomatosa atípica

Mesotelioma maligno variedad mixta

Neumonía atípica
Usted es traumatólogo y está observando las radiografías de columna de una
paciente de 75 años mientras que la misma le relata que sufre de dolor articular
matinal que aumenta con la deambulación y cede con el reposo. Ud. se detiene
en ver la disminución del espacio articular con osteofitos marginales a nivel
lumbosacro. Recuerda cuando cursó Patología 2 y arriba a las siguientes
conclusiones: *

Se trata de un Osteosarcoma con claros indicios de invasión de partes blandas.

Se trata de la Enfermedad de Paget dado que es frecuente en ese grupo etario y


localización.

Se trata de una Fractura dado que la paciente padece osteoporosis.

Se trata de Osteoartritis degenerativa idiopática propia del envejecimiento.


Usted es un residente en la especialidad de Anatomía Patológica y es llamado por
el equipo de ginecología para ir al quirófano por una consulta intraoperatoria. La
paciente tiene 38 años de edad, es obesa y presenta un blastoma anexial
derecho de 11 x 6 x 5 cm, que en los estudios ultrasonográficos mostró áreas
hiperecogénicas en su interior. La superficie de la pieza que le entregan es lisa y
congestiva, y al cortarla nota múltiples áreas sólidas mientras rezuma un material
amarillento viscoso con algunos pelos (observe la imagen macroscópica). De
acuerdo a la macroscopía de la pieza, que resulta muy característica, usted ya
tiene un diagnóstico presuntivo que debe comunicar a los ginecólogos y
consignar formalmente por escrito. Respecto de esta entidad, responda: *

En general, se asocia a niveles elevados de CA-125.

Su extirpación genera el descenso inmediato de β-HCG.

Su comportamiento biológico suele ser benigno.

Es un tumor del cordón sexual.


Una niña de 10 años ha experimentado cefaleas de intensidad creciente al
levantarse durante el último mes. En la exploración se observa edema de papila
bilateral. Una RMN cerebral revela una masa sólida circunscripta de 3 cm en el
4to ventrículo. Se reconoce dilatación del 3er ventrículo y de los ventrículos
laterales. La masa es extirpada y al examen histológico muestra seudorrosetas
perivasculares con células tumorales regulares redondeadas dispuestas
alrededor de los vasos. ¿Cuál de las siguientes neoplasias es más probable en
esta paciente? *

Glioblastoma

Ependimoma

Astrocitoma

Meduloblastoma

Un hombre de 57 años ingiere una comida muy copiosa junto con abundante
cantidad de alcohol. Las molestias que comenzó a sentir le provocan fuertes
vómitos. Al poco tiempo desarrolla dolor en la parte inferior del tórax con
hematemesis. ¿Cuál de las siguientes alteraciones esofágicas es más probable
que tenga este paciente? *

Desgarro

Várices

Estenosis

Acalasia
Una paciente de 50 años acude a la consulta por un "bulto" a nivel del lóbulo
tiroideo derecho. Presenta dolor a la palpación y refiere haber tenido un episodio
gripal la semana anterior. Se decide punzar el nódulo, Ud. espera encontrar: *

Células gigantes multinucleadas, numerosos linfocitos y células foliculares con


características benignas.

Numerosos linfocitos, células de Hürthle y células foliculares con características


benignas.

Células gigantes multinucleadas, linfocitos, células epitelioides y material amorfo


compatible con caseum.

Numerosos linfocitos, abundantes células escamosas atípicas que en sectores


forman perlas córneas.

Paciente de 75 años de edad, ex tabaquista, con antecedentes de infarto de


miocardio y diagnóstico actual de insuficiencia cardíaca congestiva. Consulta
por disnea de reposo, intolerancia al decúbito y crepitantes en ambas bases
pulmonares, de horas de evolución. ¿Cuál es el patrón histológico pulmonar más
probable en este contexto clínico? *

Ingurgitación capilar alveolar, precipitado intraalveolar eosinófilo granular y


macrófagos cargados con hemosiderina.

Infección necrotizante de paredes de bronquios y bronquíolos con gran cantidad de


exudado polimorfonuclear neutrófilo.

Agrandamiento permanente anormal de los espacios aéreos distales al bronquíolo


terminal, acompañado por destrucción de sus paredes y sin fibrosis evidente.

Inflamación crónica de las vías aéreas a predominio linfocitario y agrandamiento de


glándulas secretoras de moco de tráquea y bronquios.
En referencia a los “Trastornos Mieloproliferativos crónicos”, señale cual de los
siguientes enunciados es INCORRECTO. *

La detección genómica del gen de fusión BCR-ABL no tiene implicación en el


diagnóstico y terapéutica de la Leucemia mieloide crónica.

A través de las vías RAS, JAK/STAT y AKAT e independientemente de los factores de


crecimiento, se genera proliferación de progenitores granulocíticos y
megacariocíticos.

La Policitemia Vera es una neoplasia mieloproliferativa en la que se ha descrito la


presencia de la mutación JAK2 en más del 90 % de los afectados.

Todos los Sindromes Mieloproliferativos pueden transformarse en Leucemias agudas


y una fase que se caracteriza por la fibrosis medular asociada a anemia,
trombocitopenia y esplenomegalia.

Respecto a la "Patología Prostática", señale cual/es de los siguientes enunciados


es INCORRECTO: *

Un PSA sérico mayor a 4 ng/ml es indicador altamente específico de patología


neoplásica maligna de origen prostático.

El incremento de la densidad ósea a nivel de un cuerpo vertebral en un hombre de 75


años es sospechoso de una posible metástasis por adenocarcinoma de próstata.

El sistema de Gleason sirve para graduar el cáncer de próstata en función de los tipos
histológicos. La estadificación se realiza en función del sistema TNM (tamaño del
tumor, ganglios linfáticos regionales comprometidos, metástasis a distancia).

La hiperplasia prostática benigna suele surgir en la parte central de la próstata,


comprimiendo a la uretra y generando sintomatología precoz debido a la obstrucción
del flujo urinario.
En referencia a los “Tumores Testiculares”, señale cuales de los siguientes
enunciados es CORRECTO. Recuerde que deberá marcar todas las opciones
correctas, considerando que el marcar correctas de menos o 1 respuesta
incorrecta, dará por anulado el ejercicio. *

Entre los tumores de células no germinales están los tumores de los cordones
sexuales-estroma gonadal y los linfomas no Hodgkin.

El seminoma es el tumor benigno más frecuente de testículo.

La mayoría de los tumores testiculares de células germinales se originan a partir de


una lesión precursora denominada neoplasia intratubular de células germinales.

El seminoma espermatocítico afecta predominantemente a varones adolescentes y


su pronóstico es peor que el seminoma clásico.

Un varón adulto que presenta ginecomastia hace pensar entre sus causas que se
trate de un tumor de testículo funcionante, productor de hormonas sexuales, como el
tumor de células de Leydig.

Cuando un tumor de testículo marca positivamente con alfa feto proteína hace
sospechar que se trate del carcinoma embrionario.

Atrás Enviar

Nunca envíes contraseñas a través de Formularios de Google.

Este formulario se creó en Facultad de Medicina. Noti car uso inadecuado

Formularios

También podría gustarte